You are on page 1of 308

LESSON 1

LIMIT OF A FUNCTION

The term function denotes functional relationship between two or more quantities. Functional relationship
means if one quantity is given, other can be determined. In a demand function stating relationship
between price and quantity demanded, if price is given, quantity demanded can be calculated. Similarly in
the total cost function stating relationship between total cost and units produced, if units produced are
given, the total cost can be determined. The generatlised forms of functions are y = f (x), y =  (x), y =
g(x).
A function consists two things—constants and variables as in a demand function x = 100 – P2.
Constants are those quantities whose value is fixed during certin mathematical process. Constants
are classified as absolute and arbitrary. Absolute constants are those quantities whose value is fixed in all
processes whereas arbitrary constants are those whose value may differ in different processes. Arbitrary
constants are denoted by first alphabets as a, b, c etc. On the other hand, variables are those quantities
whose value vary within a mathematical process as P and x in the demand function. These variables are
usually denoted by latter alphabets as x, y, z etc. These are also classified as dependent and independent
variables. Independent variables are those variables which can take any arbitrary value. Its value is not
determined by any other factore. The limit within which its value vary is known as domain of the
function. Dependent variables are those whose value is determined by the value of independent variable.
The limit within which dependent variable can take values is known as range of function. Almost all the
functioons used for stating economic and business application have positive domain as well as range.
All the functions used in mathematics are classified on different basis. One of the most important
classifications in the algebraic and non-algebraic functions. Algebric functions are those functions in
which dependent variable is related with independent variable by using mathematical operatiosn only as
addition, multiplication, substraction, division, power and root. For example x = 40 – 2P, TC(x) = 1000 +
5x, x = 250  P 2 are algebraic functions.
Such functions are most widely used in business and economics. All other functions are known as
non-algebraic or transcendental functions. These are classifiedas trignometrical, inverse trignometrical,
logarithmic and exponential functions.
Trignometrical functions are those where independent variable appears with some trignometrical
ratio as y = sin x. Inverse trignometrical functions involve inverse of trignometrical ratios as y = tan–1x. It
is equal to x = tan y which is a trignometrical ratio. These functions are used iin science.
Exponential functions are defined as those functiosn where independent variable appears as power
of some constant base as y = ex where e is a constant equal to 2.7183 (it is the base of natural log also
known as napierian log). This function is based upon certain practical assumptions.
Logarithmic functions are those where independent variable appears with log ratio as y = log x or y
= log10x. These functions are related with exponential functions. They are taken as inverse of the
exponential function.
In addition to this, there are a number of other classifications of functions as continuous and
discontinuous functions, explicit and implicit functions. Univariate, bivariate and multivariate functions
etc.

Meaning of Limit
Function denotes the relationship between dependent and independent variable as y = f(x), y is a
dependent variable and x is an independent variable. An independent variable may be assigned value in
two ways viz. a fixed value as x = a or tending towards certain constant as x  a. Value of the function
when independent variable is assigned a fixed value is known as defined value of the function, but it is
known as limit of the function if independent variable is tending towards certain constant. The implication
is the x is not equal to a.
Mathematically limit of a function f(x) when x  a is denoted by lim f  x  .
x a

It can be defined as limit of a function f(x) for a given value of the argument x is that constant l to
which the function continuously approaches as x approaches the given value such that the difference
between constant and the function may be made as small as we would lke by making the argument x
approaching sufficiently near to the assigned value a. Symbolically, f(x)  l as x  a but |(x – a)| > 0. In
other words, the function f(x) is said to tend to the limit l as x tends to a if  > 0 is chosen arbitrarily
small and it is possible to find  > 0 such that whenever |x – a| < , | f(x)–l | < , i.e., f(x) can be made as
close to l as we would like by taking x sufficiently close to a. It is denoted as f(x)  l as x  l which
means f(x) is l as x tends to a.
Although it is one of the most difficult concept in mathematics, it is generally used in practice. It is
used to describe all those situations where target is such that it is possible to reach near the target, but it is
not possible to achieve the target. In other words it is possible to reach near the point, but it is not possible
to touch that point. For example, achieving optimum leave of installed capacity or achieving optimum
level of production are instances of limit.
The entire mathematical system of differential and integral calculus is based on the concept of limit
of a function and related conditions of continuty of a function. Both ideas are fundamental and occupy an
important position in a formal course on Calculus.

Evaluating Limit of a Function


One the basis of the definition of limit, limit of a function can be obtained by putting x = a + h where h 
0 or x = a – h where h  0. h is a constant introduced to avoid the equality of x and a. This method can be
applied in all conditions and is known as evaluating limit by first principle. It is the direct method of
evaluating limit of a function.
x2  1
Example. Evaluate lim f  x  =
x 1 x 1
Solution. Alpplying direct method, let x = lim 1  h  .
h0

1  h  1
2

 lim f  x  = lim
x 1 h0 1  h 1
1  h 2  2h  1
= lim
h0 h
= lim  h  2 
h0

= 2 Ans.
xn  an
Example. Evaluate lim f  x  =
xa xa
Solution. Let x = lim  a  h 
h0
 a  h  an
n

 lim f  x  = lim
xa h0 aha
 n n 1 n  n  1 n 2 2 
 a  na h  a h ......  hn   a n
= lim
 2 
h0 h
[Applying binomial expansion]
n  n  1 n 2 2
na n 1h a h  ......  hn
= lim 2
h0 h
= na n–1 [as all the successive terms involve h
which is tending towards zero]
But this method can not be applied in all conditions. In those conditions, other methods based on
this are preferred.

Properties of Limit
This limit has various properties. These are used for evaluating limit. All these are examplified below:
1. Limit of a constant is always equal to constant, i.e., lim k = k. For example, lim 5 = 5. It is
x a x 3

because it does not involve any term of x.


2. Limit of the sum (or difference) of two functions is equal to sum (or difference) of their limits
i.e., lim  f  x   g  x   = lim f  x   lim g  x 
xa xa xa

where f (x) and g(x) are two different functions. It can be extended to sum or difference of any
 n 
lim   fi  x   =  lim fi  x  .
n
number of functions as
xa
i 1  i 1
xa

For example lim [(2x + 3) + 3x2] = lim (2x + 3) + lim 3x2 = 13 + 75 = 88.
x 5 x 5 x 5

3. Limit of the product of two or more functions is equal to the product of their limit
i.e., lim [f (x) . g(x)] = lim f (x) . lim g(x)
x a x a x a

For example lim [(x3) . (2x + 5) = d (x3).


x 2

lim (2x + 5) = 8 + 9 = 17.


x 2

4. Limit of the quotient of two functions is equal to quotient of their limits provided the limit of
divisor is not equal to zero.

 f  x  lim f  x 
xa
i.e., lim   = where lim g(x)  0.
x  a g  x x a
  lim g ( x)
xa

 x2  2  lim  x 2  2  6
x2
For example lim   =  6
x2
 x 1  lim  x  1 1
x2

5. Limit of the nth power of a function is equal to nth power of the limit,
n
i.e., lim [f(x)n] =  lim f  x  
x a  x  a 
2
For example, lim (2x + 3)2 =  lim  2 x  3  = 49.
x 2  x  2 
6. Limit of the nth root of a function is equal to nth root of the limit of a function
i.e., lim  n f  x   = lim f  x 
 
n
xa xa

For example lim  4 x  = 4 lim x  4 16


x  16 x  16

= 2.

Methods of evaluating limit of a function


This limit can be evaluated under three different conditions, viz., when x tends towards a definite quantity
as x  2 or x  –5; when x tend toward zero and when x tends towards infinity. There are different
methods which can be used for evaluating under different conditions. (all these are over and above
evaluating limit by direct method). These cases are discussed below separately.
(a) When x tends towards a definite quantity : When independent variable tends towards a
definite quantity which may be positive or negative, the limit can be evaluated by any of the following
methods.
(i) By putting x = a : It is the casiest method of evaluating limit. It gives defined value as limit of the
function. But it can be used only when given funtion is a polynomial function or a rational function and
denominator is equal to zero for x = a. In addition to this the method can be used only if putting x = a
does not give any indeterminant form explained in the note below.
4 x4  3x2  1
Example. Evaluate lim f (x) =
x 1 x2  7
Solution. Since f (x) is defined for x = 1, so putting x = 1,
4 1  3 1  1
lim f (x) =
x 1 1 7
6 3
= 
8 4
Note: Although this method is easy, but it can not be used if limit comes in indeterminant form. It is
0  b
taken as in indeterminant form if it comes in any of these form, viz., , or where b is a constant.
0  0
Limit is not determinable. If the defined value of a function takes any of these forms, it also amount to
defined value as indeterminable. These results are not defined in mathematics.
(ii) By factorisation method : This method is used when given function is a rational function. It
involves factorising numerator and denominator to the extent possible, cancelling the common factor and
putting x = a in the simplified form.
2 x2  7 x  6
Example. Evaluate f (x) = when x  2.
5 x 2  11x  2
Solution. When x = 2, function is not defnined, so applying factorisation method
 2 x  3 x  3
lim f (x) = [Factorisation]
x 2  5 x  1 x  2 
2x  3
=
5x  1
Putting x = 2
1
lim f (x) = Ans.
x 2 9
x3  5 x 2  2 x  2
Example. Evaluate lim f (x) = 3
x 1 x  2x2  6x  3
Solution. When x = 1, f (x) is not defined, so factorising the rational function.
 x  1  x 2  4 x  2 
 x  1  x 2  3x  3
f (x) =

x2  4x  2
=
x 2  3x  3
Putting x = 1
1  4 1  2
lim f (x) = = –5
x 1 1  3 1  3
xn  an
Example. Evaluate f ( x) =
xa xa
Solution. Using factorisation method
 x  a  xn1  xn2 a  xn3a 2 .....  a n1 
f (x) =
xa
= xn–1 = xn–2a + xn–3 a2 ..... + an–1
Putting x = a
lim f (x) = an–1 + an–1 + an–1 + ..... n times
x a

= nan–1.
(iii) By rationalisation method: This method is used when the given function is in the irrational
form. The method involves rationalisation of the numerator or denominator as the case may be
simplifying the function after rationalisation putting x = a in the simplified form. The purpose of
rationalisation is to simplify by rationalisin the numerator or denominator as the case may be.
Example. Evaluate x2  4
f ( x) =
x2 x  2  3x  2
Solution. When x = 2, f (x) is not defined, so rationalising the denominator.
x2  4 x  2  3x  2
f (x) = 
x  2  3x  2 x  2  3x  2
x 2
 4   x  2  3 x  2 
=
x  2  3x  2
[(a – b) (a + b) = a2 – b2]

 x  2  x  2   x  2  3x  2 
=
2  x  2 
 x  2   x  2  3 x  2 
=
2
Putting x = 2
4  2  2
lim f (x) =
x 2 2
= – 8.
x2  4
Example. Evaluate lim f (x) =
x 2 x 2  x2
Solution. Since f (x) is not defined for x = 2, Rationalising the denominator

x2  4 x   2  x  2 
f (x) = 
x  2  x2 x   2  x  2 

=
 x2  4  x   2  x  2 
x  2  x  2  2 2. x  2
x  2 x  2  x   2  x  2  
=  
2 2. x  2
x  2  x   2  x  2  
=  
2 2
Putting x = 2
2  2   2  
lim f (x) =  
x 2
2 2
= 2.
(b) When x tends towards zero: In case x  0, same methods can be applied for evaluating limit
depending upon nature of function.
x 2  2ax  a 2
Example. Evaluate lim f (x) =
x 0 a3
0  2a  0   a 2
=
a3
1
= .
a
x
Example. Evaluate lim f (x) =
x 0 1 x  1 x
Solution. Rationalising the denominator,
x 1 x  1 x
f (x) = 
1 x  1 x 1 x  1 x
x  1  x  1  x 
=
1  x  1  x 
1 x  1 x
=
2
2
Putting x = 0 = = 1.
2
(c) When x tends towards  : In this case, limit can be evaluated by dividing all the terms of
numerator and denominator by the term having maximum power among all the terms of numerator and
denominator taken together. It is in the case of rational functions. After this put x = , now all the terms
having x in the denominator will be zero.
x3  3x 2  6 x  5
Example. Evaluate lim f (x) =
x x3  2 x  6
Solution. Term having maximum power is x3; so dividing numerator as well as denominator by x3
x3  3x 2  6 x  5
f (x) = x3
x3  2 x  6
x3
3 6 5
1  2  3
= x x x
2 6
1 2  3
x x
Putting x = 
3 6 5
1
 
lim f (x) =   
x 2 6
1 
 
= 1. [Any number divided by infinity is equal to zero]
1  2  3  4  .....n
Example. Evaluate the limit of the following sequence where n  .
n2
Solution. The numerator is forming an arithmetical progression involving first n numbers, so their
sum is given by
n n  n  1
Sn =  2a   n  1 a  or
2 2
in this case
n  n  1 n2  n
 lim f (n) = 
n 2n 2 2n 2
Dividing both sides by n2
1
1
f (n) = n
2
Putting n = 
1 1
 lim f (n) = [ = 0 as n = ]
n 2 n
Example. Evaluate the limit of the following sequence a + ar + ar2 ........ n
where n   if r > 1, r = 1, r < 1.
Solution. The sequence is forming a geometrical progression with a as the first term and r as the
common ratio
 r n  1
 Sn =   if r > 1
 r 1 
= na if r = 1
1  r n 
= a  if r < 1
 1 r 
 a  r n  1 
lim Sn =   = [as rn is if  n  ]
n  r  1 

lim Sn = lim [na] = 


n n

 a 1  r n  
But lim Sn = lim  
n n
 1  r 
 a   ar n 
= lim   lim  
n  1 r 
  n   1  r 
a
[because rn is approximated as zero when n  
=
1 r
as r < 1]
a
So limit exists only when r < 1 and is equal to .
1 r
Interpretation of e in limit form
e is a constant taking approximate value of 2.7183. It is the base of natural log system also known as
n
 1
Napierian log. This e is defined in terms of limit as lim 1   = e.
n n  
Example. Evaluate lim 1  x  1/ x

x0

Solution. Let x = 1 , then if x  0, y must tend towards , only then they are equal so substituting
y
x = 1 in the function.
y
y
 1
lim 1  x  = lim 1  
1/ x
y
x0
 y
= e (by definition)
x 5
 1
Example. Evaluate lim 1  
x x  
x 5 x 5
 1  1  1
Solution. lim 1   = lim 1   . lim 1  
x
 x x

x x  x  
(Applying law of indices)
  1
5

= e . 1  lim  1    1
 x    x  
= e.
3x
 1
Example. Evaluate lim 1  
x
 x
3x x x x
 1  1  1  1
Solution. lim 1   = lim 1   . lim 1   . lim 1  
x
 x x
 x x x x x
= e.e.e
= e3 .
x
 2
Example. Evaluate lim 1  
x
 x
Solution. Let 2  1 or x = 2y if x  , y  
x y
x 2y
 2  1
lim 1   = lim 1  
x
 x y
 y
y y
 1  1
= lim 1   . lim 1 
y
 y y y 
= e.e.e
= e2.

APPLICATIONS OF LIMIT IN BUSINESS AND ECONOMICS


In has been described in the meaning, limit is a very practical concept. The entire theory of differential
and integral calculus is based on this concept which is turn have large applications in business and
economics. Not only this, it is used for testing the continuity of functions.
The concept limit is directly and indirectly applied in business and economics. Almost all
applications are based either on the interpretation of e or geometrical progression with common ratio less
than unit. The important applications related with e are continuous compounding and related with
geometrical progressions are explained in other applications.

1. Continuous compounding and discounting


As stated above, this application is based on e interpretation. It involves compounding or
discounting.
In case of compounding involving interest on interest
Amount = P (1 + r)t
where P is principle, r is rate of interest in relation to 1 and t is time in years.
It gives amount due after t years if interest is compounded yearly.
But if interest is compounded half-yearly effective rate is half and time is doubled.
2t
 r
A(2) = P 1  
 2
where A(2) denotes amount after interest compounded twice a year.
If interest compounded quarterly—
4t
 r
A(4) = P 1  
 4
If interest compounded monthly—
12 t
 r 
A(12) = P 1  
 12 
If interest compounded m times—
mt
 r
A(m) = P 1  
 m
If interest compounded continuously then number of times interest compounded will tend to infinity;
and
 mt

A(m) = lim  P 1  
r

m
  m 
  m rt 

=  lim 1     m 
r r
m
 m   r .rt  mt 
  
  
rt
 m

  r  r


 m    m  
= P lim 1
 
 m


as lim 1  r 
 e
r
= P. en
 m    m  
 
So if interest is compounded continuously A = Pen where e is a constant. The value of en are
provided by readymade tables known as exponential table giving values of en corresponding to rt.
Example. Determine the amount at the end of 4 years of an investment of Rs. 1000 if the annual
interest rate is 6 per cent being compounded
(i) annually
(ii) half-yearly
(iii) continuously.
Solution. (i) Amount = P (1 + r)t
= 1000 (1 + 0.6)4
= 1000 (1.2626) [From compound interest table]
= Rs. 1262.50
2t
 r
(ii) Amount = P 1  
 2
= 1000 (1 + .03)8
= 1000 (1.2668) [From compound interest table]
= Rs. 1266.80
(iii) Amount = pen
= 1000 (e) (.06)4
= 1000 e.24
= 1000 (1.2712) [From exponential table]
= Rs. 1271.20
In case of discounting, it is just the reverse of compounding, r becomes rate of discount; so P = A (1
2t
 r
+ r)–4 if discounting is annually, if it is half yearly P = A 1  
 2
Therefore if discounting is m times
 mt
 r
p(m) = A  1  
 2
and when it is continuously: the number if times, m is tending towards .
  mt

present value = lim  A 1   
r
So
m
  m  
 rt
 m

  r  r


 m    m  
= A lim 1
 
= Ae–rt
Value of e–rt are providing by exponential table according to rt.
Example. Mr. X has borrowed certain money for 3 years on the condition that he will pay Rs. 500 at
the end of 3 years. Find the money borrowed if interest compounded (i) annually, (ii) continuously at 8
per cent.
Solution. Let he borrows Rs. P, then
(i) A = P (1 + r)t
or, P = A (1 + r)–t
= 500 (1 + .80)–3
= 500 (0.7938) (From present value table)
= Rs. 396.90.
(ii) P = Ae–rt
–(0.8) (3)
= 500e
–.24
= 500e
= 500 (.786628) (From exp. table)
= Rs. 393.32 app.

2. Measurement of rate of growth


It is also based on e. It is used for the measurement of rate of growth of various economic
phenomenon which change continuously as population, prices etc. In such cases
A = Per
If A, P and t are given r can be obtained which is the rate of growth.
A
 ert =
P
It can be simplified in two different ways.
(i) A/P is a constant, and is equal to ert. Find value of rt corresponding to A/P in the exponential
table. Say it is x
then rt = x
x
r = .
t
(ii) Take natural log of both sides
 rt ln(e) = ln  A  [ln is denoting naturallog]
P

or rt ln = ln  A  [as lne = 1. log of any number with same


P
base is alwyas 1].
 A
P
r = ln   [value of ln  A  can be obtained from
t P
natural log table]
Example. Population of Delhi has increased from 40 lacs during 1970 to 80 lakhs at the end of
1980. Find the rate of growth in population.
Solution. Population growth is continuous
 A = Pert
or 80 = 40e10
or e10 = 2
Simplifying it by both the methods
(i) In the exponential table ert is equal to 2 when rt = .695 app
 10 r = .695
r = .695
or 6.95 per cent
(ii) Taking natural log of both sides
10 r = ln2
r = = 6.93 per cent app.
3. Other applications
These applications are based on the limit of sum of infinite terms of a geometrical progression being
less than unit. These are explained below.
Example. What will be the present value of Rs. 1,000 flowing every year at 10% rate of discount for
an infinite number of year.
1000 1000
Solution. Total present value =  ........
1  .10 1  .012
1000  1 1 
= 1   ........ 
1.10  1.10 1.10 2 
1
It is forming a geometrical progression with a = 1 and r = which is less than one.
1.10
 
1000  1 
 Total present value =  
1.10 1  1 
 1.10 
1000
= = Rs. 10000.
.10
Example. Mr. X has made an initial investment of Rs. 1,000. What will be total investment after
1
infinite number of years if of the previous years investment is invested again and again.
10

2
1
Solution. Total investment = 1000 + 1000  1  + 1000   ..........t
10  10 
where t  
 t 1 
i
= lim 1000   
t 
 i 1 10  

 2

= 1000 1        ........... 
1 1 1
 10  10   10  
1
It is forming a geometrical progression with a = 1 and r =
10

= 1000 
1 
1 
1  
 10 
10000
= Rs. .
9
Example. A firm has purchased a machinery for Rs. c being depreciated at r per cent per annum on
diminishing balance method. Using limit, show that sum of depreciation of all the years is equa to initial
cost. (r may be used as in relation to 1.)
Solution. Total depreciation = cr + (c – cr) r + [(c – cr) – (c – cr)r]r + .......t year
= cr [1 + (1 – r) + [(1 – r) – (1 – r)r] r.......I year]
= cr [1 + (1 – r) + (1 – r)2 ........t years]
Total depreciation = [cr [1 + (1 – r) + (1 – r)2 ...........t]
[1 + (1 – r) + [(1 – r)2 ...........t years is forming a geometrical progression with a = 1 and r = (1 – r)
which is less than one]
 1 
So total depreciation = cr  
 (1  (1  r ) 

CONTINUITY OF FUNCTIONS
Functions can also be classified as continuous and discontinuous functions. In simple word a
function is taken as continuous if its graphical presentation gives a smooth curve. The interval for which it
gives a smooth curve. Otherwise, function is known as discontinuous.
The continuity of a function can be tested with the help of limit concept. There are two possible
cases in this situation, viz., continuity at a point and continuity in an interval. Both the situations are
explained below.

Continuity at a point
It involves testing the continuity of a function at a particular value of x. It is taken as continuous at a
point of the following conditions are fulfilled:
(a) Defined value exists at that point, i.e., f(a) exists.
(b) Limit at that point exists i.e., lim f(x) exists.
x a

(c) Both defined value and limit are equal, i.e., f(a) = lim f(x)
x a

If any of these conditions are not fulfilled, the function is taken as discontinuous. There are different
types of discontinuities based on the condition not fulfilled. It is taken as missing point discontinuity if
defined value does not exist. If limit does not exist at that point, the discontinuity is taken as finite
discontinuity. But if both does not exist, the discontinuity is known as infinite discontinuity.
 x3  1
 if x  1
Example. Given f(x) = x  1

 3x if x  1
Determine whether function is continuous at x = 1.
Solution. For continuity of the function,
(1) Definied value, f (1) = 3(1) = 3.[as f(x) = 3x when x = 1]
 x  1  x 2  x  1
(2) Left hand limit of f (x), lim f(x) =
x 1 x 1
= 1+1+1=3
Right hand limit of f(x), lim f(x) = 3 (1) = 3
x 1

So both left hand and right hand limit are existing and equal, lim f(x) = 3.
x 1

(3) Both defined value and limit are existing and equal, therefore, function is continuous at x = 1.
x 2  3 x  10
Example. Given f(x) – for x  2.
x2
What value should be assigend to f(2) to make f(x) continuous at x = 2.

Solution. lim f(x) =


 x  2  x  5  [By Factorising]
x 2 x2
= (2 + 5) = 7
So f(2) must be equal to 7 for making f(x) continuous at x = 2. Only then both defined value and
limit will be existing and equal.

2. Continuity in an interval
The limit concept can also be used for testing the continuity of a function in an interval. This
interval may be open or close. Hence continuity in an interval may be
(a) continuity in an open interval
(b) continuity in a close interval.
(a) Continuity in an open interval : An open interval is defined as that interval in which end points
are not included. All the values in between end points are included. It is denoted by (a, b). So continuity
in the open interval of (a, b) amounts to testing continuity at all points in between a and b excluding a and
b. If the function is continuous at all the points in between a and b, it is taken as continuous in the given
interval, otherwise not.
Testing of continuity in the open interval is same as testing at a point because at each point, function
should fulfill all the three conditions for continuity. It may be applied in the form of taking some points in
the given interval and testing the continuity at each point separately.
(b) Continuity in a close interval : A close interval is that interval in which end points are also
included. It is denoted by [a, b] read as close interval of a and b. The continuity of the function in this
close interval requires three conditions:
(i) Right hand continuity at the point a
(ii) Continuity in the open interval of (a, b)
(iii) Left hand continuity at the point b.
(i) A function is taken as having rith hand continuity at the point a if the following three conditions
are fulfilled.
(a) Defined value of f(x), i.e., f(a) is existing.
(b) right hand limit of f(x) i.e., lim f(x) is existing.
x  a

(c) f(a) = lim f(x)


x  a

If any of these conditions is not fulfilled, the function is taken as discontinuous at x = a.


(ii) Continuity in the open interval of (a, b) is same as explained earlier.
(iii) Left hand continuity at this point b will be existing only if:
(a) Defined value of f(x), i.e., f(b) is existing
(b) left hand limit of f(x), i.e., lim f(x) is existing
x  b

(c) both are equal i.e., f(b) = lim f(x)


x  b

It implies that if a function is continuous in the close interval, it is always continuous in the same
open interval. (as it is one of the conditions for continuity in the close interval). But if a function is
continuous in the open interval, it may not necessarily be continuous in the close interval. It may be that
function which have only one side continuity either left hand side or right hand continuity.

Continuous Functions
There are some functions which are always in its domain. If a function appears in any of these
forms, it can be directly stated that function is continuous without applying tests. These are as follows:
(a) Polynomial function : A function appearing in the form f(x) =a0xn + a1xn–1 + a1xn–2 + ..... + anx0
where n is a non-negative integer, is known as polynomial function of nth degree. Such functions are
always continuous for all the interval because defined value and limit at all points exist and are equal. So
if the given function is a polynomial function, it can be taken as continuous in the given interval.
(b) Rational function : A rational function appearing in the form of division of two functions is
taken as continuous for all the interval if it does not include any point where denominator is equal to zero.
If it includes any point where denominator is equal to zero, neither defined value nor limit will exist, so
function will be discontinuous.
It can be applied in the form as to put the denominator of rational function equal to zero and solve
for x. If the given interval consists of that value of x function is taken as discontinuous, otherwise
continuous.
x3  1
Example. Given f(x) = , determine all values of x for which f(x) is continuous.
x2  9
Solution. The given f(x) is a rational function, so putting denomination x2 – 9 = 0, x = ± 3 so f(x) is
continuous for all intervals excluding –3 and 3.
Example. Given f(x) = d X  R.
Determine its continuity in the interval (–, –3), (–, –3), (3, +), (3, +), (–3, 3).
Solution. The given function f(x) is defined for all values except for less than 3 and greater than –3
excluding both. For all values in between –3 and +3, it becomes root of negative which is not defined in
real number system. So limit will also not exist and function is discontinuous for value between –3 and 3.
According to the given intervals.
(i) (–, –3), function is continuous
(ii) (–, –3), function is continuous
(iii) (3, +), function is continuous
(iv) (3, +), function is continuous
(v) (–3, 3), function is discontinuous.
1
Example. Let G(x) = , 2, x  R
x2
Tell for what interval the function G(x) is continuous.
Solution. If x is a real number denominator it is defined only wnen x > 2.
 G(x) is Continuous for the interval (2, )

APPLICATIONS IN BUSINESS AND ECONOMICS


The same continuity of function is also relevant is business and economics. It can be directly applied
for testing whether a function is continuous at a point or in given interval. In addition to this, it is
necessary that a function must be continuous for differentiation which it turn has number of applications.
Some are explained below.
Example. An electric company changes from its customers the following amount for services; Rs.
3.00 for the first 20 kilowatts hours or less, Rs. 0.10 per kilowatt hour for the next 80 kilowatt hours; and
0.08 per kilowatt hour for any hours above 100 kilowatt hours. Determine the total cost function of x
kilowatt hours, TC(x) and test its continuity at x = 20 kilowatt hour and x = 100 kilowatt hours.
Solution. Accordingly,
3 if 0  x  20

TX(x) = 3  .10  x  20  if 20  x  100
11  .05  x  100  if x  100

At x = 20
Defined value, TC (20) = 3
lim TC(x) = 3 (comparing left hand and right hand limit)
x  20

So both are existing and equal, TC(x) is continuous.


At x = 100
Defined value, TC(100) = 11
lim TC(x) = 11 (comparing left hand and right hand limit)
x  100

d Function is continuous at x = 100.


Example. The total cost function of a firm stating total cost at different levels of output is defined as
 20 x if 0  x  100
TC(x) = 18 x if 100  x  200
16 x if x  200
Test wheter the function is continuou in the intervals (100, 200), (100, 200), (100, 200).
Solution. For open interval (100, 200), TC(x) is defined as 18x which is polynomial function so it is
continuous.
(ii) For the close interval (100, 200), TC(x) is defined as 20x for x = 100 but its right hand limit is
given 18x, so it is not continuous at x = 100 and function is discontinuous for this interval.
(iii) For the interval (100, 200), TC(x) is defined as 18x which is continuous.
LESSON 2

INDEFINITE INTEGRATION

Calculus is an important branch of mathematics. It is broadly classified as differential and integral


calculus. Differential calculus gives the derivative of a function used as an instantaneous rate of change.
Integral calculas on the other hand, deals with the total rate of change.
Integral calculus is reverse of the process of differentiation (also known as antidifferentiation) had
its own applications in business and economics. Two important applications are obtaining the original
function from its derivative function and the measurement of area under curve.
The integral of a function is denoted by sign  (known as integral). Integration is also related to a
variable just like differentiation. So function is alwyas followed by dx or du or any variable in relation to
which function is integrated. Integration of a function can be technically defined as reverse of
differentiation. It is as follows:
dy f  x  x   f ( x)
Let y = f(x) and its derivative, = lim
dx x  0 x
dy
Putting it equal to f(x) = f(x)
dx
Then f(x) . dx = f(x) + c.
In this, left hand side denotes the integral of f(x) with respect to x and right hand side states it as
equal to f(x) + c, where f(x) is the original function and c is an arbitrary constant. The same is know as
integration and the result obtained is described as indefinite integral of f(x) with respect to x. So indefinite
integral is used for obtaining the original function from its derivative.
Integration is reverse of the process of differentiation. In case of differentiation, the power of
variable is multiplied in the base and the power is reduced by one for algebraic function. So integral with
involve increasing the power of the variable by one and dividing by the increased power. For example,
if y = x2 + 2x – 5,
dy
= 2x + 2 or dy = (2x + 2) dx
dx
Then (2x + 2) dx = + 2x + c
= x2 + 2x + c.
where c is arbitrary constant. This provides for that constant which has been ommitted by
differentiation. The value of c can be obtained under certain conditions as in the above example if y value
is given for certain value of x.

Different Methods of Integration


There are different methods of integration used in different conditions. Some of these are as
discussed below:
(a) Simple integraiton. It just involves the reversing process of differentiation for obtaining
integral. It is erxplained below in the context of algebraic functions, exponential and logarithmic
functions.
(i) Algebraic function : If the function to be integrated is an algebraic function, the itnegral can be
obtained by increasing the power of variable by one and dividing the variable by increased power. Some
standard forms of simple integration are as follows:
1. If dy = f(x)dx,  f ( x)dx = f(x) + c
2. If dy = a [f(x)dx] where a is a constant,

 af ( x)dx = a  f ( x)dx  af ( x)  c


3. If dy = f1( x)  f 2( x) dx 
 f ( x)  f ( x)dx
1 2 =  f ( x)dx   f ( x)dx
1 2

= f1(x) + f2(x) + c.
x n 1
4. If dy = xn . dx,  x .dx  n  1  c .
n

Example. Find the integral of the following:


(i)  xdx (ii)  dx (iii) 
3
x2

1 1 
 x    x  2
2
(iv)  4  dx (v) dx
x
x2
Solution. (i)  x.dx  2
c

(ii)  dx  1.dx  x  c
2
1
x3
 x dx  c
2 2 2/3
(iii) x .dx =
2
1
3
3 5/3
= x c
5
1 1 
 x  x  x1/ 4  dx
4
(iv)  4  dx =
x
x 3 x 3 / 4
=  c
3 3/ 4
x 3 4 3 / 4
=  x c
3 3

  x  2 x  4 x  4 dx
2 2
(v) dx =

x3
=  2x2  4x  c
3
(ii) Logarithmic function : If a logarithmic function where independent variable appears with a log
dy 1
ratio is given as y = lnx (ln is denoting natural log with the base e). Its derivative,  , so the integral
dx x
1 1
of with respect to x is lnx or  dx = lnx + c.
x x
Therefore, it can be generalised that whenever in a rational function, numerator is the derivative of
the denominator or is equal to some constant multiplied by numerator, the integral of the function is the
1
natural log of denominator. For example in  dx , 1 the numerator is the derivative of the x appearing in
x
denominator; therefore, its integral is lnx + c.
Example. Evaluate the following:
4x  6 x2  2 x x3  x
(i)  x3  3x  1 dx (ii)  x3  3x 2  5 dx (iii)  x 4  2 x 2  2 dx
Solution. (i) Let y = x2 + 3x + 1,
dy
 = 2x + 3
dx
It shows that numerator is 2(2x–3); so applying log form:
4x  6  2 x  3 . dx = 2ln(x2 + 3x + 1) + c.
x 2
 3x  1
. dx = 2  2
x  3x  1
(ii) Let y = x3 + 3x2 + 5.
dy
 = 3x2 + 6x
dx
1
It show that numerator is 3x 2  6 x 
2
Applying log form.
x2  2x 1 3x 2  6 x
 x3  3x 2  5 . dx =
3  x3  3x 2  5
. dx

in  x3  3x 2  5   c
1
=
3
(iii) Let y = x4 – 2x2 + 2,
dy
 = 4x3 – 4x
dx
1
It shows that numerator is  4 x3  4 x 
4
So applying log form:
x3  x 1 4 x3  4 x
x 4
 2x  2
2
. dx = 
4 x  2 x2  2
4
dx

= ln [x4 – 2x2 + 2] + c.
(iii) Exponential function : All those functions where independent variable appears as a power of
constant base, are known as exponential function. If the base is equal to e, the function is known as
dy
natural exponential function as y = ex. If derivative,  e x . So its integral,  e x .dx  e x  c
dx
dy
Similarly if y = ebx; = bebx and  b.e x dx  ebx  c
dx
Example. Evaluate the following :

 abe (ii)  4e x . dx   2e  e x  dx


bx 3x
(i) . dx (iii)

du du
Solution. (i) Put u = bx, = b or dx =
dx b
Making this substitution :
du
 abe . b   ae du
u u
= a  eu . du

= aeu + c
= aebx + c.
du
(ii) Putting u = –x, = –1 or dx = –du.
dx
Making this substitution :

 4e
x
. dx   4eu .  du  = 4 eu . du
= –4eu + c
= –4e–x + c.

  2e  e x  dx =  2e . dx   e x . dx
3x 3x
(iii)

2e 3 x x
= –e + c.
3
(b) Integratio by substitution : In case of some function, especially for composite functions, the
integral can be obtained by making substitution. For example,   f  x  dx can be obtained by
n

du [because if u = f(x), du = f(x), or dx = du


substituting u = f(x), an dx = ]
f ( x) dx f  x
du 1  u n 1 
u . 
n
  + c. f(x) must be a constant, otherwise it is not possible. The substitution
f   x  f ( x)  n  1 
is made in such a way that the integral can be simplified to the possible extent.
Example. Find the integral of the following :

 3x  4 . dx  x  5  3x 
28
(i) (ii) dx

x2  2 x e3 x
(iii)  . dx (iv)  . dx
x3  3x 2  1 1  2e 
3 x2

Solution. (i) Let u = 3x + 4,


du du
 = 3 or dx = .
dx 3
Making this substitution,
du 1
 3x  4 . dx =  n.
3 3
 u . du

1 u 3/ 2
= +c
3 3/ 2
2
=  3x  4  + c.
3/ 2

9
du du
(ii) Let u = 5 + 3x2,  = 6x or dx =
dx 6x
Making this substitution

 x  5  3x 
du
2 8
 x. u  . 6 x
8
. dx =

1
  u  . du
8
=
6
1 u9
= +c
6 9

=
 5  3x  2 2

+ c.
54
du
(iii) Let u = x3 + 3x2 + 1,  = 3x2 + 6x
dx
du
or dx =

3 x  2x
2

Making this substitution :
x2  2x x2  2x

du
x3  3x 2  1
.dx =  u . 3 x2  2 x 
1 1
3 u
= .du

1 u1/ 2
= +c
3 1/ 2

=
2
3
x 3
 3x 21 + c.

du
(iv) Let u = 1 – 2x3x,  = –6e3x
dx
du
or dx =
6e3 x
Making this substitution
e3 x e3 x du
 . dx =  u 2 . 6 e 3 x
1  2e  3x 2
1 1
6  u2
=  . du

1 u 1
=  +c
6 1
1
= + c.
6 1  2e3 x 
(c) Integration by parts : Integration can also be done by parts. This method is used when function
involves product of two or more functions. The integration of such functions is based on reversing the
process of differentiation of product form as explained below :
Let y = uv. (both u and v are taken as functions of x)
dy dv du
 = u. +v.
dx dx dx
Integration both sides
dy dv du
 dx =  u. dx   v. dx
uv =  u.dv   v.du
  u.dv = uv –  v.du
So according to this method, if a function involves product of two functions, one is taken as u and
other equal to dv. Usually the function which can be differentiated easily is taken as u and which can not
be integrated easily is taken as dv. In addition to this, the choice should simplify  v.du .
Example. Evaluate the following :

x .e  lnx . dx  x . lnx dx
3 x2 n
(i) dx (ii) (iii)

xe x 1  x lnx x
(iv)   x  1 2
. dx (v)  x
e . dx

du du
Solution. (i) Let u = x2  = 2x or dx =
dx 2x
Making this substitution :
du
x e  x . u . e . 2x
3 x2 u
dx =

1
=
2  u . eu . dx

Now integrating by parts :


u = u, du = 1
dv = eu, v = eu
1 1
 u . eu . du = ueu   eu . du 
2 2 
1
= ueu  eu  + c
2
1  2 x2
x e  ex  + c
2
=
2  
1 x2 2
= e  x  1 + c.
2 
(ii)  lnx dx = 1. lnx dx

Using integration by parts:


1
Let u = lnx, du =
x
dv = 1 v = x
1
1. lnx dx = x . lnx –  x . . dx
x
= x . lnx – x + c
= x [lnx – 1] + I.

x
n
(iii) For integration by parts . lnx . dx
1
Let u = lnx, du =
x
x n 1
dv = xn, v=
n 1
x n 1 x n 1 1
  x . lnx dx = lnx . –  n  1 . x . dx
n

n 1
x n 1 1
= lnx . –
n 1 n 1  x n . dx

n 1
x n 1 x
= lnx . – 2 + c
n  1  n  1

x n 1  1 
 lnx 
n  1 
= + c.
 n  1 
xe x  x  1 e x  e x
(iv)   x  1 2
. dx =   x  12 dx

ex ex
=   x  1  x  12 dx
dx 

ex
Integrating  x  1 dx by parts,
1 1
u = , du =
x 1  x  1
2

dv = ex v = ex

ex  1 
ex
  x  1 –    
x
dx = e .
  x  1 
2
x 1
ex ex
=  +c
x  1  x  12
Putting this in (i)
xe x ex ex ex
  x  1 2
dx =
x 1

 x  1   x  1
2
 2 + c

ex
= + c.
x 1
1  x lnx  e x 1
  xe dx   lnx . e x dx
x
(v) dx =
x
1
 xe
x
Integrating dx by parts

1
Let dv = , v = lnx
x
u = ex du = ex
1
 x e . dx
x
= ex . lnx –  lnx . e x + c

Putting this result in the integral


1 x lnx x
 e dx = ex . lnx –  lnx x . e x dx   lnx . e x dx + c
x
= ex lnx + c.
(d) Integration by partial fractions : In case a function appears as a quotient of two polynomials,
the integration can be done by partial fractions. Such functions are known as rational functions as f(x) =
g  x
. These may be proper rational functions or improper rational functions. If the power of numerator
h x
is less than the power of denominator, the function is known as proper rational function. But if the power
of numerator is equal to or less than the power of denominator, the function is known as improper rational
functions. Such functions can be changed to proper rational function by long division. For example, f(x) =
2 x 2  3x  4 3
is an improper rational function which is equal to (2x + 1) + is a rational form.
x 1 x 1
The integration of such functions can be obtained by resolving the function into partial fractions
except when the derivative of denominator is equal to or multiple of numerator (where log form can be
used). After resolving the rational function into partial functions, integration of each partial fraction can
be obtained by simple integration either algebraic functions or logarithmic functions as the case may be.
The standard forms of partial fraction are as follows:
Factors of the denominator The partial fraction corresponding to factor
1. Linear factor
A
(ax + b)
ax  b
A A
2. (i) Repeated linear factor (ax + b)2 
 ax  b  ax  b
2
A B N
(ii) Repeated linear factor (ax + b)n   .......
 ax  b   ax  b 
n 1
n
ax  b
Ax  B
3. Quadratic factor (ax2 + bx + c) which
ax  bx  c
2

cannot be factorised with linear factors.


Example. Evaluate the following:
x 1 x 1 x3
(i) . dx (ii) . dx (iii) . dx
x  4x  5
2
x  x2  2x
3
x 4  3x 2  2
dx x5
(iv) (v) . dx
x  x3  x  1 x  2
2

x 1
Solution. (i) Converting into partial fractions.
x  4x  5
2

x 1 x 1 A B
=  
x  4x  5
2
 x  5 x  1 x  5 x  1
x  1  A  x  1  B  x  5 
 x  5 x  1
Putting x = 1, 2 = 6B
B = d.
Putting x = –5 –4 = –64
A = d.
x 1 2 1
So x 2
 4x  5
dx =  3 x  5 . dx   3 x  1 . dx
2 1
= ln  x  5  ln  x  1 + c
3 3
(integrating using log)
x 1
(ii) Converting into partial fractions
x3  x 2  2 x
x 1 x 1
=
x  x  2x
32
x  x  1 x  2 
A B C
=  
x x 1 x  2
x  1  A  x  1 x  2   B  x  x  2   C  x  x  1
=
x  x  1 x  2 
For solving A, B and D
Set x = 0, –1 = A (1) (–2)
1
A =
2
x = –1, –2 = B (–1) (–3)
1
B =
3
x = 2, 1 = C(2) (3)
1
C =
6
x 1 1 2 1
So x 3
 x2  2 x
=  2x dx    3 x  1 dx   6  x  2 dx
1 2 1
= lnx  ln  x  1  ln  x  2  + c.
2 3 6
du du
(iii) Let u = x2, = 2x, dx = .
dx 2x
Substituting this in the function,
x.u du 1 u
u 2
.
 3u  2 2 x
=  2
2 u  3u  2
. du

u
Resolving into partial fractions
u  3u  2
2

u u A B
=  
u  3u  2
2
 2  2 u  1 u  2 u 1
u  A  u  1  B  u  2 
 u  2  u  1
For solving A and B.
Set u = –1, –1 = B (–1 + 2)
B = –1
u = –2, –2 = A (–2 + 1) or A=2

dx = 1   2 du   1 du 
1 u
 
2 u  3u  2
2
2 u2 u 1 

= 2 ln u  2  ln u  1 + c
ln  u  1
= ln (u + 2) – +c
2

ln  x 2  1
= ln (x2 + 2) – +c
2
(Putting u = x2)
1
(iv) Converting into partial fractions
x  x3
1 1 A B C
=   
xx 3
x 1  x 1  x  x 1  x 1  x
1  A 1  x 1  x   B  x 1  x   C  x 1  x 
x 1  x 1  x 
Simplifying it for A, B and C.
Set x = 0, 1 = A
or A = 1
Set x = 1 1 = C(1) (2)
1
C =
2
if x = –1 1 = B (–1) (2)
1
B = –
2
1 1 1
 x . dx   2 1  x  . dx   2 1  x  dx
1
So  xx 3
. dx =

1 1
= lnx  ln 1  x   ln 1  x  + I
2 2
x5
(vi) Converting into partial fractions
 x  1 x  2 
2

x5 A B C
=  
 x  1 x  2  x  1  x  2 x2
2 2

x  5  A  x  2   B  x  1  C  x  1 x  2 
2

 x  1 x  2
2

For solving A, B and C.


Simplify x + 5 = A(x + 2)2 + B(x + 1) + C(x + 1) (x + 2)
or x + 5 = x2(A + C) + x (4A + B + 3C) + (4A + B + 2C)
This shows
A+C = 0 ... (i)
4A + B + 3C = 1 ... (ii)
4A + B = 2C = 5 ... (iii)
(Comparing the coefficients on the left hand side and right hand side)
Solving simultaneously
C = –4, A = 4, B = –3.
x5 4 3 4
  x  1 x  2 2
dx  x  1 dx    x  2 2
dx  
x2
dx

3
= 4 ln (x + 1) + – 4 ln (x + 2) + c
x2

= 4 ln  x  1   3 + c.
 x  2 x2
Applications of Indefinite Integral in Business and Economics
Integration is broadly classified as definite and indefinite integration. Both these have their own
applications. The most important application of integration, i.e., measurement of area under a curve, is
based on definite integration, but indefinite integration has its own applications. These are in the form of
obtaining original function from its derivative function. For example, finding total cost function from the
marginal cost function. Some of these are explained below:
(a) Derivation of total cost function and average cost function from the marginal cost function:
As has been described earlier, integration is reverse of differentitaion. So if marginal cost function, MC(x)
is integrated with respect to x, it will give total cost function, TC(x) [Since MC(x) is the derivative of
TC(x)]. Therefore,

 MC  x  dx = TC (x) + C

where TC(x) is denoting the integral of MC(x) and C is arbitrary constant which was eliminated
from TC(x) in the process of differentiation. TC (x) is same as total variable cost function.
The value of C can be obtained if the total cost of some units of x is given. Put the value of x in TC
(x) = TC (x) + C = given cost and solve for C. So TC (x) = TC (x) + C (value obtained). This C is the
fiexed cost part of total cost and is same as total cost at zero level of output. So it can be obtained by
setting x = 0 in such case.
If the total cost function is obtained, average cost function can be obtained just be dividing the TC
(x) by x (as averate cost is the total cost divided by number of units produced).
TC ( x )
Therefore AC(x) = .
x
Example. The marginal cost function of manufacturing x pairs of shoes is MC (x) = 6 + 10x – 6x2.
The total cost of producing one pair of shoes is Rs. 12.00. Determine the total and average cost function.
Solution. Given MC(x) = 6 + 10x – 6x2
 TC(x) = f MC(x) . dx
= f (6 + 10x – 6x2) dx
= 6x + 5x2 – 2x3 + C.
Given TC(1) = 6(1) + 5(1) – 2(1) + C = 12
 C = 3
Putting C = 3 in TC(x)
TC(x) = 6x + 5x2 – 2x3 + 3
TC ( x) 6 x  5 x 2  2 x 3  3
and AC(x) = 
x x
3
= 6 + 5x – 2x2 + .
x
Example. The marginal cost of producing an item is given by MC(x) = x where x
x  400
2

represents the unit produced. Determine its total cost function and average cost function if cost at zero
level of output is Rs. 900.
Solution. MC(x) = x
x  400
2

 x
TC(x) =  x  400
2
. dx
Using substitution method, let u = x2 + 400
du du
= 2x or dx = .
dx 2x
u du
Making this substitution TC(x) =  .
u 2x
1 1
2 u
= .du

1  u1/ 2 
=   +C
2  1/ 2 
= u +C

= x 2  400 + C
Given total cost at zero level is Rs. 900.
 TC(0) = 0  400 + C = 900
C = 880
Putting C = Rs. 880 in TC(x),

TC(x) = x 2  400 + 880

x2  400  800
AC(x) =
x
Example. The cost of producing x th unit is given by MC(x) = 20e0.5x. Determine the corresponding
total cost function and average cost function if total cost at zero level is Rs. 80.
Solution. Given MC(x) = 20 e0.5x
  20e
0.5 x
TC(x) = . dx

20e0.5 x
= +C
0.5
= 40 e0.5x + C.
Using TC (0) = Rs. 80
TC (0) = 40 e0.5(0) + C = 80
C = Rs. 40
Putting C = Rs. 40 TC(x) = 40 e0.5x + 40
40e0.5 x  40
AC(x) = .
x
Example. The marginal cost function of a firm is MC(x) = lnx if x  1. Find its total cost function
and average cost function if total cost function if total cost of one unit is Rs. 10.
Solution. Given MC(x) = lnx
 TC(x) = flnx dx
= f 1.lnx dx
= x . lnx – x + C [integrating by parts]
= x [lnx – 1] + C.
Using TC(1) = Rs. 10
TC(1) = 1[ln 1 – 1] + C = 10
C = 11 [ln 1 = 0]
Putting C = 11
TC(x) = x (lnx – 1) + 11
x  lnx  1  1
AC(x) = .
x
(b) Derivation of total revenue function and average revenue function from marginal revenue
function: This indefinite integration can also be applied for obtaining total revenue function, TR(x) from
marginal revenue function, MR(x), which in turn can be used for deriving average revenuie function,
AR(x) also known as demand function. It is as follow.
TR(x) =  MR  x  dx = TR(x) + C
where TR(x) is simply the integral of MR(x) and C is the abritrary constant. The value of C is
evaluated assuming total revenue at zero level is zero. So setting TR(0) = 0, value of C is obtained.
Corresponding to the TR(x),
TR  x 
AR(x) =
x
Example. The additional revenue of xth unit is given by MR(x) = x2 – 12x + 27. Determine its total
revenue function and average revenue function.
Solution. Given MR(x) = x2 – 12x + 27
 x  12x  27  dx
2
TR(x) =

x3
= – 6x2 + 27x + C
3
Assuming TR(0) = 0
TR(C) = 0 + C = 0
C = 0.
x3
So, TR(x) = – 6x2 + 27I
3
x3
 6 x 2  27 x
AR(x) = 3
x
x2
or P = – 6x + 27.
3
Example. Find the total revenue function TR(x) and demand function of the marginal revenue
function.
ab
MR(x) = – C.
 x  b
2
ab
Solugion. Given MR(x) = –C
 x  b
2

 ab 
 TR(x) =    x  b   dx
 2
 C
 
ab
= – – Cx + C1 [C1 is arbitrary constant]
xb
putting TR(0) = 0
ab
TR(0) = – – C(0) + C1 = 0
0b
C1 = a
ab ab
 TR(x) = – – CX + a = – Cx (Simplified form)
 x  b xb
ab
 Cx
and AR(x) = + x  b
x
a
or P = + – C.
xb
Example. A firm’s marginal revenue function
a ax
MR(x) =  +k
x  b  x  b 2
a
Show that is demand function is P = + k.
xb
a ax
Solution. Given MR(x) =  +I
x  b  x  b 2

 a ax 
 TR(x) =   x  b  x  b 2  dx
   k
 
a ax
=  x  b dx    x  b  2
dx   kdx

ab
= a ln (x + b) – – a lin (x + b) + kx + C
xb
ab
= – + kx + C
xb
Assuming when x = 0, TR(0) = 0,
 ab
TR(0) = + k(0) + C = 0
0b
C = a
Putting C = a
 ab
TR(x) = + kx + a
xb
ax  ab
= + kc (Simplifying +
xb xb
a)
ax
 kx
 AR(x) = x  b
x
a
or P = +k
xb
ax
[Note. is obtained by partial fraction method]
 x  b
2

100
Example. The additional revenue of xth unit of a commodity is given by MR(x) = . Determine
x2
the corresponding total revenue function and demand function.
100
Solution. Given MR(x) =
x2
100
 TR(x) =  x  2 . dx
= 100 ln (x + 2) + C
Assuming TR(0) = 0
TR(0) = 100 [ln 2] + C = 0
C = –100 ln 2
So, TR(x) = 100 ln (x + 2) – 100 ln 2
  x  2 
= 100 ln  
  2 
 x 
= 100 ln   1 
 2 
 x 
100 ln   1 
Corresponding AR(x) =  2 
x
 x 
100 ln   1 
P =  2 
x
(c) Derivation of demand function: Another application of indefinite integration is for obtaining
the demand function if the price elasticity of demand is given. Although it involves differential equation,
but if price elasticity is constant at all prices, the demand function can be obtained easily b indefinite
integration. If it is given as –k at all prices, the demand function corresponding to this will be as follow.
dx P
Price elasticity of demand EP = . where d is the derivative of demand function in relation to
dP x
P . P and x denotes the price and quantity respectively.
Given Ep = –k
dx P
So, . = –k
dP x
dx dP
or, = –k .
x P
dx dP
integrating both sides  x
= –k 
P
[as –k is constant]

or lns + C1 = –k lnp + C2 [C1 and C2 are arbitrary constants]


or lnx = –k lnP + C3
[Combing C1 and C2 and putting equal to C3]
or lnx = –k lnP + lnC [Putting C3 = lnC as C3 > 0]
lnx = lnC – k lnP
C
or lnx = ln
Pk
C
or x =
Pk
or x = CP–k (The demand function having constant elasticity –I at all prices).

DEFINITE INTEGRATION
Integration is broadly classified as indefinite and definite integration. Indefinite integration involves
antiderivative of a function. It is basically used for obtaining original function from its derivative
function. It is known as indefinite because the constant of integration, c is arbitrary.
Definite integral is one step ahead of indefinite integral. It is mainly used for the measurement of
b

area under a curve. If a function f(x) is continuous in the close interval [a, b], then the symbol  f  x  dx
a
denotes the value of definite integral of F over the interval [a, b].
Historically the basic concept of the definite integration was used in about 200 B.C. But Newton and
Leibnitz first evolved out a fundamental theorem of the calculas which states how to calculate the area
under a curve with the help of definite integration.
According to this theorem, if f(x) is a continuous function in the close interval [a, b], then the area
under the curve between a and b is given by evaluating the integral of f(x) with respect to x with the limits
a and b. (These are known as lower bound and upper bound respectively). It is shown in figure 1. Its
value can be obtained in this way.
Fig. 1
b

 f  x  dx
b
= F ( x) |
a a

= F(b) – F(a)
This gives a numeric value denoting area.
[Note : Value of the arbitrary constant c is zero in this case].
Example. Evaluate the following:
3 5 8

  3x  5x  1 dx (ii)  e dx (iii)  lnx dx


2 x
(i)
1 1 2

 3x 2 5 x 2 3
3

1       x |
2
Solution. (i) 3 x 5 x 1 dx = 
 3 2  1

=  x3  5 x 2  x  |
3

 2  1

=  33  5 . 32  3    13  5 .  12  1
 2   2 
83
=
2
5 5

 e dx  e | = e5 – e1
x x
(ii)
1
1

= 145.69 app.
(Value of e5 and e1 are given by exponential table)
8
8
(iii)  lnx dx =  x lnx  x  |
2 2
= (8ln 8 – 8) – (2ln 2 – 2)
= 8ln 8 – 2ln 2 – 6
= 9.249
[value of ln 8 and ln 2 are given by natural log table]
Example. Find the area under the curve y = 4x – x2 in between x = 1 and x = 3 on x-axis.
3

  4 x  x  dx
2
Solution. Area =
1

x3 3
= 2 x2  |
31
  3   1 
2 2

=  2  3     2 1 
2 2

 3   3 
22
= square units.
3
Example. Calculate, by integration, the area of the portion of the parabola y2 = 4ax which is cut off
by the line y = 3x.
Solution. Both y2 = 4ax and y = 3x, cut each other at the points satisfying both
equations.
 y2 = 4ax ... (i)
y = 3x
or y2 = 9x2
if left hand side is equal, putting right hand expressions equal and solving for x ... (ii)
4ax = 9x2
or 9x2 – 4ax = 0
or x (9x – 4a) = 0
4a
or x = 0 or x=
9
Corresponding values of y are y = 0 and
4a
y = .
3
4a / 3

 area of the portion of porabola = 


0
y . dx

y
where dx = dy
2a
(Differentiating y2 = 4ax with respect to x)
4a / 3 2
y
 = 
0
2a
. dy

4a / 3
1
=
2a 0
y 2 . dy
1  y3 
4a / 3
=  | 
2a  3 0 

32 a 2
= square unit.
81

EXERCISE
1. Evaluate the following definite integral
x 1
4 3

1 x dx  x x  4  dx
2 2
(i) (ii)
1

2 1 2

  4 x  1 2 dx  x  x  1 dx
2
(iii) (iv)
0 3

2a

(v)   a  x  dx
a

2. Find the area bounded by the curve x2y = x2 – 4 on x-axis when x = 2 and x = 4
3. Find the area bounded by the curve y = x3 – 4x on the x-axis.
4. Find the area bounded by the curve y = x2 – 5x + 6 on the x-axis.
 x 2 if x  2
5. Find the area under the curve f(x) = 
  x  6 if x  2
In between x = 0 and 3.

ANSWERS
20 76 13 13 7 2
1. (i) , (ii) , (iii) , (iv)  , (v) a
3 x 3 12 2
2. 1 sq. unit 3. 8 sq. unit
1 37
4. sq. unit 5. sq. unit
6 6

Applications in Business and Economics


The most important application of definite integration is the measurement of area under a curve.
This very application is widely used in different situations in business and economics. Some of these are
as follows:
(a) Measurement of consumer’s surplus : In economics, consumer’s surplus is defined as the
difference between the price consumer is willing to pay and the price which he actually pays. If the price
consumer is willing to pay, is more than the price he actually pays, the difference is consumer’s surplus.
Otherwise there will be no consumer’s surplus. The price which the consumer is willing to pay is given
by demand function and the price which he pays is given by market demand and supply.
Diagrammatically it is shown in figure 2. Let P = f(x) is a price function stating the price consumer
is willing to pay for different values of x and P is the price fixed by market demand and supply at which
consumer is purchasing x units. Then total price consumer is willing to pay, is given by the integral of f(x)
 
with the limits 0, x whereas price actually paid is p x . So the difference, i.e., area of is the
measurement of consumer’s surplus.
x

Therefore, Consumer’s surplus =  f  x  dx  p x


0

Figure 2

An alternative method is to measure consumer’s surplus with the help of integrating demand
function in relation to price [i.e., x = f (p) which is inverse of price function]. It involves obtaining the
demand function corresponding to price function and integrating it with respect to P with the limits
 P, B  where B is the price at which quantity demanded is zero. It can be obtained by setting x = 0 in the
demand function.
B

Therefore consumer’s surplus =  f   p  dp


p

Both gives the same result.


Example. Find the consumer’s surplus if the consumer’s price function is p = 45 – 2x – x2 and he
purchase 5 units.
Solution. If consumer purchases 5 units, it means market price,
P = 45 – 2(5) – (5)2
= Rs. 10
5

  45  2 x  x  dx  10  5
2
 Consumer’s surplus =
0

 x3  5
=  45 x  x 2
  | – 50
 3 0
475
= – 50
3
375
= Rs. or Rs. 108.33 app.
3
Alternatively, inhere function of the given price function, i.e., demand function will be
45 – 2x – x2 – P = 0
or x2 + 2x – 45 + P = 0
2  4  4  45  P 
 x =
2
= 46  P – 1
[negative value not possible, so ignored]
45

 Consumer’s surplus = 
10
46  P  1

[P = Rs. 45 when x = 0]
2 45 45
=   46  P  |  P |
3/ 2

3 10 10

=   2  144  – 35
 3 
325
= Rs. on Rs. 108.33 app.
3
Example. The demand equation of a commodity is P – ln (25 – x) = 0. Find the consumer’s surplus
if he purchases 15 units.
Solution. Given P – ln (25 – x) = 0
P = ln (25 – x)
When x = 15 units, p = ln 10
15

 Consumer’s surplus =  ln  25  x  dx  ln 1015


0
15
=   25  x  ln  25  x   1 |   ln 10 15 (integrating)
0

= –10 [ln (10) – 1] + (25)[ln 25 – 1] – (ln 10) 15


= 25 [ln 25 – ln 10] – 15 (Simplification)
= 25 ln 25  – 15
 10 
= 25 ln (2.5) – 15
= 25 (.9163) – 15
= Rs. 7.91 app.
[value of ln 2.5 is given by natural log table]
Example. A monopolist’sk demand and total cost function are P = 25 – x and TC(x) = 10 + 9x
respectively. Determine the consumer’s surplus (assuming monopolist maximises profit).
Solution. In case of monopoly, the firm supplies units where it makes maximum profit. It is given
by setting marginal revenue equal to marginal cost. It is shown below.
Given P = 25 – x TC(x) = 10 + 9x
So TR(x) = P . x = 25x – x2 MC(x) = 9
dTR ( x )
MR(x) = = 25 – 2x
dx
Setting MR(x) = MC(x) and solving for x
25 – 2x = 9
x = 8 unit.
Profit is maximum at x = 8 unit as marginal cost is constant.
 P = 25 – 8 = Rs. 17
8

Consumer’s surplus =   25  x  dx  17 8


0

 x2  8
=  25 x   | – 136
 2 0
= 168 – 136
= Rs. 32.
(b) Measurement of Producer’s Surplus. As in the case of consumer’s surplus, producer’s surplus
is the difference between the price received by the producer and the price at which producer is willing to
supply. For example, if producer sells product at Rs. 10.00 per unit whereas he is willing to sell at Rs.
8.00, per unit, the producer’s surplus is Rs. 2.00. The price producer is willing to accept is given by his
supply curve; and the price he actually gets is determined by demand and supply. Diagrammatically it is
shown by figure 3.

Figure 3
x

Therefore Producer’s Surplus = p x   g  x  dx


0

where p and x are the market price and quantity supplied. g(x) is the supply function of the
x

producer. p x denotes the price producer gets by supplying x units and  g  x  dx


0
gives the price

producer is willing to accept. So difference represents producers surplus.


An alternative method is to take the inverse of given supply function P = g(x) which will be x =
g(P) denoting x as a function of P. This integral of g(P) with the limit (P0, ) gives the producer’s surplus.
P0 is denoting the price at which producer is willing to supply any unit it can be obtained by setting x = 0
in the either function and solving for P.
P
Therefore Producer’s Surplus =  g   P  dp
P0

Example. The supply equation of a commodity is 10P – 2 x + 200 = 0 and market price is Rs.
8.00. Find the producer’s surplus.
Solution. Given surplus equation
10P  2 x  300 = 0
or 10P  2 x  300 = 0
1
P = x  300
5
1
when P = 8, quantity supplied will be 8 = x  300
5
or, 1600 = x + 300
 x = 1300 units.
1300
1
Producer’s surplus = 8 (1300) – 
0
5
x  300 dx

1 3/ 2 2
1300
= 10,400 –  x  300  . |
5 3 0
2  1300

= 10,400 –   
3/ 2
 x 300 | 
15  0 

= 20,400 – 7,840 (simplification)


= Rs. 2560.
Example. The supply function of a firm is x = 1 (2P – 3). Find the producer’s surplus if P = Rs. 5.
Solution. Given supply function, x = ln (2p – 3),
So when p = Rs. 5, x = ln (10 – 3)
= ln 7 units
Also when x = 0, 0 = ln (2p – 3)
or, p = Rs. 2 [as ln 1 = 0]
5

Producer’s surplus =  ln  2 p  3 dp
2
5
=  2 p  3[ln  2 p  3  1]| (integrating by parts)
0

= 7 [ln 7 – 1] – 1 [ln 1 – 1]
= 7 ln 7 – 7 – ln 1 + 1
= 7 ln 7 – 6 [ln 1 = 0]
= 7 (1.9459) – 6
= Rs. 7.62 app.
Example. Under pure competition, the demand and supply functions of a commodity are P =
8
 2 and P = 1/2 (x + 3) respectively. Determine the consumer’s and producer’s surplus at
x 1
equilibrium price.
8
Solution. p = 2 (Demand)
x 1
1
p = (x + 3) (Supply)
2
So equilibrium quantity is given by
8 1
2 = (x + 3)
x 1 2
or, 16 – 2 (x + 1)2 = (x + 3) (x + 1)
x2 + 8x – 9 = 0
(x + 9) (x – 1) = 0
So x = –9 or 1
x = 1 [as x = –9 is not possible]
Putting x = 1, in demand or supply function, equilibrium price, P = Rs. 2.
 8 
1

So consumer’s surplus =   x  1  2  dx  2 1


0
1
= [8ln  x  1  2 x]|  2
0

= [(8ln 2 – 2) – (8ln 1 – 0)] – 2


= 8ln 2 – 4 [as ln 1 = 0]
= 8(.6932) – 4
= Rs. 1.55 app.
1
1
Producer’s surplus = 2(1) –  2  x  3 dx
0

1  x 2  1
= 2–   3 x  | 
2  2  0

= 2 – 1  7 
2 2
7
= 2–
4
1
= Rs. or 0.25 app.
4
(c) determination of maximized Profit : Definite integration can also be applied for determining
the maximized profit if the marginal revenue and marginal cost functions are given. Marginal revenue
gives the rate of change in total revenue and marginal cost gives the rate of change in total cost. The firm
makes maximum profit when the rate of change in both are equal and afterwards rate of change in total
cost is more than the rate of change in total revenue (i.e., marginal cost is increasing). Diagrammatically it
is shown in figure 4.

Figure 4

The area shaded represents the maximized profit. It is the area bounded by marginal revenue
function MP(x) and marginal cost function MC(x) with the limit XI . X is denoting the profit
maximizing output obtained by setting MR(x) = MC(x) and solving X. At this MC(x) must be increasing
for maximum profit.
X

So maximized profit =   MR  x   MC  x  dx


0

This actually gives the additional profit only. So loss at zero level of output must be subtracted from
this for obtaining actual profit. It is same as fixed cost in the short-run
X

So maximized profit =   MR  x   MC  x  dx  C


0

where C is the fixed cost or loss at zero level of output.


If it happen that marginal revenue is equal to marginal cost at two level of output as show in
figure 5.
Figure 5

Till OX, marginal cost lies above marginal revenue representing loss. From X to X1, marginal
revenue lies above marginal cost represent profit. So maxmimized profit is the difference between profit
area and loss area.
X1 X

Therefore Maximized profit =   MR  x   MC  x  dx    MC  x   MR  x  dx


X 0

X1

=   MR  x   MC  x  dx
X

Example. A firm’s marginal revenue function, MR(x) = 1000 – 4x and marginal cost function,
MC(x)3x2 – 118x + 1315. Determine the maximized profit using definite integration assuming fixed cost
is Rs. 595.
Solution. Given MR(x) = 1000 – 4x, MC(x) = 3x2 – 118x + 1315.
For profit maximizing output, put MR(x) = MC(x) and solve for x.
 1000 – 4x = 3x2 – 118x + 1315
or, 3x2 – 114x + 315 = 0
114  114   4  3  315 
2

x =
6
= 3 or 35
But it is making maximum profit when X = 35 only as marginal cost is increasing at this level only.
(It can be verified by the derivative of MC(x) at x = 35. It is positive.
35

 1000  4 x   3x  118 x  1315   dx  595


2
So Maximized profit =
0
35

  3x  114 x  315 dx  595


2
=
0

x  57 x 2  315 x  |  595


35
3
=
0

= Rs. 1000.
(d) Determining the present value of an annuity: Annuity is the fix amount which flows after
regular intervals. It may be inflowing as regular monthly income or it may be outflowing as payment of
insurance premium or loan instalment. If it is inflowing, the present value involves discounting of each
annuity at a given rate. The sum of the present value of each annuity gives the total present value of the
annuity. It may be discounted with time intervals as annually, half yearly, quarterly, monthly or
continuously. If the annuity is discounted annually the present value can be obtained easily using ready
made tables. If it involves, half-lyearly ormonthly, the same tables can be used with effective rate and
time corresponding to discounting time (as half yearly amounts to yearly if rate is half and time doubled).
But if annuity is discounted continuously, the present value can be obtained using definite integratin.
It is given by
t

 Ae
 rt
P = dt
0

where P is denoteing total present value


A is the amount of annuity
e–rt is the discounting factor
r is rate of discount in relation to 1
t is the time period in years.
Ae  rt t
= | (integrating by substitution method)
r 0
A
= 1  e rt 
 (Simplified)
r
[Note. Values at e–rt can be obtained from exponential table giving values at e–rt corresponding to
rt].
Example. Find the present value of Rs. 5000 inflowing annually of 5 years if the rate of discount is
8 per cent per annum being discounted continuously.
5

 5000e
08t
Solution. Total present value = dt
0

5000e 08t 5
= |
0.8 0
5000
= 1  e4 
0.8
= 62,500 [1 – 67032]
= Rs. 20,605 app.
(e) Determining the future value of an annuity: The definite integration can also be used for
determining the future value of an annuity. It involves compounding at a given rate. It can also be with
different time intervals as yearly, half yearly, quarterly, monthly or continuously.
If annuity is compounded annually, half yearly, monthly, the future value can be obtained by using
ready made tables. But if compounding is continuously, the future value can be obtained using definite
integration. It is as follows:
t

 Pe
rt
Future value, A = dt
0

Where P is the annuity amount.


ert is the compounding factor.
r is the rate of compounding in relation to one.
t is the time period in years.
Pe rt t
A = |
r 0
P rt
=  e  1
r
[Note. The value of ert is given by exponential table corresponding to rt]
Example. Mr. X opens a recurring deposit account of Rs. 1000 per annum for 10 years. If the rate of
interest is 10 per cent per annum compounded continuously. What will be the total balance after 10 years.
10

 1000e
0.10t
Solution. Total balance, A = dt
0

1000e0.10t 10
= |
0.10 0
= 10000 [e1 – 1]
= 10000 [2.7183 – 1]
= Rs. 17183.
(f) Other Applications: In addition to various applications described above, there are other
applications in business and economics based on definite integration. In general, whenever, rate of change
is given for a continuous function, the accumulated value for certain limits can be obtained using this. For
instance, if marginal revenue function denoting rate of change in total revenue is given and additional
revenue is required if sale is increased from 10 units to 20 units; it can be obtained directly using definite
20

integration as to additional revenue =  MR  x  dx . Similarly if the marginal cost function is given and
10
additional cost of increasing output from one level to another level is required, it can be obtained by
integrating the given marginal cost function with the given limits. If the rate of cost of repairing is given,
total repairing cost for certain period can be obtained by this integration. All these situations are
exemplified below:
Example. A firms cost of producing xth unit is given by MC(x) = 10 + 2x. Find the additional cost if
output is increased from 10 to 20 unit.
20

Solution. Additional cost =  10  2 x  dx


10
20
= 10x + x 2 |
10

= (200 + 400) – (100 + 100).


= Rs. 400.
Example. The daily rate of sales for a new product is given by f(t) = 100 t e–0.05t where t is the
number of days, the product has been in market. Determine the total sales of first 10 days.
20

 100te
0.05t
Solution. Total sales = dt
10

20

= 100  t e
0.05t
dt
10

  t e0.05t 20 20 e0.05t  
= 100   |  [Integrating by parts]
  0.05 0 0 0.05  

= 100  20e1 1 e1  1  (Integrating second term)


0.05  0.05 
= 2000 [20 – 40 e–1]
= 2000 [20 – 40 (.3679)]
= 2000 [5.28]
= Rs. 10,560.
Example. A publishing company is contemplating the purchase of a new Printing machine at a cost
of Rs. 28,000. The company believes that after a short period of adjustment, savings in printing will offset
the cost. The rate of cost savings over a period of 10 years is estimated to be C(x) = 500 + 1000 x where x
represents years and C(x) is the savings (in Rs.) per year at any given time. Will the machine pay for itself
in 5 years? If not, in how many years of operation would the machine pay for itself.
5

Solution. Total savings of 5 yrs =   500  100 x  dx


0
5
= 500 x  50 x 2 |
0

= Rs. 15,000.
So it will not pay for itself in 5 yrs. Let it pay for Rs. 28,000 in t years, then
t

Rs. 28,000 =   500  100 x  dx


0

500 x  50 x  |
t
2
=
0

28,000 = 500 t + 500 t2


or t2 + t – 56 = 0
 t = 7 or –8 years
(Solving quadratic equation for t)
t = 7 years (as –8 years is not possible)
Example. The purchase price of a car is Rs. 75,000. The rate of cost for repairs is given by C =
600 (1 – e–0.5t) where t represents years of use since purchases. Find the cumulative repair cost at the end
of 5 yrs. Also find the equation to give the time in years at which the cumulative repair cost equals the
original cost.
5

 600 1  e  dt
0.5t
Solution. Total cumulative repair cost =
0

 e 0.5t  5 
= 600  t   |
 0.5  0 
 e 25   e0  
= 600  5   
 0.5   0.5  
= 600 [3 + 2e–2.5]
= 600 [3 + 2 [.0821]]
= Rs. 1898.50 app.
Let total repair cost equal to initial cost in t years, then equation will be:
t

 600 1  e  dt
0.5t
= 75000.
0

IMPROPER INTEGRAL
Definite integral can be of two types, i.e., proper definite and improper definite integrals. Proper
integral is one where the function is defined in the close finite interval as the definite integral of f(x) being
defined in the closed finite interval of [a, b]. So symbolically
b

  x  dx
a

is an example of proper integral. So if the function is defined in the interval and interval is finite, the
integral is known as proper integral.
But i. the integral is not proper, it is known as improper integral. It may be either because the
interval is not finite or because function is not defined in the interval. Both these are discussed below.
(a) Improper integral due to infinite limits of integration. It may be in any of these three forms.
b

(a) lower bound is tending towards infinity as  f  x  dx as shown in figure 6.




(b) upper bound is tending towards infinity as  f  x  dx as shown in figure 7.


(c) both upper and lower bounds are tending towards infinity as  f  x  dx as shown in fig. 8.


Fig. 6
Fig. 7

Fig. 8

These improper integrals can be evaluated in the same way as proper integrals with some difference.
The important steps are as follows.
1. Evaluate the indefinite integral of the integrand by suitable method, i.e., if
y = f(x) and ฀f(x) = F(x).
2. Evaluate it assuming finite limits as [a, b], i.e.,
b
F  x  | = F(b) – F(a).
a

3. Take the limit of definite integral when any of the bounds or both the bounds are tending towards
infinity which will be as follows:
(a) if lower bound is tending towards infinity
lim  F  b   F  a  
a  

(b) if upper bound is tending towards infinity


lim  F  b   F  a  
b

(c) if both bounds are tending towards infinity


lim  F  b   F  a  
b  
b

So improper integration involves the limit of proper integral if any or both bounds are tending
towards infinity. It means improper integral is the limit of proper. Such an integral is known as
convergent if the limit exists and divergent if the limit does not exist.
In case of convergent area under the curve is finite but it is infinite if integral is divergent. This
improper integral has a number of applications in commerce and economics. Area under the normal curve
involves evaluation of improper integral. Similarly discounting in perpetuity involves improper integrals.
These are discussed in the latter part of chapter.
Example. Evaluate the following improper integrals and determine whether integral is convergent
or divergent.
0  
1
 e dx  (iii)  x e dx
3x 3 x
2
(i) (ii) dx
 1 x 0

 
1
 e dx  x dx
x
(iv) (v)
 1

Solution. (i) Let lower bound is a,


 0

e 
3x
dx = lim
a  
 a

 e3 x 0 
= lim  |
a  
 3 a
 1 e3 a 
= lim  
a 3

 3 
1
= 1  e  
2
1
as e  0
=
3
So limit is existing and integral is convergent with finite area.
(ii) Let upper bound is b
 b
1 1

1 x
dx = lim 
b  
1 x

= lim  2. x | 
b

b 1

= lim  2. x  2 
b 
= 
So limit is not existing and integral is divergent with no finite area.
du du
(iii) Let u = x2, = 2x or dx =
dx 2x
Substituting this in the integrand, it becomes
 
u . e . eu du 1
0 2 x = 2 0 u . e du
u


1
Assuming upper bound is b =
20 u . e  u du

1 
b

 b    u . e . e du 
u
= lim
2 0 
1  
b u
b e
= blim  ueu |   du   (integrating by parts)
2     0
0
1  

lim  ue b  0    d  b  1 
1
=
2 b   

lim 1  beb  eb  


1
=
2 b   
1
= [as both be–b & e–b are zero when b  ]
2
Since limit is existing, integral is convergent and area is finite.
(iv) Let lower bound is a and upper bound b,
 b

 e dx = lim  e x dx
x
a  
 b a

 x b
= lim e | 
a    a
b

= e – e–
= .
Since limit is not existing, integral is divergent and area is infinite.
 
1
(vi) Let upper bound is b 1 x dx = blim

1
 b
= lim  ln x | 
b
1
1

= lim ln b  ln 1
b

= .
So limit is not existing, integral is divergent and area is infinite.
(b) Improper integral due to an infinite integrand. If the second condition that integrand is not
defined in the interval, the integral is improper due to an infinite integrand. Limits of integrand may be
finite or infinite. In this case also, there are three different possibilities which together with their
evaluation methods are as follows:
(a) Infinite discontinuity at lower bound a, let it is equal to x, then, (as shown in figure 9)
b
lim 
xa
x

 b

lim  F  x  |  = is the integral.
xa  x
Fig. 9

Fig. 10
Fig. 11

(b) Infinite discontinuity at the upper bound b, let it is equal to x, then (as shown in figure 10).
x
lim f  x  dx = lim  F  x  |  is the integral
xb 
x

xb a

a

(c) Infinite discontinuity at a point c in the interval [a, b], let it is equal to x, then integral can be
obtained by summation form (as shown in figure 11).
b x b

 f  x  dx = lim  f  x  dx  lim  f  x  dx
a
xc
a
xc
x

if the limit of both the integrands is existing when x  c the integral is convergent. If the limit of
any of the integrand is not existing (or both are not existing), the integral is divergent.
Example. Evaluate the following definite integrals:
1 1 3
1 1 1
(i)  dx (ii)  dx (iii)  x  3 dx
0
x 0 x 0

1 

x  x lnx dx
3
(iv) dx (v)
1 1

Solution. (i) At lower bound, 0 integrand is infinite, so let it is x, then


1 1
1 1
0 x
0 x dx = xlim x
dx

= lim ln x | 
1

x0 x

= lim [ln 1 – lnx]


x 0
= [ln 1 – ln 0]
= – [as ln 0 is not defined and taken as –]
So limit is not existing and integral is divergent.
1
1
(ii) Given 0 x
dx . At lower bound, the integrand is infinite, so let it is x, then

1 1
1 1

0 x
dx = lim 
x0
x x
dx

= lim  2 x | 
1

x0 x

= lim  2  2 x 

= 2
So limit is existing, integral is convergent.
(iii) At uppe bound, integrand is infinite, so it is x, then
3 x
1 1
0 x  3 dx = lim
x3 x  3
0
dx

= lim ln  x  3 | 
x

x3 0

= lim ln  x    ln  3


x 3

= ln 0  ln  3

Since ln 0 is not defined, it integral is divergent.


(iv) The integrand is discontinuous at 0 in between lower bound and upper bound, so let it is x, then
1 x 1

 x dx = lim  x dx  lim  x dx
3 3 3
x0 x0
1 1 x

 x 2 x   x 2 1
= lim  |   lim  |
x  0 2 x0
 1  2 x

 1 1   1 1  
= lim        
x0
 2 x 2   2 2 x  
= – + 
So neither limit is existing, the integral is divergent.

(v)  x lnx dx is infinite at 0. So let it is x, then


1
 x 

 x lnx dx
1
= lim  x lnx dx  lim  x lnx dx
x0
1
x0
x

Also let upper bound is b


x b
= lim  x lnx dx  lim  x lnx dx
x0 x0
1 b x

 x2  1  x  x2  1 b
= lim  lnx  | 
 0 
lim lnx  | 
x0
2  2  1  bx  
2 
 2 x  

 x2  1 1  1 
= lim  lnx  2   2 ln 1  2  
x0 2
    
 b2  1  x2  1 
 lim  lnb   lnx  2  
x0
b 
2  2  2  
= – + 
Therefore neither limit exist, the integral is divergent.
Applications of improper integrals in business and economics
Such improper integrals are often used in business and economics as area under the normal curve
involving improper integration, Similarly discounting in prepetuity is also based on this. Some examples
in support of this are as follows:
Example. A businessman is expected to earn Rs. 20,000 per annum in perpetuity. If it is assumed
that rate of discount is 10 per annum being discounted continuously. What is the total present value?
t

 Ae
 rt
Solution. Total present value = dt where t  
0

 Putting A = Rs. 20,000, r = 10 per annum.


[e–rt is the discounting factor when discounted continuously]
t

Total present value = lim  20,000 e dt


.10 t
t 
0

  e .10t t  
= 20000  lim  | 
t   .10 0
  

lim  e.10t  1 
20000 
=
.10 t   


= 2,00,000  lim 1  e.10t 
t    
= Rs. 2,00,000
t

 Ae
 rt
[Note: Present value of annuities = dt
0

A
= 1  e rt 
r
as proved in definite integration.]
If it is in perpetuity, t  
 Present value = lim  A 1  e rt  

t  r

A  rt
= e  0 if t   
r
Example. A company has taken a newly constructed commercial complex on lease on which rent is
paid perpetually. If the average rent is paid in a month is Rs. 1000 and rate of discount is 10 per cent
being discounted continuously. Determine the total present value. (Annual rent = Rs. 12,000)
t 
Solution. Total present value = lim
t 

0
 12,000 e .10t dt 

  e .10t  t 
= lim 12000   |
t
  .10  0 

= lim 12000 1  e.10t 


t 
 .10 
= Rs. 1,20,000.
Example. After an advertisement campaign, a product has sales rate S() = 1000 e–0.5t wher  is the
number of months since the close of campaign. Find (i) the total sales for first 3 months all the close of
campaign (ii) The total sales as a result of the campaign.
Solution. (i) Total sales of first 3 months
3

1000 e
0.5t
= dt
0

 e 0.5t 3 
= 1000  |
 0.5 0 
1000
= 1  e1.5 
0.5 
= 2000 (1 – 0.7768)
= Rs. 1553.60 app.

1000 e
0.5t
(ii) Total sales of campaign = dt
0

t 
= lim   1000 e 0.5t dt 
t
0 
1000 
= lim e0.5t  1 
0.5 t    

= 2000  lim 1  e0.5t  


t   
= Rs. 2,000 [as lim e0.5t = C]
t 
Example. The equity share of a company with the face value of Rs. 100 is expected to ykeld 20 per
cent in perpetuity. Determine the value of equity share if rate of discount is 10 per cent per annum.
Solution. Give face value = Rs. 100 Yield = 20 per cent
 Dividend per year = Rs. 20
t

 20 e
0.10 t
So value of equity share = dt where t  
0

t 
= lim   20 e 0.10 t dt 
t
0 
20 A
= = Rs. 200[Using standard form ]
10 r
Example. A company’s dividend on equity share is according to Dt = D0 (egt) where D0 is the
constant, g is the growth rate growing continuously. Determine the total present value of dividend of first
t years if rate of discount is r per cent. Also determine total present value if t  . [Assuming r > g]
Solution. Given D1 = D0 egt. So present value = D0 egt e–rt.

= D0 e(g – r)t
t
e
g  r t
 Total present value = D
0
0 dt

t
 g  r t
= D0  e dt
0

 e g  r  t t
= D0  |
 g  r 0

D0   g  r t
= e  1
g r 
Taking its limit when t  
D0 
lim e   1 
g r t
=

g r  t   

D0
=  1
g r
[because e(g – r)t = 0 when t   as r > g or (g – r) t < 0]
= D0
g r
Example. In a certain industry revenue flows according to function. TR(x) = a + bx where x denotes
the time. Determine the present value of total revenue of the first x years. Also determine total revenue if
x  . (if discounted continuously)
Solution. Given TR(x) = a + bx, so its present value, P = TR(x) . e–rx
x

  a  bx  e
 rx
P = dx
0
= erx   a  bx  b2  |
x
[integrating by parts]
 r x 0

= e rx   a  bx  b2   d 0   a  b2 
 r x   r r 
a b bx
= 1  e rx   2 1  e rx   e rx 
r r r
when x  
a b bx
P = 1  e   2 1  e   e 

r r r
a b ar  b
=  
r r2 r2
LESSON 3

SIMPLE DIFFERENTIATION

Introduction
The term ‘calculus’ means method of calculation. This meaning does not have much relevance in
the practical use of the term in relation to differentiation. Calculus is classified as differential and integral.
Differential calculus is related withk average rate of change.
It is one of the important branch of mathematics. It is becoming more and more important since its
evolution. Although an idea was evolved about it far back as Greek antiquity. But the credit for the
development of the subject goes to the mathematician and philosopher Gothfried W. Leibniz (1646-1716)
and to mathematician and physicist Isac Newton (1642-1727). They made the most significant
contributions to the development of the mathematical system of calculus simultaneously and
independently.
Initially it was applied to various problems of physical sciences only but now it has been commonly
applied in the study of commerce and economics. Various aspect of economics as price elasticity of
demand and supply, marginal function, firm’s equalibrium under different market conditions, laws of
returns etc. involve applications of differential calculus. Similarly it has applications at the macro level as
multiplier and accelerator.
It also has various applications in commerce. But the applications in economics as such can not be
separated rigidly because of close linkage between two disciplines. Although some specific applications
include inventory control, measurement of rate of growth, optimum operating level etc.

Meaning of Differntiation
As mentioned above, differential calculus is related with average rate of change. The method
involved is known as differentiation and the result obtained is known as derivative or differential
coefficient.
Practically derivative can be defined as the ratio of change in the value of function to change in the
value of independent variable where the change in the latter is tending towards zero. Technically, if y =
y
f(x), then lim is the derivative of y with respect to x where
x  0  x

y denotes change in y
x denotes change in x
dy
This derivative is often denoted by or f(x) or y1.
dx
Therefore, derivation is the rate of change in the value of function when limit of change in the value
of independent variable is tending towards zero.
On the basis of above interpretation, the steps involved in differentiation are described below:
(i) introduces certain change x (delta x) in the independent variable x and find the corresponding
change in the value of function. Technically, if y = f(x), then introducing x change in x and y
change in y.
y + y = f (x + x)
y = (x + x) – y
= f (x + x) – f(x)
(ii) divide both side of x giving the ratio of change in value of function to change in the value of
independent variable. The same is also known as average rate of change.
y f  x  x   f  x 
Therefore =
x x
(iii) Lastly take its limit when x  0. This limit must exist for the derivative.
y f  x  x   f  x 
Therefore lim = lim
x  0 x x  0 x
dy f  x  x   f  x 
or = lim
dx x  0 x
The same is the derivative of y with respect to x. The derivative obtained by following steps is
known as derivative by first principle.
Example. If y = xn, find its derivative by first principle.
Solution. Given y = xn .
then y + y = (x + x)n (introducing change)
y = (x + x)n – xy
= (x + x)n – xn
 n  n  1 n  2 2 
=  x n  nx n 1x  x x ....  x n   x n
 2 
(Apply binomial distribution)
n  n  1
= nx n 1x  x n  2 x 2 ....  x n (simplifying)
2
Dividing both sides by x
n  n  1 n2 2
nx n1x  x x ....  x n
y 2
=
x x
n  n  1
= nx n 1  x n  2 x 2 ....  x n
2
Taking its limit when x  0.
y  n  n  1 n  2 
lim = lim  nx n 1  x x....  x n 
x  0 x x  0
 2 
dy
or = nxn–1.
dx

Derivative as Instantaneous Rate of Change


This derivative is defined both mathematically and geometrically. According to mathematical
interpretation, derivative is used both as velocity of the function and instantaneous rate of change. In
physical sciences, it is used as velocity of the function giving rate of movement. In commerce and
economics, derivative is used as instantaneous rate of change.
Instantaneous rate of change denotes the rate of change in the value of function at a particular value
of independent variable. Derivative gives the same as it is the rate of change in y when limit of rate of
change in x is tending towards zero which amounts to a particular value of x. Therefore derivative gives
the rate of change in the value of function at a given value of independent variable. This rate may be
constant or vary with the value of independent variable. It will be constant if derivative does not involve
independent variable, otherwise it will vary.
P
Example. The demand function of a commodity is x = 20 – . Find the rate of change in quantity
2
demanded.
P
Solution. Given x = 20 –
2
P  P
then x + x = 20 –
2
P
x = 
2
Dividing both sides by P and taking its limit when P  0
x 1
lim =
P0 P 2

or dx = – 1 which is constant at all prices.


dp 2
Example. The total cost of producing x units of a commodity is TC(x) = x3 + 100. Show that rate of
change in total cost is varying at all levels.
Solution. Given TC(x) = x2 + 100.
Introducing x change in x
TC(x) + TC(x) = (x + x)2 + 100
TC(x) = (x + x)2 + 100 – (x2 + 100)
= x2 + x2 + 2xx + 100 – x2 – 100
= x2 + 2xx.
Dividing both sides by x
TC  x 
= x + 2x
x
Taking its limit when x  0
TC  x 
lim = 2x
x  0 x
dTC  x 
or = 2x
x
which implies it is changing at all values of x.

Standard Forms of Differentiation


There are some standard forms of differentiation which can be used for obtaining the derivatives of
the functions appearing in different forms. All these standard forms are based on the first principle of
differentiation. (Proofs have been ommitted). These are applicable to all forms of functions as such. These
are as follows:
dy
1. If y = c (a constant) = 0
dx
dy c.d f  x
2. If y = c . f(x) =
dx dx
dy c.d f  x
3. If y = f(x) ± g(x) = (Summation form)
dx dx
dy d f  x d g  x
4. If y = f(x) . g(x) =  (Product form)
dx dx dx
f  x dy d g  x d f  x
5. If y = = f  x  g  x (Quotient form)
g  x dx dx dx

d f  x d g  x
g  x  f  x
dy dx dx
6. If y = f(u) and u = g(x) = (Chain rule)
dx g  x
2

7. Implicit Functions: Functions are also classified as explicit and implicit function. Explicit
functions are characterised as those where one variable can be expressed as a function of another variable
only as y = f(x). Otherwise it is taken as implicit function as f(x, y) = 0. For example, y = x2 + 2x + 4 is an
explicit function whereas x2 + 2xy + y2 = 0 is an implicit function. The derivative of implicit function is
obtained using chain rule. For example derivative of y2 with respect to x can be obtained assuming u = y2,
then
du
= du . dy
dx dy dx
On this basis, the rule is that all terms involving y will be first differentiated with respect to y and
dy dy
multiply it by . After this solve it for . The same is the derivative of implicit function.
dx dx
8. Parametric Functions: Two functions are taken as parametric functions if they have common
independent variable as y = f(t) and x = (t). This common independent variable is known as parametric.
Then
dy
dy
= dt
dx dx
dt
9. Inverse Functions: Two functions are taken as inverse if the independent variable of one function
becomes the dependent variable of the other and vice-versa. For example, if y = f(x), then x = f (y). Then
dy dx = 1
.
dx dy
dy 1
or =
dx dx
dy

Derivative of Algebraic Functions


Algebraic functions are defined as those functions where quantities are felated by mathematic
operations such as +, –, , power and square root only. For example, if
x2  4
y = x 2  100, y  etc.
x2
The rule of differentiation of algebraic functions is multiply the power of x in the base and reduce
the power by one. It is based on the first principle. So if
dy
y = xn , = nxn–1
dx
as proved by first principle earlier.
All the standard forms stated above are applicable as such.
x 1  x 1 dy x
Example. If y = , prove that =1+
x 1  x 1 dx x 1
2

x 1  x 1
Solution. Given y =
x 1  x 1
x 1  x 1 x 1  x 1
= 
x 1  x 1 x 1  x 1
(Rationalising the denominator)

x  1  x  1  2 . x2  1
=
x 1 x 1
2 x  2 . x2  1
=
2
 y = x+ x2  1

= 1   x 2  1 . 2 x
dy 1 1/ 2

dx 2
= 1+ x
x 12

x dy 
a a  a 2  x 2 
 
Example. If y = , prove that =
a  a2  x2 dx x2 a2  x2

Solution. Given y = x
a  a2  x2

x a  a2  x2
= . (Rationalisation)
a  a2  x2 a  a2  x2
x a  a 2  x
=  
a a  x
2 2 2
a  a2  x2
y =
x


dy
=

x . a a  a2  x2  a  a2  x2    dxdx
dx x3
(Quotient form)
1
x .   a2  x2 
2
1/ 2 
 2x   a  a2  x2

 
=
x2
 x2
 a  a2  x2
= a x2 2

x2
 x2  a . a2  x2  a2  x2
=
x2 . a2  x2

a  a  a 2  x 2 
=  
x . a x
2 2 2

t2 1 t2 1
Example. Given x = a . , y = at
t2 1 t2 1
dy t 4  2t 2  1
Shoow that =
dx 2t

t2 1
Solution. Given x = a
t2 1

dx   2 
= a  1  t 2  1
1/ 2
t 2
 1 2t   t 2  1 2t 

 2  t  1  t  1 
2
dt 2
 

 2 
1/ 2 
= a  t 2  1  . 2t 
 t 1  t 2  1 
2


2at
=
t  1 .  t 2  1
2 1/ 2 3/ 2

Similalry
dy   2 
= a . t . 1  t 2  1 
1/ 2

.
t 2
 1 2t   t 2  1 2t


t2 1 
 2  t 1 t  1 t2 1 
2
dt 2
 

2at 2 t2 1
= a.
t  1 t  1 t2 1
2 1/ 2 2 3/ 2
2at 2  a  t 2  1 t 2  1
=
t  1 t  1
2 1/ 2 2 3/ 2

2at 2  at 4  a
=
t  1 t  1
2 1/ 2 2 3/ 2

dy dy dx
= /
dx dt dt
2at 2  at 4  a
t  1 t  1
2 1/ 2 2 3/ 2

=
2at
t  1 t  1
2 1/ 2 2 3/ 2

at 4  2at 2  a
=
2at
t 4  2t 2  1
=
2t
dy
Example. Given x2y + xy2 = 10, find .
dx
Solution. Differentiating implicitly both sides,
dy dy
x2 . + y . 2x + x . 2y . + y2 = 0
dx dx
dy
Simplifying for
dx
dy 2
 x  2 xy  = – 2 xy  y 2 
dx 
  2 xy  y 2 
= 2
dy
dx x  2 xy

Derivative of Exponential Function


Another type of non-algebraic function used in business and economics is exponential functions.
These functions re defined as those where independent variable appears as power of some constant base.
For example y = 10x of y = ex likewise. These functions are classified as natural and other exponential
functions. If the base of e (basis of natural log system having an approximate value of 2.7183), the
function is known as natural exponential, otherwise other exponential function. Such functions are often
used ot denote continuous growth.
Derivative of natural exponential functions is based on the rule if
dy
y = ex, then = e x.
dx
It can be proved by first principle (proof ignored). Derivative of the other exponential function can
be obtained easily with the help of logarithmic differentiation which is explained in the next section.
Standard forms are applicable as such as exemplified below:
Example. Differentiate y = xn eax + b with respect to x.
Solution. Given y = xn eax + b . a + eax + b nxn–1
= xn – 1 eax + b[ax + n]
Example. Differentiate the following with respect to x.
2
x
(i) y = x ex (ii) y = e (iii) y = e 2x
 e x
x
1
(vi) y = 10ex–x/50 (v) y = (x + 2) e x  2
Solution. (i) y = x . ex
dy de x dx
 = x. + ex .
dx dx dx
= x . ex + ex
= ex [x + 1]
2
x
(ii) y = e
x
2
de x 2 dx
 ex .
dy
 = x . dx 2 dx
dx x

= x . e . 22x  e
2 2
x x

x
2
x
= e 2 2 x2  1
x
x2
(iii) y = e + e–x
dy d 2x d x
 = e 2x
.  e x .
dx dx dx
1
1
 2 x  2 .2  e x
2x 
= e .
2
2
x
= e 2 2 x2  1
x
(iv) y = 10 xe–x/50
dy  de x / 50 dx 
 = 10  x.  e  x / 50 . 
dx  dx dx 

= 10  xe x / 50  1  e x / 50 
 50 

= 10e–x/50   x  1
 50 
1
(v) y = (x + 2) e x  2
1
dy de x  2
1
d  x  2
 = (x + 2)  e x2 .
dx dx dx
1
1 1
= (x + 2) e x  2 .  e x2
 x  2
2

 1 
1

 x  2  1
x2
= e
 
 x 1 
1

= e x2  
 x  2

Derivative of Logarithmic Functions


Logarithmic Functions are also used for denoting functions used in business and economics. Such
functions are defined as those whose independent variable appears with log ratio as y = log ax or y =
log10x likewise. Such functions are also classified as natural logarithmic functions, common logarithmic
functions and other logarithmic functions based on the base of log. In case of natural log, base is e [a
constant with approximate value of 2.7183] as y = logax also denoted by y = lnx. But if base of log is 10, it
is known as common log as y = log10x also denoted by logx. If base of log is other than 10 and e, it is
known as other logarithmic function as y = logax and a  10 + e. In such cases base must be specified.
It is also possible to change the base of log as if y = lnx with base e, then with base 10 it becomes y
= log x whose log e is a constant.
log e
dy 1
Derivative can be obtained directly for only natural logarithmic functions as if x = lnx, then 
dx x
(prof has been ommitted). So if other than natural log function is given, first change its base to e and then
differentiate. The standard forms are applicable in the same way.
Example. Differentiate the following with respect to x.

(i) y = ln (x2 + 1) (ii) y = ln  1 x  2


(iii) y = ln2 (x + 1)

(iv) y = x . lnx (v) y = ln . x 1


x 1
Solution. (i) y = ln (x2 + 1)


dy
= 2
1
.
d x2  1  
dx x 1 dx
2x
=
x 1
2

(ii) y = ln  1 x  2

dy 1 d 1  x2
 = .
dx 1  x2 dx
1 1
1  x 2  .2 x
1/ 2
= .
1 x 2 2
x
=
1 x 2
(iii) y = ln2 (x + 1)
or y = [ln (x + 1)]2
dy 2 ln  x  1
 = 2 ln (x + 1) .
dx dx
2 ln  x  1
=
x 1
(iv) y = x . lnx
dy 1
= x. + lnx
dx x
= 1 + lnx.

(v) y = ln . x 1
x 1
x 1
dy x 1 x 1
= .d
dx x 1 dx

x 1 1  x 1
. 
1/ 2
 x  1   x  1
=  .
x 1 2  x 1   x  1
1/ 2
x 1  x 1  1
=   .
x 1  x 1   x  1
2

 1 
=  
  x  1 x  1 
1
= 
x 1
2

In addition to this, logarithmic differentiation can be used in different ways. It is based on the
derivative of logarithmic function. It basically involves three steps (i) taking natural log of both the sides,
dy
(ii) Differentiating implicitly both sides with respect to x, and (iii) solving for . Some forms based on
dx
these are as follows:
(a) Derivative of natural exponential function:
as y = ex . Taking natural log of both sides
lny = x . lne [lne = 1]
 lny = x
Differentiating both sides with respect to e
1 dy = 1
.
y dx
dy
 = y
dx
= e x.
(b) Derivative of other exponential functions as y = ax. Taking natural log of both sides
lny = x . lna [lna is a constant]
Differentiating both sides with respect to x.
1 dy = lna
.
y dx
dy
or = lna . y
dx
= lna . ax
(c) Derivative of a function raised to the power another function as y = [f (x)]g(x). It can be
differentiated only with the help of logarithmic differentiation. Taking natural log of both sides
lny = g(x) . ln f (x).
Differentiating both sides with respect to x

1 dy = g(x) . 1
. f (x) + ln f (x) . g(x)
f  x
.
y dx

dy  g( x ) 
=  . f ( x )  ln f ( x ).g ( x ) . y
dx  f(x) 
 g( x ) 
. f ( x )  ln f ( x ).g ( x ) .  f ( x )
g( x )
= 
 f(x) 
Logarithmic differentiation can also be applied for differentiating algebraic and exponential
functions if these appears in combined forms.
Example. Differentiate the following with respect to x :
x
(i) y = 5x + 2 (ii) y = ax . xa (iii) y = x e
(iii) y = x . lny (v) y = xlnx
Solution. (i) y = 5 x + 2. Taking natural log of both sides lny = (x + 2) lns. Differentiating both sides
with respect to x.
1 dy = lns
.
y dx
dy
= lnx . y
dx
= lns . 5x + 2
(ii) y = ax . xa

dy x
= a .
d x    x . d a 
n
a
x

dx dx dx
= ax . axa – 1 + xa . ax . lna
 da x 
= ax . xa–1 [a + x . lna]   a x .ln a 
 dx 
x
(iii) y = s x e . Taking natural log of both sides and differentiating.
lny = ex . lnx
1 dy = e x . 1  ln x . e x
.
y dx x
dy
= e x  1 ln x  .y
dx x 

= e x  1 ln x  xe
x

x 
(iv) y = x . lny. Differentiating implicity both sides with respect to x.
dy
= x . 1 . dy + lny
dx y dx

dy  x 
1
dx  y 
or = lny

dy  ln y 
 =
dx yx
(v) y = xlnx. Taking natural log of both sides and differentiating with respect to x.
lny = lny . lnx
= (lnx)2
 1 dy = d . ln x
.
y dx y
dy
= 2 .ln x . xln x
dx y
= 2 . lnx . xlnx – 1.
x

Example. If  x  ye x y
= a, then prove that
dy
y. + x = 2y
dx
x

Solution. given  x  ye x y


= a, taking natural log of both sides:

ln (x – y) + x . lne = lna
x y

or ln (x – y) + x = lna [as lne = 1]


x y
Differentiating both sides with respect to x.

 x  y   x 1 
dy 
1  dy  
 dx 
1    = 0.
x  y  dx   x  y
2
dy  1 x  1 y
   =  
dx  x  y  x  y  
2
x  y  x  y 2

dy  y  x  2 y
  =
dx   x  y  
2
 x  y
2

x  2 y
2 
. x  y
dy 2
y. =
dx  x  y
dy
y = = –x + 2y
dx
y dy
or + y = 2y.
dx
dy y2
Example. If y = x x
x .......
, prove that x . 
dx 1  yln x
x .......
Solution. Given y = xx
as y  x x 
x ....... 
= xy
 
Taking natural log of both sides and differentiating:
lny = y . lnx
1 dy = y . 1  ln x . dy
.
y dx x dx

dy  1  y
  ln x  =
dx  y  x

dy  1 
x = y 
dx 1  y .ln x 
y3
=
1 y .ln x
dy ln x
Example. If xy = ex – y, prove that 
dx 1  ln x 2
Solution. Given xy = ex – y, taking natural log of both sides.
y . lnx = (x – y) . lne
y . lnx = (x – y) (as lne = 1)
or y [lnx + 1] = x
or y = x
1 ln x
1
dy
1  ln x   x .
x
 =
1  ln x 
3
dx

ln x
=
1 ln x 
2

Applications based on derivation as instantaneous rate of change


As explained earlier, derivative is used as instantaneous rate of change in business and economics. It
denotes the rate of change in a function at a level. Whenever we want to find out the rate of change in the
function at a particular level then find the derivative of the function and evaluate it for that level. Some
examples are given below to explain this:
Example. A firm has a revenue function given by TR (x) = 10x where x is the units demanded and
TR (x) is gross revenue with a production cost function given by
2
 x 
TC (x) = 1,00,000 + 50  
1000 
Find the rate of change in profit if x = 1,00,000 units.
2
 x 
Solution. Given TR (x) = 10x, TC (x) = 1,00,000 + 50  
1000 
 Profit function, n(x) = TR (x) – TC (x)
2
 x 
= 10x – 1,00,000 – 50  
1000 
Rate of change in profit at x will be:
dx  x  2x
= 10 – 50
dx 1000000
x
= 10 –
10 ,000
Putting x = 1,00,000 units
1,00 ,000
= 10 – =0
10 ,000
So rate of change in profit at x = 1,00,000 is zero indicating that profit is constant at this level.
Example. The demand function of a commodity at x = 200 – p2 where x units are demanded
when price is Rs. p per unit. Determine the level of output where rate of change in total revenue is zero.
Solution. Given x = 200  p2

or P = 200  x 2

 TR (x) = P . x =  200  x2 x 
Rate of change in TR (x) will be:
dTR  x  1
 200  x 2 
1 / 2
= 200  x 2  x .  2x
dx 2
x2
= 200  x 2

200  x 2
200  2 x 2
=
200  x 2
Putting it equal to zero and solving for x
200  2 x 2
= 0
200  x 2
 200 – 2x2 = 0
x2 = 100
x = 10 units.
Therefore rate of change in total revenue is zero at x = 10 units.

APPLICATIONS
The applications based on simple differentiation (derivative concept) are explained below:

1. Measurement of price elasticity of demand


Price elasticity of demand is an important economic concept. It is based on law of demand. Low of
demand states the quantitative relationship between price and quantity demanded and if price is high,
quantity demanded is low and vice-versa. The same quantitative relationships are quantified by price
elasticity of demand. It is always negative because of negative relationships between price and quantity
stated by law of demand. In economics, it is defined as
Ey = Relative change in quantity demanded
Relative change in price
Symbolically, if demand function is x = f (p)
x
x p
then Ep = x  . for change in price.
p p x
p
This gives the price elasticity of demand for certain p change in price. But if price elasticity of
demand is required at a given price p instead of certain change then limit of p will tend towards zero and
Ep = lim x . p at a price p
p 0 p x

 x dx 
= dx . p as lim  
dp x  p 0 p dp 
If a demand function is given and price elasticity of demand is required at a price, it is given by
Ep = dx . p
dp x

where dx is the derivative of demand function with respect to p, p is the price at which elasticity is
dp
required, and x is quantity demanded at that price.
Example. The demand function of a commodity is x = 100  p 2 . Determine the price elasticity of
demand (i) when price decreases from Rs. 8 to Rs. 6, (ii) at price Rs. 8.
Solution. (i) Given 100  p 2
 when p = Rs. 8 x = 6 units
and when p = Rs. 6 x = 8 units
So p = Rs. 2 x = –2
 Ep = x . p  2 . 8  4 .
p x 2 6 3
(ii) At p = Rs. 8, I = 6 units

 Ep =
x p d
. 
 100  p 2 . p
p x dp x
p p
 100  p 
= .
2 x

 p2  100  p2  x given 
=
x3  
64 16
=  
36 9
Example. The linear demand function of a commodity shows that quantity demanded is nil when
price is Rs. 50 per unit and it is just 1000 units hen price is zero. Determine the price and quantity where
price elasticity and demand is unit.
Solution. Linear demand function can be obtained using intercept form as intercept of x-axis is 1000
units and on y-axis is Rs. 50 [given].
x P
 = 1
1000 50
or x + 20 P = 1000
or x = 1000 – 20 P ... (i)
Corresponding Ep = dx P  20P
dp x x
20 P
= –1 [unit mean –1 for Ep]
x
or x = 20 P ... (ii)
Solving (i) and (ii) simultaneously for P
1000 – 20 P = 20 P
or 40 P = 1000
 P = Rs. 25
Corresponding x = 20 (25) = 500 unit.
So it has unit elasticity when P = Rs. 25 and x = 500 units.
a
Example. The demand function of a commodity is P = where a and b are positive constants.
xb
1
Show that it has constant price elasticity of at all prices.
b
a
Solution. Given P = = ax–b
xb
dp
 = –abx–b–1
dx
dx = 1
or
dp abx b 1

Now Ep = dx . P
dp x
1 P P ax  b
= .   [Putting p = ax–b]
abx  b 1 x abx  b abx  b
1
=
b
1
It is constant at all prices. So whatever may be the price, price elasticity of demand remains .
b

2. Measurement of Price Elasticity of Supply


Price elasticity of supply at a price can be measured with the help of simple differentiation in the
same way as price elasticity of demand.
This concept is based on the law of supply which states the positive relationship between price and
quantity supplied (from firm’s point of view). It is defined for a supply function : x = g(P) as
Ep = Relative change in quantities supplied
Relative change in price
x
x P
= x  . for P change in price.
P P x
P
At given price, price elasticity of supply is given by:
Ep = lim x . p  dx . p
p 0 p x dx x
where dx is the derivative of supply function with respect to p. it always takes positive value
dp
because of positive relationship between price and quantity supplied stated by law of supply.
Example. The weekly supply of butter (x kg) depends on the price (Rs. p per kg) according to
x = 100 p  12 + 150
Determine the price elasticity of supply at p = Rs. 21.
Solution. Given x = 100 p  12 + 150
 when p = Rs. 21, x = 450 units.
 Ep = dx . p
dp x
1 1/ 2 p
= 100 .  p  12  .
2 x
50 p 50  21 7
=  
 p  12 x 3  450  9

3. Measurement of price elasticity of demand and supply at equilibrium price


In case demand and supply functions for a commodity are given, it is possible to measure both price
elasticity of demand and supply at equilibrium price using simple differentiation. The first step is the
determination of equilibrium price and quantity using demand and supply function. Equilibrium price is
the price at which both quantity demanded and supplied are equal and corresponding quantity is known as
equilibrium quantity, So
X = f(p) (demand function)
and X = g(p) (Supply function)
Then put f(p) = g(p) and some for p, this gives equilibrium price putting this in either demand
function or supply function and solving for x gives equilibrium quantity.
At this equilibrium price, price elasticity of demand is obtained by using demand function and price
elasticity of supply is obtained by using supply function. The only difference in this application and the
previous two is that in this elasticities are calculated at equilibrium price only whereas in earlier two at
any arbitrary price (given).
Example. A firm has the following demand and supply functions

P = 100  X 2 (Demand)
X = –10 + 2P (Supply)
Determine the price elasticity of demand and supply at equilibrium price.

Solution. Given P = 100  X 2 or X = 100  P


2

or P2 = 100 – X2.
For equilibrium condition, Put X = –10 + 2P in the demand function
 P2 = 100 – [–10 + 2P]2
= 100 – 100 + 40P – 4P2
or 5P2 – 40P = 0
P–8 = 0 (equilibrium price)
 P = 8
Corresponding equilibrium quantity
X = 10 + 2 (8) = 6 units
So price elasticity of demand at P = Rs. 8 with X = 6 units will be:

dX P d 100  P 2 P
Ep = .  .
dP X dP X
1 P
100  P  .  2 P .
2 1/ 2
=
2 X
P2 P2

 
=
100  P 2 X X

64 16
= 
36 9
dX P d  10  2 P  P
Ep = . = .
dP X dP X
2 P 2(8)
= 
X 6
8
= .
3
Example. A firm finds that both quantity demanded and supplied are 30 units when market price is
Rs. 8 per unit. It is known that demand is nil if price is increased to Rs. 12 per unit and firm is not willing
to supply if price is reduced to Rs. 5 per unit. Determine the firm’s price elasticity of demand and supply
at equilibrium price assuming linear relationship between price and quantity in both cases.
Solution. The demand equation being straight line, so its equation can be obtained as:
P  12 X 0
 =
12  8 0  30
or –30 (P – 12) = 4X
15
or x = 90 – P (Demand)
2
Similarly supply function will be
P5 X 0
=
58 0  30
or –30 (P – 5) = –3X
or X = 10P + 50 (Supply)
Using these price elasticity of demand and supply at equilibrium price, P = Rs. 8 will be:
dx P 15 P 15(8)
EP = . = .  = –2
dP X 2 X 2(30)
dx P 10 P 10(8) 8
ES = . =  
dP x x 30 0

4. Derivation of marginal revenue function [MR(x)] from total revenue function TR(x)
Total revenue function, TR(x), gives the total revenue of x units sold. It can be obtained from the
demand function as TR(x) = P . x where is the price when x units are demanded. On the other hand,
marginal revenue function, MR(x) gives the additional revenue of xth unit. In economics, marginal
revenue is defined as change in total revenue caused by change in number of units sold as additional
revenue of xth unit = TR(x) – TR (x – 1). It has an important relationship with total revenue as total
revenue will increase till marginal is positive, it will be zero if marginal is zero and decrease if marginal is
negative.
MR(x) can be obtained with the help of simple differentiation if TR(x) is given. MR(x) gives the rate
of change in total revenue which in turn is also given by derivative of total revenue function. Therefore,
MR(x) is equal to simple derivation of TR(x):
d TR  x 
 MR(x) =
dx
It gives the rate of change in total revenue at x giving additional revenue of (x + 1) the unit.
Example. The demand function of a commodity is x = 100  p 2 . Determine the corresponding
marginal revenue function and find marginal revenue when x = 6 units.
Solution. Given x = 100  p 2

or p = 100  x 2 (corresponding inverse function)

TR(x) p . x =  100  x  x 2

Now MR(x) =
d TR  x 
=
d  100  x 2 x 
dx dx
1
100  x  .  2 x
2 1/ 2
= 100  x 2  x .
2
x2
= 100  x 2 
100  x2
100  2 x 2
MR(x) =
100  x 2
100  2  6 
2
28 7
MR(6) =   = Rs. 3.50.
100   6  8 2
2

Example. The demand function of a commodity is p = 15 . 300  2 x where Rs. p is the price per
unit when x units are demanded. Determine the level of output where marginal revenue is equal to zero
and price corresponding to the level.
Solution. Given p = 15 . 300  2 x
 
TR(x) = p . x = 15 . 300  2 x x 
= 15 x . 300  2 x
d TR  x  d 15 x . 300  2 x
corresponding, MR(x) = =
dx dx

= 15  x 1  300  2 x 1/ 2 .  2  300  2 x 


 2 
 x 
= 15   300  2 x 
 300  2 x 
 300  3x 
MR(x) = 15  
 300  2 x 
Putting this equal to zero and solving for x
 300  3x 
15   = 0
 300  2 x 
or 300 – 3x = 0
x = 100 units
At x = 100 units, p = 15 300  2 100
= Rs. 150.
So marginal revenue is zero when x = 100 units and corresponding price is p = Rs. 150.
Example. A firm can sell 100 units if it charges Rs. 5 per unit. It knows that for each Rs. 0.10
reduction in price, it can sell 20 more units. Determine the firm’s marginal revenue function. Use it to
determine marginal revenue when x = 100 units.
Solution. Given when P = Rs. 5
quantity demanded, x = 100 units
 when P = Rs. 4
quantity demanded, x = 300 units.
(as for each Rs. 0.10 reduction, quantity demanded increases by 20 units, so if price declines 10
times to Rs. 4, quantity demanded also increases 10 times to 300 units).
Using two point form:
P5 X  100
=
54 100  300
or –200 (P – 5) = X – 100
or x = –200P + 1100
11 X
or P = 
2 200

So corresponding, TR(x) = P . X =  11  X  X
 2 200 
11 X2
= X
2 200
11 X2 
d X 
d TR  X  2 200 
 MR(X) = =
dx dx
11 X
= 
2 100
when X = 100 units
11 100 9
MR (100) =   = Rs. 4.50.
2 100 2

5. Measurement of Price elasticity in terms of average revenue and marginal revenue


Price elasticity of demand, as defined earlier, is the ratio of relative change in quantity demanded
and relative change in price. In economics there are three important methods of its measurement, viz. (i)
total expenditure method, (ii) point method, (iii) arc method.
According to point method, price elasticity of demand at a point on the demand curve is equal to
lower half of the tangent at that point divided by upper half of his tangent as in figure, price elasticity of
BC
demand of B is equal to .
AB

The same result, if further analysed using relationship between average revenue and marginal
revenue gives elasticity of demand:
AR
Ep =
AR  MR
 1 
or MR = AR 1  
 E p 
[For proof, see any text book on Economics].
The same result can be provided using simple differentiation. It is as follow:
TR(X) = P . X
d TR  X  dX dP
= P. X. [Using product form)
dX dX dX
dp
or MR(x) = P + X
dX
p
= P+X.
X . Ep
 dX p X . E p dX dP p 
Ep  . or    
 dp X P dP dX X . E p 

 1 
MR = P 1  
 E p 
 1 
= AR 1   [P = AR]
 E p 
 1 
MR = AR 1   [ Ep is always negative]
 E p 
AR
or Ep =
AR  MR
dp
Alternatively, MR = P + X .
dP
X dP
= P+P. .
P dP
[dividing and multiply second form by p]
 1 
= P  P . 
 E p 

 1   X dP 1 
MR = AR 1   as .  
 E p   P dX E p 
Example. AR and MR denotes the average and marginal revenue of any output, show that
AR
EP =
AR  MR
at this output. Also find the price elasticity of demand function P = a  bX and verify the above for the
price-elasticity.
Solution. First part already proved
Given P = a  bX
dP 1
=  a  bX  .  b =
1/ 2 b b
So 
dX 2 a  bX 2p
dX 2 P
=
dP b
So price elasticity of demand:
dX p 2 P p
Ep = . = .
dP x b X
2 P 2
=
bX
AR
Using Ep = ,
AR  MR
requires MR at this level which will be:
P = a  bX
 TR(X) = P . X = a  bX X
d TR  X 
Corresponding MR(X) = = a  bX + X
dX
 b 
  (Using product form)
 2 . a  bx 
bX
= P–
2P
2 P 2  bX
=
2p
p
Now Ep =
 2 P 2  bX 
P 
 2P 
P2 p
=
2 P 2  2 P 2  bX
2P 2
= ... (iii)
bX
So both (i) and (ii) are equal except negative sign. It is because
AR
Ep =
AR  MR
is based on the assumption that Ep is negative. It has already been incorporated in the formula.
Example. The demand equation of a commodity is x = 60 – P2. Determine the price elasticity of
demand when marginal revenue is Rs. 4.00.
Solution. Given X = 60 – P2
and MR = Rs. 4.00
So price elasticity of demand requires average revenue when marginal revenue is Rs. 4. It can be
obtained only if x is known at which MR = Rs. 4.00.
 P = 60  X

TR(x) = P . X =  60  X x 
d TR  x 
MR(x) =
dx
x  1
= 60  x 
2 60  x
2  60  X   X
=
2 . 60  X
2P2  X
MR(x) = =4 (Given)
2P
or 8P = 2P2 – X
or x = 2P2 – 8P ... (ii)
Using (i) and (ii) equations
60 – P2 = 2P2 – 8P
or 3P2 – 8P –60 = 0
3P2 – 18P + 10P – 60 = 0
3P [P – 6] + 10 [P – 6] = 0
(3P + 10) (P – 6) = 0
10
So either P = (not possible, so it is ignored)
3
or P = Rs. 6
or AR = Rs. 6 when MR = Rs. 4
AR 6
 Ep =  = 3.
AR  MR 6  4

6. Derivation of marginal cost function from total cost function


Total cost function gives the functional relationship between total cost and number of units
produced. Marginal cost function gives the additional cost of last unit. It can be obtained from total cost
function with the help of simple differentiation. The derivation of marginal cost function from total cost
function is explained:
d TC  X 
MC(X) = .
dX
Example. A firm’s total cost function is TC(X) = X3 + 2000. Determine the corresponding average
cost function, A(X) marginal cost function, M(X) and also determine the level of output where average
cost equals marginal cost.
Solution. Given TC(X) = X3 + 2000
TC  X  2000
So AC(X) = = X2 +
X X
d TC  X 
and MC(X) = = 3X2
dX
(Putting both equal and solving for X)
X 2  2000
= 3X2
X
or 2X3 = 2000
X3 = 1000
X = 10 units.

SUCCESSIVE DIFFERENTIATION
So far we have discussed the first order differentiation. But it is also possible to obtain the derivative
of the first derivative itself and this process may go on till it is possible. The process is known as
successive differentiation. It involves the derivative of the derivatives. These are also known as higher
order derivatives. The method and standard forms are same as in the case of first-order derivatives. For
example, if y = 12x2, its first-order derivative is 24x, its derivative 24 is known as second-order successive
dy d 2 y d 3 y
derivative, and likewise. These are denoted as , , and so on or y1, y2, y3 and so on or
dx dx 2 dx 3
f   x  , f   x  , f   x  and so on.
Example. If y = ln  x  1  x  , show that (1 + x2) y2 + xy1 = 0
2
 

y = ln  x  1  x  ,
2
Solution. Given
 
1  1 
1  2 1  x 
dy 2 1/ 2
 y1 = = . 2x
dx x  1  x2  
1 x 
= 1  
x  1 x  1  x2 
2

1  1  x2  x 
=  
x  1  x 2  1  x 2 

y1 = 1
1  x2
Cross-multiplying, squaring both sides
(1 + x2) (y1)2 = 1
Differentiating both sides with respect to x.
(1 + x2) 2y1y2 + (y1)2 . 2x = 0
Taking 2y1 common from both sides:
(1 + x2) + xy1 = 0.
1 1
d2y dy
Example. If 2x = y m  y m , prove that (x2 – 1)  x. = m2y.
dx 2 dx
1 1
Solution. Given 2x = y m  y m
Differentiating both sides w.r.t. I
1  m 1 dy  1   m 1 dy 
1 1
2 =  y .    y . 
m dx  m  dx 

1  m   dy
1 1
=  y  y m

my   dx
Cross multiplying:
1  m   dy
1 1
2my =  y  y m

my   dx
dy
= 2 . x2  1 .
dx

 a  b   a  b   4ab
2

= 2
 m1  
1 1

 y  y m
  4 y m
.y
 
= 4 x2  4

= 2. x2  1
dy
or my = x2  1 .
dx
Squaring both sides and differentiating implicitly w.r.t. x
2
 dy 
(my)2 = (x2 – 1)  
 dx 
2
dy d 2 y  dy 
m2 . 2y
dy
dx
=  x  1 2 dx . dx2   dx  . 2 x
2

dy
Taking 2 as common from both sides
dx
d2y dy
m2y = (x2 – 1) 2
x
dx dx
d2y dy
or (x2 – 1) 2
x = m2y.
dx dx
Example. If y = x 1  x  1 , prove that
1
(x2 – 1) y2 + xy1 = y.
4
Solution. Given y = x 1  x 1

1 1
y1 = 
x . x 1 2 . x 1
x 1  x 1
y1 =
2 . x2  1
y
=
2 x2  1

or 2. x 2  1 . y1 = –y
Squaring both sides and differentiating w.r.t. x
4(x2 – 1) (y1)2 = y2
4[(x2 – 1) 2y1y2 + (y1)2 . 2x] = 2yy1
Taking 2y1 as common
4[(x2 – 1)y2 + xy] = y
1
or (y2 – 1)y2 + xy1 = y.
4
Example. If y = (a + bx) e–nx, show that
y2 + 2ny1 + n2y = 0
Solution. Given y = (a + bx) e–nx
 y1 = (a + bx)e–nx – n + e–nx . b
= – ny + e–nx . b [product form]
[Putting y = (a + bx)e–nx]
and y2 = –ny1 + e–nx . –bn
= –n [–ny + c–nx b] + e–nx . –bn
= n2y – nbe–nx – nbe–nx
= n2y – 2nbe–nx
Talking L.H.S. and putting values of y1 and y2
y2 + 2ny1 + n2y
or n2y – 2nbe–nx + 2n [–ny + be–nx] + n2y
or n2y – 2nbe–nx – 2n2y + 2nbe–nx + n2y
= 0 R.H.L.

Interpretation of first derivative and second derivative


There exist an important relationship between first derivative and second derivative. Both these
together enables us to know the dimension of change in the nature of function and rate of change in that
dimension.
The first derivative gives the rate of change in the value of the function (also known as slope of the
function). It may be positive, zero or negative indicating whether the function is increasing, constant or
decreasing.
But even if it is increasing, it may be at increasing rate (in case of increasing return) at constant rate
(in case of constant return) or at decreasing rate (in the case of third phase of laws of returns). For
instance, in case of imperfect competition, total revenue increases upto certain level but at a decreasing
rate. The same can be answered with the help of algebraic value of second derivative. If the second
derivative is positive, function changes at increasing rate; if zero at constant rate; and if negative at
decreasing rate.
Therefore to conclude, if first derivative gives the dimension in which value of the function change
dy
and second derivative gives the rate at which it is changing in that dimension. For example: if 0
dx
d2y dy d2y
and 2  0 , it means function is increasing at increasing rate; if  0 and 2 = 0, it means function
dx dx dx
dy d2y
is increasing at constant rate if  0 and  0 , it means function is decreasing at increasing the
dx dx 2
rare. Mathematically, first derivative gives the rate of change of function, second derivative gives the rate
of change of the rate. First derivative gives the slope of the function whereas second derivative gives
slope of the slope. First derivative gives the velocity whereas second derivative gives the rate of
acceleration. First derivative give the rate of change in total function whereas second derivative gives rate
of change in marginal functions.
Example. A firm selling its product under imperfect competition is given by x = 100  p 2 . Show
that the corresponding total revenue function increases at decreasing rate till it is increasing.
Solution. Given x = 100  p 2
or p = 100  p 2

So corresponding TR(x) = p . x =  100  p  x 2

Rate of total revenue is given by:


d TR  x  1
100  x   2 x
2 1/ 2
= 100  p 2 . x .
dx 2
 x2 100  x 2  x 2
=
100  x 2 100  x 2
100  2 x 2
=
100  x 2
100  2 x 2
=
p
It will be positive till x = 10 units, to TR(x) is increasing till x = 10.
d 2TR  x 
Rate of increase till x = 10 is given by
dx 2

=
100  x   4x   100  2x 
2 2

100  x2
100  x   4 x   100  2 x 
2 2

=
100  x  2 3/ 2

300 x  2 x 2 
= 
 
3/ 2
100  x 2
which is negative so total revenue is increasing till x = 10 units, but at decreasing rate.
Example. Show that the total cost function, TC(x) = ax2 + bx + c where a > 0, b > 0; c > 0 is an
increasing function at increasing rate.
Solution. Given TC(x) = ax2 + bx + c
d TC  x 
So rate of change in TC(x) = = 2ax + b
dx
which is positive as both a and b are positive. So is it increasing function.
d 2 TC  x 
Further = 2a > 0, so it is increasing at increasing rate.
dx 2

Applications based on Successive Differentiation


In addition to, second derivative gives the rate of change in marginal functions, there are a number
of applications in business and economics based on the successive differentiation. Some of these are
explained below.
1. Determining the convex and concavity of functions
In economics, the concept of convexity and concavity is significant. There are a number of curves
which should be either convex or concave from below. For example, indifference curve must be convex
from below whereas production possibility curve is concave from below. The same can be determined
with the help of algebraic value of first derivative and second derivative.
A function is taken as convex from below if tangent at all points is below the curve (or curve is
above the tangent). There can be three possibilities in this case as shown below.
At x = x1
d g  p d 2 g  p
In case (i) > 0 but >0
dp dp 2
d AC  x  d 2 AC  x 
In case (ii) = 0 but >0
dx dx 2
d f  p d 2 f  p
In case (iii) < 0 but > 0.
dp dp 2

(i) (ii) (iii)


Supply function Average cost function demand function

For inputs, a function is taken as convex from below if its second derivative id positive irrespective
of the value of first derivative. But it must be positive in case of supply function as shown in figure (i), it
must be negative in case of demand function as shown in figure (ii), it may be first negative, then zero and
positive in case of average cost function as shown in figure (iii). In addition to this, tangent position
changes in anti-clockwise direction in this case.
Otherwise function is taken as concave from below. In this case, tangent is above the curve (or curve
is below tangent). There can be three possibilities in the case also as shown below.
(i) (i) (i)
Total Revenue Average Production Production Possibilities
function function curve

At x = x1
d TR  x  d TR  x 
2
In (i) case, > 0 but <0
dx dx 2
d AP  x  d 2 AP  x 
(ii) case = 0 but <0
dx dx 2
d f  x d 2 f  x
(iii) case < 0 but <0
dx dx 2
So in each case, first derivative may take any value, but second derivative is always negative. In
addition to this, tangent changes its position in clockwise direction.
a
Example. Show that the demand curve P = – C is downward sloping and convex from below
xb
(p is price x is quantity and a, b and c are positive constants).
a
Solution. Given p = –c
xb
dp
 = <0
dx
d2 p 2a
So demand curve is sloping downwards  > 0 which implies demand curve is convex
dx 2
 x  b
from below.
Example. An indifference map is defined by the relation (x + h) y  k = a where h, k and a are
positive constants. Show that the indifference curves are downwards sloping and convex from below:
Solution. Given (x + h) yk = a
a
or x+h =
yk
a
or x = –h
yk
dx = a
 3/ 2 < 0
dy 2 y  k 
So indifference curves are sloping downward
2
d x 3a
= 3/ 2 < 0,
4 y  b
2
dy
So in difference curves are convex from below.
d2 p 
Example. It is given that a demand curve is convex from below  2  0  at all points where p is
 dx 
price per unit and x is quantity demanded. Show that the marginal revenue curve is also convex from
d2 p d2 p 3 d2 p
below if either 2
is positive or 2
is negative and numerically less than . . Does a similar
dx dx x dx 2
property held for demand curve that are concave from below.
Solution. Let the demand function is p = f(x), then corresponding total revenue function:
TR(x) = p . x
d TR  x  dx dp
 = MR(x) = p .  x. [Product form]
dx dx dx
dp
or MR(x) = p + x .
dx
d MR  x  dp d 2 p dp dx
 =  x. 2  .
dx dx dx dx dx
dp d 2 p dp
=  x. 2 
dx dx dx
2 dp d2 p
=  x. 2
dx dx
For its convexity from below:
d 2 MR  x  d2 p d 3 p d 2 p dx
= 2. 2
 x. 3  2 .
dx dx dx dx dx
3d 2 p d3 p
= 2
 x. 3
dx dx
must be positive.
3d 2 p d3 p
  x . = 0
dx2 dx3
3 d2 p d3 p
or .  3 = 0 (Dividing both sides by x)
x dx2 dx
d3 p 3 d2 p d3 p d2 p
if is positive, then .  will be positive as is already positive (Given). So
dx 3 x dx2 dx 3 dx2
corresponding MR(x) is convex from below:
d2 p 3 d2 p d 2 p
If < 0, then .  (Given)
dx2 x dx2 dx 2
d2 p 3 d2 p
So MR(x) is convex also if < 0 but less than . .
dx2 x dx2
d2 p
In case demand curve is concave from below will be negative, then
dx2
d 2 MR  x  3 d2 p d3 p
(i) = .  3
dx x dx2 dx
2
d p 3 d2 p d3 p
will be positive if > 0 and .  3 which is not fulfilled. So MR(x) will not be convex
dx2 x dx2 dx
from below if demand curve is concave from below in this case.
d3 p 3 d2 p d3 p
(ii) if 3
< 0 then . 2  3 < 0,
dx x dx dx
So MR(xx) is again concave from below.
2. Determining the relatively extreme values of functions
Another important application based on successive differentiation is determination of relatively
extreme values of a function (also known as maxima and minima). This in turn, has a number of
important applications in business and economics.
It involves the following steps:
(a) Determining the point where it takes relatively extreme value : if y = f (x) is the function, then
put dy/dx = 0 and solve for x, gives the value of x where f (x) takes relatively extreme value.
(b) Determining whether function is taking relatively maximum, relatively minimum or point of
inflection (neither increasing, nor decreasing) at the point given by step (a), as
d2 y
if > 0, relatively minimum.
dx2
d2 y
if = 0, point of inflection.
dx2
d2 y
if < 0, relatively maximum.
dx2
It is also possible to determine the relative maxima and minima using first derivative in place of
second derivative in the second step. First step is same in this situation also.
dy
(i) It is relatively maximum at the point where = 0, if derivative is positive for this any value of x
dx
before this and negative for any after this.
dy
(ii) It is relatively minimum at the point where = 0, if first derivative is negative for any values
dx
of x before this and positive for any value after this.
(iii) It is a point of inflection of first derivative is zero for some value before and after this.
x 2  27
Example. Find relatively extreme values of a function y = and also determine maxima and
x6
minima.
x 2  27
Solution. Given y =
x6
For relatively extreme values its first derivative will be equal to zero.
dy  x  6  2 x   x 2  27 
 =
 x  6
2
dx

x 2  12 x  27
=
 x  6
2

Putting it equal to zero and solving for x


x 2  12 x  27
= 0
 x  6
2

x2 – 12x + 27 = 0
(x – 3) (x – 9) = 0
So either x = 3 or x=9
 x  6   2 x  12    x 2  12 x  27  . 2  x  6 
2
d2 y
When x = 3, =
dx2  x  6
4

9  2 x  12  53 2
=  
 x  6
4
81 3

d2 y 54 2
and when x = 9, 2
= 
dx 81 3
Therefore at x = 3, it takes relatively maximum value, i.e., 18 and at x = 9, it takes relatively
minimum value, i.e., 6.
Example. Find the value of a, b and c of a function
f (x) = ax2 + bx + c
if it has relatively maximum value of 7 at x = 1 and the function passes through the point (2, –2).
Solution. Given f (x) = ax2 + bx + c
If it has relatively maximum value of 7 at x = 1
d f  x
 = 2ax + b = 0
dx
Putting x = 1, 2a + b = 0 ... (i)
Also when x = 1, f (x) = 7
and when x = 2, f (x) = –2
a+b+c = 7 ... (ii)
4a + 2b + c = –2 ... (iii)
Subtracting (ii) and (iii),
3a + b = –9 ... (iv)
Solving (i) and (iv) simultaneously for a and b
a = –9, b = 18
Putting these in (ii) equation, c = –2.
3. Determining the equilibrium of firm
Equilibrium is described as the state of stability. A firm is said to be in equilibrium when it is not
willing to change its price and output. It occurs when firm makes maximum profit (as in economic
analysis, it is assumed that a firm is willing to maximise profit).
It can be determined with the help of first derivative and second derivative if firm’s demand
function with total cost function is given. The method is based on the conditions given in economics.
A firm’s equilibrium occurs at that level where marginal revenue equals marginal cost and marginal
cost cuts marginal revenue curve from below. Mathematically, it requires:
(i) Putting MR(x) = MC(x) and solving for x
d 2TR  x  d 2Tc  x 
(ii) At this value of x,  , for profit maximisation.
dx 2 dx 2
If the second condition is not fulfilled, the firm will not make maximum profit.
Example. The demand function of a commodity is
P = 1000 – 2x
 TR(x) = P . x = (1000 – 2x) x
= 1000x – 2x2
d TR  x 
and MR(x) = = 1000 – 4x ... (i)
dx
Similarly TC(x) = x3 – 5ax2 + 1315x + 2000
d TC  x 
 MC(x) = = 3x2 – 118x + 1315 ... (ii)
dx
For equilibrium MR equals MC
 Putting MR(x) = MC(x) and for x:
1000 – 4x = 3x2 – 118x + 1315
or 3x2 – 114x + 315 = 0
or x2 – 38x + 105 = 0
or (x – 3) (x – 35) = 0
 either x = 3 or x = 35
d 2TR  x 
At x = = –4
dx 2
= 6x – 118 = –100
So –4 > 100.
d 2TR  x 
At x = 35, = –4
dx 2
d 2TC  x 
= 6x – 118 = 92
dx 2
So –4 < 92
 Firm is making maximum profit when x = 35 unit as
d 2TR  x  d 2TC  x 
2

dx dx 3
At this level, P = 1000 – 2 (35) = Rs. 930.
MAXIMA AND MINIMA
In calculus, extreme values of a function are of two types, viz., relatively extreme and absolutely
extreme values. Relatively extreme values also known as local extreme values. They are extreme in
relation to surrounding values of a function. Some are described are maxima and manima. Absolutely
extreme values are extreme in the entire range of the function. These values are also known as global
extreme values or maximum and minimum. These may be more than one relatively extreme values of a
function but there is only one absolutely extreme value of a function in the figure.

f (x) has relatively maximum values at B and D and relatively minimum of point A and C. But it has
absolutely maximum at B and absolutely minimum at C.
Calculus gives only relatively extreme values. For example if a profit function is given, similarly if
average cost function is given, objective is to determine the level of output where cost per unit is
minimum.
Absolutely extreme values can be obtained from relatively extreme values by simple comparison. If
a function has two relatively maximum values, then absolutely maximum can be obtained by comparing
both relatively maximum values of a function. But if the function has only one relatively extreme value it
becomes absolutely extreme also.
The relatively extreme values can be obtained using calculus in the same way as explained in the
determination of relatively extreme values under Successive Differentiation. The step required are
summarised below:
dy
(i) If y = f (x), then put = 0 and solve for x.
dx
d2 y
(ii) At this values of x, if > 0, y takes relatively minimum value,
dx2
d2 y
if = 0, point of inflection, and
dx2
d2 y
if < 0, y takes relatively maximum values.
dx2
Example. If f (x) = ax2 + bx + c, a  0, prove that the point
 b 4ac  b 2 
 , 
 2a 4a 
is a relatively maximum if a < 0 and is a relatively minimum if a > 0.
Solution. Given f (x) = ax2 + b + c
d f  x
 = 2ax + b.
dx
Putting it equal to zero and solving for x for relatively extreme value.
 2ax + b = 0
x = –b/2a
b
At x =
2a
d 2 f  x
= 2a
dx 2
It will be positive if a > 0, so taking relatively minimum value of

 b  = a  b   b  b   c  4ac  b
2 2

f   2a   2a 
 2b      2a
It will be negative if a < 0, so taking relatively minimum value of
4ac  b 2
2a

Applications in business and economics


There are a number of important applications in business and economics based on maxima and
minima. Some of these involves maximisation, others minimisation depending on the nature of the
function. Some of the important applications are explained below:
1. Maximisation of total revenue function
If a firm’s demand function is given, it is possible to determine the corresponding level of output
whose total revenue is maximum using maxima and minima. But it will be possible only if firm is
operating under imperfect competition (where price decreases as firm sells more units). In case of perfect
competition, price is constant for a firm resulting in total revenue as an increasing function. The total
revenue is maximum when marginal revenue is zero.
d TR  x 
(i) Put = 0 and solve for x.
dx
d 2TR  x 
(ii) The value of x, where ,
dx 2
Tr(x) is maximum.
Example. The demand function of a commodity is p = 15e–x/3 for 0 d x d 8 where Rs. p is the price
per unit when x units are demanded. Determine the price and quantity for which total revenue is
maximum.
Solution. Given p = 15e–x/3
 TR(x) = p . x = 15e x / 3 
x

= 15xe–x/3
dTR  x 
and = 15  x . e x / 3 . 1  e x / 3  1
dx  3 

= 15e x / 3   x  1
 3 
For maximisation, put it equal to zero and solve for x
  x  = 0
15e x / 3   1
 3 

So either 15e x / 3 = 0 or   x  1 = 0


 3 
x
 1 = 0. [15e–x/3 = 0 not possible as it is equal to p]
3
 x = 3 units.
d 2TR  x  
x/3 1   x   x / 3 1
At x = 3, = 15 e .    1 e . 
dx 2
 3  3  3

= 5e x / 3 1  x  1


 3 
= –5e–1 < 0.
So TR(x) is maximum when x = 3 units
At this level, p = 15e–1.
Example. A watermelon grower wishes to send his produce to the city market as early as possible in
the season to catch the prices. He can send now 6 tons at a price of Rs. 60 per ton. By waiting, he
estimates that he can add 3 tons per week to what he can send now, but then the price will be reduced by
Rs. 5 per ton per week. How long should he wait for maximum revenue (use the method of Calculus).
Also determine price at that level and maximized total revenue.
Solution. Let watermelon waits for x weeks for maximum revenue.
Then quantity after x weeks = (6 + 3x) tons and price after x weeks = Rs. (60 – 5x) per ton.
 Total revenue after x weeks:
TR(x) = (60 – 5x) (6 – 3x)
For maximisation, take its first derivative w.r.t. x
dTR  x 
= (60 – 5x) (6 + 3x) – 5
dx
Putting it equal to zero and solving for x
150 – 30x = 0
or x = 5 weeks
d TR  x 
2
At x = 5 weeks, = – 30 < 0.
dx 2
Sp TR (x) is maximum at x = 5 weeks.
When p = (60 – 5x) = (60 – 25) = Rs. 35 per ton.
and TR(5) = 35(21) = Rs. 735.
Example. A shoe manufacturer produces both high grade and low grade shoes in units of x hundreds
of pairs and y hundreds of pais per day respectively. Assume that the equipment in plant is capable of
producing the shoes is this relationship:
42  5 x
y = (x < 10)
10  x
if the high grade shoes are sold at a price twice the low grade shoes. Determine the pairs of both be
produced to mximize total revenue.
Solution. Let the price of low grade shoes be Rs. p per pair, then price of high grade shoes will be
Rs. 2 p per pair. Accordingly total revenue will be:
TR = 2px + py
or TR(x) = 2px + p  42  5 x   42  5 x 
 Putting y  10  x 
 10  x   
For maximizing TR(x), take its first derivative w.r.t. x, put it equal to zero and solve for x

dTR  x 
 10  x  .  5   42  5 x  .  1 
= 2p + p  
10  x 
2
dx  

 8 
= 2p + p  
 10  x  
2

Putting it equal to zero and solving for x


 8 
2p + p   = 0
 10  x  
2

8
= 2 [Cancelling p from both sides]
10  x 
2

or (10 – x)2 = 4
 10 – x = +2 or 10 – x = –2
or x = 8 or 12
 x = 8 [x < 10, to x = 12 not possible]
d TR  x 
2
16
At x = 8  which is negative, so
10  x 
2 2
dx
 42  5  8  
RT(x) is maximum when x = 8, putting this in transformation function y =   = 1.
 10  8 
Therefore, total revenue is maximum when manufacturer produces 800 pairs of high grade and 100
pairs of low grade.
2. Minimisationof Average cost function
Maxima and minima can also be used for determining the level of output where per unit cost is
minimum corresponding to given total or average cost function. It is often used in economic analysis. It
gives the optimum level of output.
dA  x  d 2 AC  x 
(i) Put = 0 and solve for x (ii) the value of x for which > 0, per unit cost is
dx dx 2
minimum corresponding average cost can be obtained by putting this x in AC(x).
Example. A train which is found to consume fuel at the rate of (0.0002x2 + 0.02x + 0.32) gallons
per hour when travelling at x miles per hour. Determine (i) the speed at which the fuel consumption per
mile is at minimum. (ii) if fuel cost Rs. 0.50 per gallon and other costs are Rs. 0.09 per hour, what speed
will minimize the total cost per mile.
Solution. Given fuel consumption for x miles is (0.0002x2 + 0.02x + 0.32) gallons when speed is x
miles per hour.
TC(x) = 0.0002x2 + 0.02x + 0.32
Corresponding AC(x) giving per miles fuel consumption will be:
AC  x  0.32
AC(x) =  0.00002 x  0.02 
x x
For minimising it, take its first derivative w.r.t. x
dAC  x  0.32
 = 0.0002 
dx x2
Putting it equal to zero solving for x
0.32
0.0002 – = 0
x2
or x2 = 1600
x = 40 miles per hour.
d 2 AC  x  0.64
At x = 40 miles per hours, = >0
dx 2
x3
 The fuel consumption is minimum at x = 40 miles per hour.
(ii) If fuel colst Rs. 0.50 per gallon and other costs are Rs. 0.09 per hour, then total cost of x miles
travelled in one hour will be
TC(x) = 0.50 [0.0002x2 + 0.02x + 0.32] + 0.09
So corresponding average cost per mile will be
TC  x 
AC(x) = = 0.50 0.0002 x  0.02  0.32   0.09
x  x  x
For minimising AC(x), take its derivative w.r.t. x
dAC  x 
= 0.50 0.0002  0.32
2 
 0.09 0.25
 2  0.0001  2
dx  x  x x
Putting it equal to zero and solving x
0.25
0.0001 – = 0
x2
or x2 = 2500
or x = 50 miles per hour.
d 2 AC  x  0.25
At x = 50 miles per hour, = > 0.
dx 2
x2
 The cost is minimum when speed is 50 miles per hour.
Example. A machine initially costs Rs. 6,400 with no scrap value. The cost of operating is Rs. 500
in the first year and increases to Rs. 800 in each successive year. Determine (i) the number of years it
should be operated for minimising total operating cost per year, and (ii) corresponding cost per year.
Solution. Capital cost = Rs. 6400
Operation cost = Rs. 500 + Rs. 1300 + Rs. 2100.... upto x years.
x
= [2(500) + (x – 1)800]
2
[Second x term of an A.P.]
x
= [800 + 200]
2
= 400x2 + 100x
 TC(x) = operating cost + capital cost
= 400x2 + 100x + 6,400.
Corresponding per year cost function will be:
TC  x  6400
AC(x) = = 400x + 100 + .
x x
For minimising it, take its derivative w.r.t. x
dAC  x  6400
 = 400 –
dx x2
Putting it equal to cost and solving for x.
6400
400 – = 0
x2
or x = 4 years
d AC  x 
2
12800
At x = 4 years, = > 0. So cost per year is minimum if machine is
dx 2
x3
6400
operated for 4 years. Corresponding AC(x) = 400 (4) + 100 + = Rs. 3300.
4
3. Maximisation of profit function
If a firm’s demand function with total cost function is given, it is possible to determine the firm’s
equilibrium by maximizing the corresponding profit function in other situations also the same can be
applied, if profit function can be obtained. It is taken as profit, function (x) = TR(x) – TC(x).
d x d 2  x 
For maximizing if (i) = 0 and solve for x, (ii) the value of x for which < 0, profit is
dx dx 2
maximum corresponding profit can be obtained by putting in (x).
Example. The total cost function of a firm is
1 3
TC(x) = x – 5x2 + 28x + 10
3
where TC(x) is the total cost of x units. A tax at the rate of Rs. 2 per unit of output is imposed and
the producer adds it to his cost. If the market demand function is given by
P = 2530 – 5x
where P is the price per unit of output, determine the profit maximising price and output.
Solution. Given P = 2530 – 5x
1 3
TC(x) = x – 5x2 + 28x + 10
3
corresponding TR(x) = P . x,
1 3 1
After tax TC(x) = x – 5x2 + 28x + 10 + 2x = x3 – 5x3 + 30x + 10
3 3
= (2530 – 5x)x
= 2530x – 5x2
(x) = TR(x) – TC(x)
1
= 2530x –5x2 – x3 + 5x2 – 30x – 10.
3
1 3
= –x + 2500x – 10
3
For maximising (x), take its derivative w.r.t. x
d x
= –x2 + 2500
dx
Putting it equal to zero and solving for x
–x2 + 2500 = 0
or x = 50 unit.
d 2  x 
At x = 50 units = –2x < 0
dx 2
So profit is maximum when x = 50 units,
Corresponding P = 2530 – 5 (50) = Rs. 2280.
Example. The relationship between sales, TR(x) and advertising cost, Rs. x is given by
32000 x
TR(x) =
500  x
It is known that gross profit is 25 per cent of the sales. Determine (i) the corresponding net profit as
a function of x, (x) (ii) the value of x which mximises (x) and (iii) maximized (x).
32000 x
Solution. (i) Given TR(x) =
500  x

So gross profit = 1  32000 x 


4  500  x 
8000 x
=
500  x
 (x) = Gross profit – x (advertisement cost)
8000 x
= – x.
500  x
(ii) For maximizing it, take its first derivative w.r.t. x,
d x  500  x  8000  8000 x
= –1
 500  x 
2
dx

8000  500 
= –1
 500  x 
2

Putting it equal to zero and solve for x


8000  500 
–1 = 0
 500  x 
2

(500 + x)2 = 8000 (500)


500 + x = 2000
x = Rs. 1500
d 2  x  8000000
At x = Rs. 1500, = <0
 500  x 
3
dx 2

So (x) is maximum when x = Rs. 1500.


8000 1500 
(iii) Corresponding profit  (1500) = – 15000
500  1500
= Rs. 4500.
Example. A coal dealer has 80 tons of coal in stock which can be sold at the rate of Rs. 3.00 per kg.
in this month. The dealer knows that price will increase by Rs. 0.10 per month on an average if he does
not sell in the current month, but coal looses weight by 1 per cent per months with a storage cost of Rs. 30
per ton per month. Determine (i) the dealer’s net revenue function assuming he stores for x months, (ii)
the profit maximizing value of x (iii) price at that level, and (iv) maximised profit.
Solution. (i) Let assume dealer stores for x months,
 Price = Rs. (3 + 0.10x) per kg.
Quantity = (8000 – 800x) kg.
Storage cost = Rs. 2400x
So (x) = Px
= (3 + 0.10x) (80000 – 800x) – 2400x
= 2,40,000 + 3200x – 80x2
(ii) For maximising, take its first derivative w.r.t. x
d x
= 3200 – 160x
dx
Putting it equal to zero and solving for x
3200 – 160x = 0
x = 20 months.
d 2  x 
At x = 20 months, = –160 < 0
dx 2
So (x) is maximum when x = 20 months.
(iii) Corresponding price, P = Rs. [3 + 0.10 (20)]
= Rs. 5.00 per kg.
(iv) Maximised profit, (2) = 2,40,000 + 3200(20) – 80(20)2
= Rs. 2,72,000.
4. Minimisation of Total Cost
Usually, total cost function gives the functional relationship between total cost and the number of
units produced. It means it is an increasing function, so minimisation of this is not possible. But
sometimes, the total cost is defined in such a way that, it decreases upto certain level and then increases.
Only in such cases, this application is relevant. The step involves for minimisation are as follow:
dTC  x 
(i) Put = 0 and solve for x
dx
d 2TC  x 
(ii) The value of x for which > 0,
dx 2
TC(x) is minimum.
Example. An open tank with a square base and vertical sides is to be constructed of the metal sheet
so as to hold a given quantity of water. Show that this cost of material will be least when depth is half of
the width.
Solution. Let capacity tank = y cubic units and its base = x units, with height = h units.
Corresponding volume of open tank will be:
length . breadth . height = y
or x.x.h = y
y
or h = units.
x2
If cost of material is Rs. C per sq. unit, then total cost,
TC(x) = cost of bottom material + cost town sides material.
= C [Area of bottom] + C [Area of front and back]
= + C [Area of two sides]
TC(x) = cx2 + 2c  x . y2   2c  x . y2 
 x   x 
2cy 2cy
= cx 2  
x x
4cy
TC(x) = cx 2 
x
For minimising it, take its derivative w.r.t. x
dTC  x  2cy
= 2cx –
dx x2
Putting it equal to zero and solving for x
4cy
2cx – = 0
x2
4cy
2cx =
x2
2 y 2x2 y
or x =  2 [as y x2h]
x2 x
or x = 2h
d TC  x 
2
8cy
At x = 2h, = 2c +
> 0.
dx 2
x3
So TC(x) is minimum when depth is half of the width.
440
Example. A cylinder with a capacity of cubic ft. is made of two types of material. The cost of
7
material of the top and bottom is Rs. 6 per sq. ft and the sides material cost Rs. 4.80 per sq. ft. Determine
the dimensions of the cylinder which minimizes total cost and minimized total cost.
440
Solution. Given volume = cubic ft.
7
440
So r2h = cubic ft.
7
440
h =
7r 2
440 7
=
7 22r 2
20
=
r2
Corresponding cost of material of top and bottom
= [Total area of top and bottom] × Rate
= (2r2) 6 = Rs. 12r2
Cost of sides = {Total area of sides} × Rate
= (2rh) . Rs. 4.80
= Rs. 9.60 rh
TC(r) = 12r2 + 9.60r .  20 
2 
r 
192
= 12r 2 
r
For minimising, TC(r), take its first derivative w.r.t. r
dTC  r  192
= 24r 
dr r
Putting it equal to zero and solving for r
192
24r  = 0
r2
192 
r2 = =8
24 
r = 2 ft.
At r = 2ft.
d 2TC  r  384
= 24r 
dr 2
24r 3
So at r = 2 ft, TC (x) is minimum
20
Corresponding h = = 5 ft.
 2
2

and minimised TC (2) = 48x + 96


= Rs. 144.
5. Maximisation of tax Revenue
The theory of maxima and minima can also be used for maximizing tax revenue. The method used is
same as in case of optimizing other functions as total revenue function, total cost function etc. The only
difference is that the level of output sold is function of the rate of tax. So total tax revenue is equal to rate
of tax multiplied by units sold, is a function of rate of tax.
The unit sold is obtained considering the production tax imposed at the rate of Rs. t per unit. It will
be different in case of perfect competition and monopoly. In the case of perfect competition, output sold
is given by equilibrium price whereas in the second case, it is same as profit maximizing output given by
equality of marginal revenue and marginal cost.
Therefore in the case of perfect competition, the tax imposed is added in the supply function which
will shift the supply function upward and increase the equilibrium price to some output (depending on the
elasticity of demand) and reduces the equilibrium quantity as shown in figure 1.
Fig 1

So if P = f(x) (Demand function)


and P = g(x) (Supply Function before tax
then P = g(x) + t (supply function after tax)
 Equilibrium output is given by putting
f(x) = g(x) + t and solving for x. It will give x as a function of tax rate t.
Corresponding to this output.
Total tax revenue = t . xt.
So for maximizing (i) it take its derivative w.r.t. t, put it equal to zero and solve for t. (ii) the value
d 2TR  t 
of t for which < 0; tax revenue is maximum.
dt 2
In the case of monopoly, the firm adds it to the marginal cost function as shown in Figure 2.
Fig 2

The profit maximizing output is given by setting MR(x) = MC(x) + t and solving for x. At this profit
must be minimum. It is same as xt. Corresponding total tax Revenue: TR(t) = t . x which can be
maximized in the same way as explained above.
Example. An industry operating under competition has the demand function P = 10 – x with the
x
supply function P = 4 + . Determine (i) the equilibrium quantity before tax; (ii) the equilibrium
2
quantity after tax if tax at the rate of Rs. t per unit is imposed; (iii) the value of t which maximizes tax
revenue; (iv) maximized tax revenue.
Solution. (i) Given P = 10 – x (Demand function)
x
and P = 4+ (Supply function before tax)
2
 Equilibrium quantity will be
x
10 – x = 4 +
2
or x = 4 units
Corresponding equilibrium price will be, P = 10 – 4 = Rs. 6.
x
(ii) After tax supply function becomes P = 4 + + t. So new equilibrium after tax will be
2
x
10 – x = 4 + +t
2
2
or xt = 4 – t.
3
2
(iii) if xt = 4 – t, total tax revenue will be
3

Tax Revenue (t) = t  4  2 t 


 3 
For maximizing it, take its first derivative w.r.t. t.
d tax Revenue  t  4
 = 4– t
dt 3
Putting it equal to zero and solving for t.
4
4– t = 0
3
or t = Rs. 3 per unit.
At t = Rs. 3 per unit,
d 2 Tax Revenue  t  4
2
= – <0
dt 3
So tax revenue is maximum when t = Rs. 5 per unit.
(iv) Maximized tax revenue = t . xt

= 3  4  4 t 
 3 
= Rs. 6.
Example. The demand and total cots function of a monopolist are P = 12 – 4x and TC(x) = 8x + x2
respectively. It the tax of Rs. t per unit is imposed. Find (i) the quantity and price corresponding to the
maximum profit, (ii) maximized profit, and (iii) the tax that maximizes the tax revenue.
Solution. (i) Given P = 12 – 4x, so total revenue, TR(x) = 12x – 4x2
Total cost after tax, TC(x) = 8x + x2 + tx
 (x) = TR(x) – TC(x)
= 12x – 4x2 – 8x – x2 – tx
= x (4 – t) – 5x2.
For maximizing (x), take its first derivative w.r.t. x,
d x
 = (4 – t) – 10x
dx
Putting it equal to zero and solving for x
(4 – t) – 10x = 0
4t
or x = units
10
4  t d 2  x 
At x= , = –10 < 0.
10 dx 2
4t
So profit is maximum when x = ,
10

Corresponding P = 12 – 4
 4  t  = 52  25
10 10

 4  t  = 4  t 4  t   5  4  t 
2

(ii) Maximized profit,    10 


 10  10  
4  t 
2

=
20
4t
(iii) When x = ,
10

Tax revenue (t) = t . x = t  4  t  .


 10 
For maximizing it, take its derivative w.r.t. t
d tax revenue  t  4  2t
=
dt 10
Putting it equal to new and solving for t
4  2t
= 0 or t = Rs. 2.
10
At t = Rs. 2,
d 2 tax revenue  t  1
2
= t < 0.
dt 5
So tax revenue is maximum when t = Rs. 2 per unit.
6. Determination of Economic-order Quantities
Economic-order quantity (EOQ) is defined as that order size where overall cost of maintaining
inventory is minimized. It is the basic concept of inventory control models.
A simple EOQ model assuming deterministic demand and supply conditions, constant cost, linear
demand, instantaneous supply, no shortage involves the following costs.
(i) Inventory Cost: It means cost of purchasing inventory. In this simple case where demand is
known with certainty with constant per unit cost. Inventory cost = No. of units demanded x cost per unit.
Since it is constant in this situation, it does not effect EOQ. So whether it is considered for calculating
EOQ or not does not effect EOQ.
(ii) Ordering Cost: It includes all those cost which are incurred from the placement of order till
supply inventory reaches the premises. If order size with ordering cost is given; then total ordering cost is
equal to number of order multiplied by cost per order. Number of orders is given by demand per time unit
divided by order size.
So total ordering cost = No. of order x per order cost.
D
= (s)
x
where D is demand per time unit betting given; x is the order size and s is the cost per order. It has
opposite relationship with order size.
(iii) Holding Cost: It is also known as storage cost, it includes all those costs which are incurred for
maintaining inventory in stock such as godown rent, insurance charges, godown keeper salary etc. It is
equal to average inventory in stock multiplied by per unit holding cost. Average inventory in stock is
given by average of opening and closing stock.
x  x  0 x
 Average inventory  2  2 
So Holding cost = .H
2  
It has direct relationship with order size.
Therefore total inventory cost, if x is order size will be give by
TC(x) = Cost of inventory + Ordering cost + Holding cost
D x
= DC + .s+ .H
x 2
where D, C, s and H are constants.
The purpose is to determine the value of x which minimizes TC(x). The method is same as explained
earlier. Same value of x is known as EOQ.
Example. A company required 1,200 units of inventory in a year. The cost of inventory is Rs. 10
per unit. Cost of ordering is Rs. 300 irrespective of the size of order and holding cost is 20 per cent of the
average value of inventory in the business. Find the total cost function TC(x) when order are placed in x
units and determine the value of x which minimizes the overall cost.
Solution. Let x is the order size which minimizes the total cost; then cost of inventory = DC =
(1,200) (Rs. 10) = Rs. 12,000.
1200
Cost of ordering = No. of orders × per order cost = (300)
x
Cost of holding = Average inventory × per unit holding cost
= x 10  20  = Rs. x
2  100 
1200
So TC(x) = 12000 + (300) + x
x
For minimizing TC(x), take its first derivative w.r.t. x
dTC  x  3,60,000
= –+1
dx x2
3,60,000
Putting it equal to zero and solving for x – + 1 = 0, x = 600 units.
x2
d 2TC  x  7,20,000
At x = 600 units, = > 0.
dx 2
x3
 TC(x) is minimum when order size is 600 units.
7. Relationship between average product and marginal product
There exists an important economic relationship between average product and marginal product.
The relationship states if average product increases, marginal product increases at a higher rate and lies
above average product; if average product decreases, marginal product decreases at a higher rate and lies
below average product but marginal product equal average product is maximum. The same is shown in
the figure.
The relationship can be produced using maxima and minima. The important is to show that both are
equal when AP is maximum. If total product is given as univariate function (involving an input) then
TP  x  dTP  x 
corresponding AP(x) = and MP(x) = . Now maximizing AP(x) requires taking
x dx
derivative of AP(x) w.r.t. x, putting it equal to zero and solving x. At this level both average product and
marginal product must be equal.

Example. The total production function of a commodity is given by TP(x) = ax + bx2 + cx2 where
TP(x) is total output and x is input. Verify that when average product is maximum, it is equal to marginal
product. Are there any restrictions for the value of a, b and c.
Solution. Given TP(x) = ax + bx2 + cx2
TP  x 
AP(x) = = a + bx + cx2
x
dTP  x 
MP(x) = = a + 2bx + 3cx2
dx
For maximizing AP(x), take the first derivative w.r.t. x
dAP  x 
= b + 3cs
x
Putting is equal to zero and solving for x
b + 2cx = 0
b
x =
2c
Putting it equal to zero and solving for x
b + 2cx = 0
b d AP  x 
2
At x = , must be negative for maximum
2c dx 2
d 2 AP  x 
 = 2c if less than zero
dx 2
So c must be negative.
b
If c must be negative, then b must be positive for x = being positive.
2c
b2 b2c b2
At x = b , AP  b  = a –  2 a
2c  2c  2c 4c 4c
 b2  b2
and MP  b  = a + 2b  b  + 3c  2  = a –
 2c   2c   4c  4c
b
So both AP(x) and MP(x) are equal when x = (where AP(x) maximum).
2c
8. Other Applications
In addition to these specific applications based on maxima and minima, there are a number of
situations where these can be applied. Some of these are exemplified below.
Example. A wire of 50 cms in length is to be cut in two parts and then bent into two circles. Find
the length of each part so that the sum of the areas of two circles is least.
Solution. Let one part = x cms, then other part = 50 – x cms.
So circumference for first circle = 2r = x
x
or r =
2
x 2 x 2
Corresponding area of this circle = r2 = 
42 4
Similarly circumference of second circle = 2r = (50 – x) cms.
50  x
or r =
2
 50  x 
2

Corresponding Area of circle = r2 =   


 2 
 50  x 
2

=
4
 Total area of two circles given by adding (i) and (ii) will be

x 2  50  x 
2

A(x) = 
4 4
For minimizing, take its first derivative, w.r.t. x
dA  x  x  50  x 
So = 
dx 2 2
2 x  50 x  25
= 
 2
x  25
Putting it equal to zero and solving for x, =0
2
or x = 250 cms
d 2 A x
At x = 25 cm, = > 0,
dx 2
So area of two circle is least if each part is of 25 cms.
Example. The production manager of a publishing company plans to include 180 sq. cms. of actual
printed matter in each page of a book under production. Each page should have 2.5 cms margin along the
top and bottom and 2 cms wide margin along the sides. What are the most economical dimensions of each
printed pages.
Solution. Let length of the printed page = x cms
180
then breadth of printed page = cms. (as Area = length × breadth)
x
Corresponding length of pate = [x + 2(2.5)] cms = (x + 5) cms
(as margin both sides is 2.5 cms)
breadth of the page will be = 180  2  2   cms
 x 
180
= + 4 cms.
x

Corresponding Area of page, A(x) = (x + 5)  180  4 


 x 
For the most economical dimension, A(x) should be minimum.
 Put first derivative of A(x) w.r.t. x equal to zero and solve for x.
dA  x   180 
= (x + 5) – 180   4 = 0
dx x3  x 
180 900 180
=   2  +4=0
x x x
or x2 = 225
or x = 15 cms.
2
d A( x) 1800
At x = 15 cms = >0
dx 2
x3
So A(x) is minimum when length of page (5 + 5) = 20 cms,
and breadth of page =  1800  4  = 16 cms.
 15 
Example. An open box with length one and a half times the breadth is made form 1800 sq. ft. of
material. Determine the dimensions of the box so as to minimize its volume and minimized volume.
Solution. Given surface area of his open box = 1800 sq. ft.
3
Let breadth of box is x ft. then length = x ft with height h ft.
2
Then Area of bottom + Area of front and box + Area of two sides + surface area
3 2 3 
or x  2   h   2  h = 1800
2 2 
3 2
or x  5 xh = 1800
2
3 2
1800  x
h = 2 ft.
5x
Corresponding volume of the open box, V(x) = length . breadth . height
 3 2
3 1800  2 x 
= x . x . 
2  5x 
 
9 3
= 540x – x
20
For maximizing V(x), take its first derivative w.r.t. x
dV ( x ) 27
= 540  x 2
dx 20
Putting it equal to zero and solving for x
27 2
540  x = 0
20
or x = 20 ft.
d 2 N ( x) 27
At x = 20 ft. 2
= x < 0,
dx 10
So volume is maximum when breadth = 20 ft.
3
1800   20 
2

Corresponding length = 30 ft, height = 2 = 12 ft.


5  20 
 Maximized volume, V(20) = (30)(20)(12) = 7200 cubic ft.

LESSON 4

PARTIAL DIFFERENTIATION
Functions used in business and economics can also be classified as univariate, bivariate and
multivariate functions. Univariate functions are those functions which involves only one independent
variable as y = f(x) whereas bivariate functions involve two independent variables as x = f(x, y),
Multivariate functions involve more than two independent variables as u = f(x, y, z). Bivariate and
multivariate functions are considered more practical. Most of the functions used in business and
economics : as demand functions, production function, utility function, total cost function often appears
as bivariate or multivariate functions involving more than one independent variable. For example,
production is a function of different inputs, quantity demanded is a function of not only the price of the
commodity itself, but also of the prices of related commodities, and consumer's income.
In the case of univariate functions, the rate of change in the value of function is given by simple
differentiation as in the demand function x = f(P), the rate of change in demand in relation to price P is
given by
x dx
lim  (simplederivative).
x  0 p dp
But in case of bivariate and mukivanat; functions, the rate of change in the value of function in
relation to one independent variable keeping others as constant is given by partial differentiation. So if
demand function appears as X = F(P, Q) where P is the price of the commodity itcr1f oosad Q is the price
of related commodity, then rate of change in X in relation to P is given by
x
lim
x  0 p
x
and is denoted by (partial of X with respect to p) assuming Q as constant.
p
Definition : It implies that partial differentiation is related only with bivaiiate and multivariate
functions. It gives the rate of change in the value of function in relation to one independent variable at a
time keeping the effect of others as constant. For example if
Z = f(x, y),
then partial of z with respect to x will be
x z f ( x  x, y )  f ( x, y )
= lim  lim
x x  0 x x  0 x

and partial of z with respect to y will be


x x z f ( x, y  y)  f ( x, y )
=  lim  lim
y y y  0 y y  0 y
The method of differentiation is same as simple differentiation and the only difference that all other
variable whose effect is taken as constant are treated as constants for differentiation. So if z = f(x, y) is
differentiated with respect to x, y is taken as constant and differentiated as constant. Similarly if z is
differentiated with respect to y, x is taken as constant and differentiated as constant. All the standard
forms of differentiation are applicable. All these are exemplified below.
z z
Example. If z = 2x2 + 3xy – 6y2, find and .
x y
z
Solution : = 4x + 3y (–6y2 is a constant, so its derivative is
x
zero)
z
= 3x – 12y.
y
z z
Example. If z = xy + inx, find and .
x y
z 1
Solution : = y+
x x
z
= x.
y
x2  y 2 z z
Example. If z = , show that x and y = x.
xy x y
x2  y 2
Solution : Given z =
xy
= x2 y–1 —x–1 y2
z
So = 2xy–1 + x–2 y2
x
z
= —x2 y–2 —2x–1 y.
y
Taking L.H.S.
x[2xy–1 +x–2 y2] + y [—x2y–2—2x–1 y]
= 2x2 y–1 + x–1 y2 —x2y–1 —2x–1 y2
= x2y–1—x–1 y2 = z = R.H.S.

 x2  y 2  z z
Example. If z = In  2 2 
find and .
x  y  x y

x2  y 2   x  y  2 x   x  y  2x 
2 2 2 2
z
Solution : = 2  
x x  y2 
  x 2
 y 2
 

2 x 2  2 xy 2  2 x3  2 xy 2
=
 x2  y 2   x2  y 2 
4 xy 2
=
x4  y 4

x 2  y 2   x  y   2 y    x  y  2 y 
 2 2 2 2 
z 4 xy 2
= 2 
y x  y2 
  x 2
 y 
2 2  x4  y 4

x2 y 2 z z
Example. If u = , show that x +y =3u
x y x y
z  s  y  2 xy 2  x 2 y2
Solution : =
x  x  y
2

2 x 2  2 xy 3  x 2 y 2
=
 x  y
2

xy  xy  2 y 2 
=
 x  y
2

u  x  y  2x2 y   x2 y 2 
=
y  x  y
2

xy  2 x 2  xy 
=
 x  y
2

z z
Putting and on the L.H.S.
x y
x 2 y  xy  2 y 2  xy 2  2 x 2  xy 

 x  y  x  y
2 2

x2 y 2  2x2 y 2  2x2 y 2  x2 y 2
=
 x  y
2

3x 2 y 2  x  y 
=
 x  y
2

 x2 y 2 
= 3   = 3u = R.H.S.
x y
Example. A television manufacture’s profit function is
(x, y) = 500x+400y–2x2 – y3– 2xy
where x denotes the number of retail outset and y denotes the advertisement outlay (in thousands of
Rs.); and (x, y) is profit (also in thousands of Rs.). Show that rate of change in profit is equal at x = 50
and y = Rs. 1,00,000.
Solution : Rate of change in relation to x is given by
x  x, y 
= 500 – 4x – 2y
x
At x = 50 and y = 100 (in thousands)
  x, y 
= 500 – 4 (50) – 2(100)
x
= 100
Rate of change in relation to y is given by
  x, y 
= 400 – 2y – 2x.
y
At x = 50 and y = 100, rate of change is :
  x, y 
= 400 –2(50) –2 (100)
y
= 100
So rate of change is equal at x = 50 and y = Rs. 1,00,000.

APPLICATIONS OF PARTIAL DIFFERENTIATION


The partial differentiation giving rate of change in relation to an independent variable keeping others
as constant is widely used in the context of bivariate and multivariate function. If a joint function is given,
corresponding marginal function can be obtained with the partial differentiation. For example if a joint
total cost function TC (x, y) is given defining the total cost of producing of x units of A product and y
units of B product; then corresponding marginal cost functions can be obtained using partial
differentiation. Some of these are explained in the following section.
1. Measurement of cross-elasticity of demand : One of the most widely used application of partial
differentiation is the measurement of cross elasticity of demand. If a commodity's joint demand function
involving the price of the commodity and price of related commodity as x = f(P, Q) where P is the price
per unit of the commodity and Q is the price per unit of the related commodity, then cross elasticity of
demand of the commodity in relation to price of related commodity denoted by Ex2 is defined as
Relative change in quantity demanded of x
Ex2 =
Relative change in the price of related commodity
x
or x = x .Q
Q y x
Q
[It gives the cross elasticity for Q change in price.]
The same will give the cross elasticity of x at a price Q if limit of change in Q is tending towards
zero.
x Q
 EXQ = lim .
x  0 Q x
x Q x
= . [ lim is same as partial of x w.r.l. Q]
Q x Q  0 Q
So the measurement of cross elasticity of demand at a particular price involves par:
differentiation. Similarly if the joint demand function of the related commodity is y = g (P, Q), then
y P
EXQ = .
p Y
[for P change in price of related
commodity]

y P
= lim . [at a price P]
Q  0 p Y
y P
= .
p Y
Example. The joint demand functions of two related commodities A and B are as follows :
x = p–0.4 Q0.5 (A commodity)
y = p– Q 0.4 1.5 (B commodity)
where x and y denote their units and PQ prices per unit. Determine their cross elasticity of demand.
Solution : Cross elasticity of demand of A commodity is relation to price of B commodity will be
x Q
EXQ = .
Q x
0.5 p 0.4Q 0.5Q
=
x
= 0.5 (Putting x = P–0.4 Q0.5)
Similarly cross elasticity of B in relation to price of A will be :
y P
Eyp = .
p Y
0.4 p 0.4Q1.5 P
=
y
= 0.4 (Putting y = P0.4 Q–1.5)
Complimentary versus Substitutes : The same cross elasticities can also be used for dividing the
relationship between two commodities in economics, there are three different possibilities, viz.,
competitive products (which serves the same purpose as two brands of paste), complimentary products
(supplementing each other as bread and butler) and no relationship exist. If the commodities are
competitive or substitutes, one can be used in place of the other commodity resulting in quantity
demanded of one commodity changes in the same direction in which the price of other commodity
changes. For example, if the price of one brand of paste increases, some of the consumers of this brand
may switch over to related brand resulting in increase of the demand of related brand. So it implies that
there exist positive relationship between change in quantity demanded of one commodity and change in
price of related commodity in this case. Therefore, if both the cross elasticities are positive, the
commodities may be taken as substitutes or competitive as in the previous example.
But two commodities are complimentary if they are consumed simultaneously as bread and butter
and if the price of one increases, quantity demanded of other decreases and vice versa. It amounts that
there exists negative relationship between change in quantity demanded for one commodity in relation n
to price of related commodity, Hence if both the cross elasticities are negative, the commodities may be
taken as complimentary,
It may be possible that both the cross elasticities take opposite algebraic signs, in such cases nothing
can be concluded about the relationship between commodities.
Example. The joint demand functions of two related commodities A and B are as follows :
8
x = (A commodity)
PQ
8
y = (B commodity)
PQ
where x and y denote their units and P and Q their price per unit respectively.
Determine both the cross elasticity at P = Rs. 2 and Q = Re. 1. Also state whether commodities are
competitive or complimentary.
8
Solution : Given x =
PQ
x Q
Ex2 = .
Q x
8 Q
= .
PQ 2 x
8 8
=   1
 PQ  x 8
[Putting P = Rs. 2 Q = Re. 1 and x = 4 units]
Similarly
y P
Eyp = .
p y
12 P
= .
P 2Q y
12
= = –1
PQy
[Putting P = Q = and y = 12 units]
Since both the cross elasticities are negative, the commodities re complimentary.
Note 1. If two commodities joint demand functions are given as
x = f(P, Q) and y = g(P, Q),
then four different elasticities can be calculated. These are as follows :
x P
Exp = . [Same as direct price elasticity of derr
p x
which always takes negative sign
bect of law of demand]
x Q
EXQ = . [Cross elasticity of x in relation to Q]
Q x
y Q
EXQ = . [Cross elasticity of y in relation on P]
P x
y Q
EYQ = . [Direct elasticity of demand’
Q Y
2. If the joint demand function involves income of the consumer also, then income elasticity of
demand can also be calculated as exemplified below:
Example. The demand (x) of passenger automobile is given by
x = 0.90 I 1.1 p–0.7
where I is the income and P is the price per car. Find (i) income elasticity of demand (ii) price elasticity of
demand.
x 1
Solution : Income elasticity of demand = .
I x
991.1 P 0.7 1
=
x
= .99 [Simplification]
x P
Price elasticity of demand = .
I x
0.63I 1.1 P 0.7 .P
=
x
= -0.63 [Simplified]

3. Measurement of marginal productivity of input : The partial differentiation can also be used
for analysing the production function. It can be used for determining the rate of change in production in
relation to one input keeping others as constant (as is in case of short run production function). The same
rate of changes measures the marginal productivity of input. For example if p = f(x, y, x) where x, y and x
denotes three inputs.
P
Then marginal productivity of x inputs = = MPx
x
P
marginal productivity of y input = = MPy
y
P
marginal productivity of z input = = MPz
z
Example. The production function of a commodity is
P = 50x – x2 + 60y – 2y2
where x and y denotes the units of two unpit. Determine (i) the marginal productivity functions of x
and y, and (ii) their contribution at x = 10 units and y = 10 units.
Solution : Given P = 50x — x2 + 60y — 2y2
P
MPx = = 50 — 2x
x
P
MPy = = 60 — 4y.
y
At x = 10 Units, y = 10 units
MPx = 30
MPy = 20.
Example. The production function of a commodity is
Q = 10L — 0.1L2 +15k2 + 2kL
where L is labour, k is capital and Q is production. Determine (i) the marginal product of two inputs,
(ii) if 10 units of capital re used, what is the upper limit for the use of labour which a rational producer
will never exceed?
Solution : (i) Given Q = 10 L — 0.1L2 + 15k — 0.2k2 +2kL
Q
MPL = = 10 — 0.2L + 2k
L
Q
MPK = = 15 — 0.4k + 2L
k
(ii) If k = 10 units
MPL = 10 — 0.2L + 20
= 30 — 0.2L.
A rational producer will employ labour till MPL is positive, so putting MPL equal to and solving for
L.
30 – 0.2L = 0.
l = 150 units.
So he will employ upto 150 units of labour, After this MPL becomes negative which will reduce the
production.
Example. The production function of a firm is given by
Q = 4L3/4 k1/4
Find the marginal productivity of labour (L) and capital (k). Also show that
Q Q
L. = + K. =Q
L k
Soluton : Given Q = 4L3/4 k1/4
Q
MPL = 3L1/4 k1/4
L
Q
MPK = = L–3/4 k –3/4
k
Taking L.H.S.
L [3L–1/4k1/14] + k [k3/4 k–3/4]
= 3L–3/4k14 + k k3/4 k–1/4]
= 4 L3/4 k1/4
= Q = R.H.S.
The same can also be used for explaining the short term input-output relationship given by laws of
variable proportion also known as law of diminishing return. It states that if output is increased by
increasing the units of one input keeping others as constant, initially output increase at increasing
proportion, then may at constant proportion, but after certain level, it will increasing at decreasing rate.
So if rate of change in marginal product is positive, it is taken an increase return; if is taken as
increase return; it it is equal to zero, it is taken as constant return; and it is negative, it implies decreasing
return.
3. Determination of profit maximising output under duopoly : Partial differentiation can also be
applied for determining the profit maximising output under duopoly is defined as that market situation in
which there are only two producer of commodity. In this situation, the total industry’s demand is fulfilled
only by two suppliers. So problem is how to distribute the demand between two firms so that each can
maximise its profit.
Various models have been evolved out by economists for analysing the firms equilibrium in this
situation. One of these is evolved out by Cournot. According to this model :
(i) The price of the product is determined by total demand and total supply of industry. At that price,
both firms sell their product.
(ii) Each firm cna vary its output assuming the output of other firm remains unchanged.
On the basis of these two assumptions, the duopolies’ equilibrium can be determined in this way
using partial differentiation :
(a) Firstly, obtain the total revenue function of each firm from the industry’s demand function. Let P =
F(x) is the industry’s demand function sitting x units are demanded when market price is Rs. P per
unit. If it is shared by two firms as x1 and x2 such that x = x1 + x2. Then total revenue function of
firm 1 will be TR1 (X1, X2) = Px1 which will be a bi-variate function. Similarly total revenue
function of firm 2 will be TR2 (X1, X2) = Px2 For example, if the demand function of industry P = a
— bx where x is shared by two firms s x1 and x2,
then P = a — b (x1 + x2)
So total revenue function of firm 1,
TR1 (X1, X2) = Px1 = ax1 bx12 — bx1x2.
Similarly total revenue function of firm 2,
TR2 (X1, X2) = Px2 = ax2 bx1x2 — bx12.
So corresponding marginal revenue functions can be obtained using partial differentiation as to;
Marginal revenue function of firm 1,

MR1 =

TR x1  x2 
x1

Marginal revenue function of firm 2,


TR  x1 , x2 
MR2 =
x1
= a – bx1 – 2bx2.

(b) Secondly, find the total cost function of both the firms; i.e., for x1 units of firm 1 and for x2 units
firm 2. Both the firms operate under different conditions, so their cost function may vary, For
example total cost function of firm 1 is
TC(x) = 5x + 400,
then for x1 units it will be
TC1 (x1) = 5x1 + 400.
Similarly for firm 2, if it is
TC (x) = 2x2 + 500.
then for x2 units it will be
TC2 (x2) = 2x22 + 500.
So corresponding marginal cost functions can be obtained using simple differentiation only.
(c) Lastly, applying the basic conditions for profit maximisation, i.e., put their marginal revenue
functions equal to marginal cost functions. This will give two simultaneous equations in terms of x1
and x2 which can be solved for x1 and x2.
It the slope of their marginal cost functions is positive at these values of x1 and x2 then firm’s make
maximum profits at these levels.
The name profit maximising outputs can be used for determining price at that level and maximsed
profit of both the frims.
Example. An industry consisting of two firms has price function P = 100 – 0.5x. The total cost
function of firm 1 is TC(x) = 5x for x units and of firm 2 is TC(x) = 0.5x2 for x units. Determine
(i) the equilibrium of both the firms, (ii) equilibrium price of the industry and (iii) maximised profit of
both firms.

Solution : Given price function P = 100 – 0.5 x when x units are demanded. Assuming two firms
share it as x1 and x2. So putting x = x1 + x2.
P = 100 – 0.5 (x1 + x2)
Total revenue function of firm 1,
TR1, (x1 + x2) = Px1
= 100x1 – 0.5x12 – 0.5x1x2
So corresponding marginal revenue function,
TR  x1 , x2 
MR1 = = 100 – x1– 5x2.
x1
Similarly total revenue function of firm 2,
TR2, (x1, x2) = Px2 = 100x2 – 0.5x1x2 – 0.5x22
So corresponding
TR2  x1 , x2 
MR2 = = 100 – 05x1–x2
x2
Total cost function of firm 1 for x1 units will be
TC (x1) = 5x1
dTC ( x1 )
So MC1 (x1) = = 5.
dx1
and total cost function of firm 2 for x2 units will be
TC (x2) = 0.5x22
dTC ( x 2 )
So MC2 (x2) = = x2
dx 2
(i) Setting their marginal revenue functions equal to their marginal cost functions and solving for x1 and
x2 .
100 – x1 – 0.5x2 = 5 ...(1)
100 – 0.5x1 – x2 = x2 ...(2)
Solving (1) and (2) for x1 and x2, x1 = 80 units and x2 = 30 units. At both these marginal cost is increasing.
So both the firm’s are in equilibrium when x1 = 80 units and x2 = 30 units.

(ii) Price at this level :


P = 100 – 0.5 (80 + 30)
= Rs. 45.
(iii) Profit of each firm will be
Firm 1, Profit = TR (80) – TC (80)
= Rs. (3600 – 400) = Rs. 3,200.
Firm 2, Profit = TR (30) – TC (30)
= Rs. (1350 – 450) = Rs. 900.

TOTAL DIFFERENTIATION
In the previous section we have been using term derivative or differentiation. If differs from
differential. Differential denotes the change in the value of function y = f(x) denoted by dy in y
corresponding to certain change in x i.e., dx. It differs from derivative as derivative gives the change in y
with respect to change in x where change in x is tending towards zero. So derivative gives the rate of
change whereas differential gives the total change. Then
dy y
 = lim
dx x 0 x

 y 
dy =  lim  dx
x 0 x
 
 dy 
=   dx
 dx 
where dy = differential of y
dy
= rate of change in y
dx
dx = differential of x.
Actually it gives the approximate change in y for a certain change in x. So it is taken as
 dy 
dy    dx
 dx 
Therefore differential gives the change in the value of function for a certain small change in the
value of independent variable.
Example. Total revenue function of a firm is TR(x) = 5x. Find change in TR(x), dTR(x) if chnge in x,
i.e., dx = .04 units with the help of derivative.
Solution : Given TR(x) = 5x and dx = 4 units
dTR ( x)
= 5
dx
 dTR ( x) 
dTR (x) =   dx = 5dx
 dx 
= (5).04
= Rs. 0.20.
Example. A firm’s total cost of producing x units is given by TC(x) = x2 + 1000. Using differential,
find change in total cost if output increased from 5 units to 5.10 units. Compare it with actual change.
Solution : Given TC(x) = x2 +1000 and dx = 5 units
 dTR ( x) 
dTC (x) =   dx
 dx 
= [2x] dx
= [2x] 0.10 = .2x
Putting x = 5 units
dTC (x) = [.2] 5 = 1
Actual differential in TC(x) = TC (5.10) – TC (5)
= 1026.1 – 1025
= 1.1 app.
The difference can not be avoided.
This will be in case of function involves only one independent variable. If function considers more
than one variable as z = f(x,y), then dz is known as total differential involving the differential of x and y
only. It is the total differential of x when y is constant and differential of y when x is constant.
z z
dx + dy
x x
Therefore
z z
where and are the partial derivatives of z with respect to x and y respectively. Both these
x y
denotes the rate of change when other variable is constant.
If change in any of the two variables is zero, the differential corresponding to the variable becomes
z z
zero. For example if dy is zero, dx become zero and dz = dy. These two are also known as partial
x x
differentials. But these are also valid for small changes in x and y.
Example. Given z = x4 + 8xy + 3y3. Find its total differential dz.
Solution : Given z = x4 + 8xy + 3y3
z z
= 4x3 + 8y and = 8x + 9y2
x y
z z
then dz = . dx + .dy
x y
= (4x3 + 8y) dx + (8x + 9y2) dy.
Example. Given the total production function
Q = ALak1–a
where Q is total production when L units of labour and k units of capital are used. Find total
differential of Q.
Solution : Given Q = ALak1–a
Q Aal a k 1 a Q
= AaLak1–a = 
L L L
= Ala (1 – a)k–a
1  a  ALa k 1 a  1  a  Q
=
k k
Q Q
then dQ = dL + dk
L k
aq
dl 
1  a  Q dk .
=
L k
Example. Given the production function.
Q = (bk–r + aL–r)–1/r, find dQ.
Solution : Given Q = (bk–r + aL–r)–1/r,
Q
 = [bk–r + aL–r]–1/r–1 –rbk–r–1
k
= bk–r–1 [bk–r +aL–r]–1/r–1
Q
 = [bk–r + aL–r]–1/r–1 –rbL–r–1
L
= aL–r–1 [bk–r +aL–r]–1/r–1
 Q   Q 
then dq =   dk +   dL
 k   L 
= bk–r–1 [bk–r + aL–r]–1/r–1. dk + aL–r–1
[bk–r + aL–r]–1/r–1. dL.

DERIVATIVE OF IMPLICIT FUNCTIONS WITH THE HELP OF PARTIAL DIFFERENTIATION


The implicit function can also be differentiated with the help of partial differentiation. f(x, y) = 0 is
dy
an implicit function as 2x2 +3xy + y2 = 0, then its derivative can can be obtained in this way :
dx
(i) Take differential of both sides :
f  x, y  f  x, y 
 . dx  dy  0
x x
(ii) Divide both sides by dx resulting into
f  x, y  dx f  x, y  dy
.  . 0
x dx x dx
dy
(iii) Simplify it for , so
dx
dy f  x, y  f  x, y 
=
dx x x
Therefore derivative of an implicit function is equal to minus of the ratio of partial derivatives of
f(x, y) with respect of x and y respectively.
Example. Show that derivative of the following implicit function is equal to minus of the ratio of
partial derivatives :
(i) x2 + y2 + 2xy = 0 (ii) x3 + y3–3axy = 0
Solution : Let f(x, y) = x2 + y2 + 2xy = 0
Taking its total differential of both sides.
f  x, y  f  x, y 
dx  dy = 0
x x
f  x 2 , y 2  2 xy  f  x 2 , y 2  2 xy 
or dx  dy = 0
x x
or (2x + 2y) dx + (2y + 2x) dy = 0

dy
Divide both sides by dx and solve for
dx
dy
(2x + 2y) + (2y + 2x) = 0
dx
dy   2x  2 y 
or = = –1
dx  2x  2 y 
(ii) Given x3 + y3= –3axy = 0
f  x 2 , y 2  2 xy  f  x 3 , y 2  2 xy 
So dx  dy = 0
x x
(3x2 – 3ay) dx + (3y2 – 3ax) dy = 0
Dividing both sides by dx
dy
(3x2 – 3ay) dx + (3y2 – 3 ) = 0
dx
dy   3x 2  3ax  ay  x 2
= 
dx  3 y 2  3ax  y 2  ax
EULAR’S THEOREM
Functions are classified in different categories on different basis. As on the basis of number of
independent variables, functions are classified as univariate, bivariate and multivariate functions.
Univariate functions have only one independent variable. Bivariate functions are known as those
functions which have two independent variables as z = f(x, y). Similarly multivariate functions are those
which have more than two independent variables as
u = f(x, y, x)
Bivariate and multivariate functions are further classified as homogeneous and non-homogeneous
functions. The basic property of homogeneous functions is that if z = f(x,y), then any constant ,
f(x, y)=x f(x, y). For example, if z = x2+ 2xy + y2, then a2z = a2x2 + 2a2xy + a2y. These are known as
homogeneous functions of nth degree and 2nd degree respectively. If n = 1, the homogeneous function is
known as linear homogeneous function. These functions are characterised as those where sum of the
power of independent variables of all the nomials (terms) is equal. The sum of such powers is known as
degree of homogeneous function. For example z = x2 + 2xy + y2 is a homogeneous function of second
degree.
A rational function appearing in the form of division of two functions is known a homogeneous
function if both numerator and denominator are homogeneous functions with the degree as difference
between the degree of numerator and denominator. If both the numerator and denominator. are
homogeneous functions of equal degree, the function is known as zero degree, homogeneous function.
Eular's Theorem is relating to homogeneous functions only. According to Eular's theorem if
z = f(x,y) of nth degree homogeneous function and first order partials exist, then
z z
(i) x.  y. = nz
x y
It has a number of corollaries, one of the important one is that :
2
  
(ii)  x. x  y. y  z = n(n–1) z
 
2 z 2 z 2  z
2
or x2 .  2 xy.  y . = n (n – 1) z.
x 2 y y y 2
[Proof has been ignored]
Example. For the production function z = axa y, show that
z z
(i) x.  y. = ( + ) z
x y
2 z 2 z 2 z
(ii) x2 .  2 xy.  y 2 . 2 = ( +฀) (a+b–1) z
x 2
y y y
Solution : Given z = axa y3 is a homogeneous function of ( + ) degree, so according to Eular’s
theorem:
z z
(i) x.  y. = ( + ) z
x y
Taking L.H.S.
  axa y    ax a y 
x.   y.
x y
or x. xaa–1 y + yax y฀–1
or axa y + ax y
or   axa y (+ )
or (+ ) z = R.H.S.

(ii) Applying corollary of Eular’s theorem


2 z 2 z 2  z
2
x2. x.  2 xy.  y . = ( + ) ( +  + 1) z.
x 2 x y y 2
Taking L.H.S.
  z     z 
x2  2 xy  y 2 .
 
dx  x  y y  y 
  
Or x2 axa  ay   2 xy  axa 1 y   y 2 axa y1 
dx y y
or x2 [(a – 1)xa-2 y] + 2xy [aaxa-1 y-1] + y2 [–1) axa y-2]
or a (a–1)axa y + 2aaxa y + (-1)axa y
or axa y [ ( – 1) + 2 +  (– 1)]
or z [( + ) ( +  – 1)] = R.H.S.
Example. Given z = (ax + by)–1, show that
z z
(i) x.  y. = – (ax + by)–1
x y
2 z 2 z 2  z
2
(ii) x2 .  2 xy.  y . = 2 (ax + by)–1
x 2 x y y 2
Solution : Given z = (ax + by)–1 is a homogeneous function of minus one degree. So applying
Eular’s Theorem.
z z
(i) x.  y. = –(ax + by)–1
x y
Taking L.H.S.
  ax  by     ax  by  
1 1

x.    y.  
x x
or x. [–(ax + by)–2 .a] + y [–(ax + by)–2. b]
or –(ax + by)–2 [ax + by]
or –(ax + by)–1 = R.H.S.
(ii) Applying a corollary of Eular’s Theorem :
2 z 2 z 2 z
x2 .  2 xy.  y 2 . 2 = 2 (ax + by)–1
x 2
x y y
Taking L.H.S.
  z    z    z 
x2  2 xy    y2.
 
dx  x  x  y  y  y 
 
or x2 (ax  by)  2  2 xy b(x  by)2 
x y

 y2  b(ax  by )2 
z 
or x2 [2a (ax + by)–3 a] +2xy [2b (ax + by)–2. a]
+ y2 [2b (ax + by)–3. b]
or (ax + by)–3 [2a2x3+ 4abxy + 2b1 y2]
or 2(ax + by)–3 [a2x2+ 2abxy + b2 y2]
or 2(ax + by)–3 [(ax + by)2)]
or 2(ax + by)–1 = R.H.S.
MAXIMA AND MINIMA OF BIVARIATE FUNCTIONS
Relatively maximum and minimum values of a bi-variate function can be obtained in the same way
as in the case of univariate functions. The only difference is that it involves partial differentiation whereas
unlvaiiate functions involve simple differentiation. The meaning of relatively maximum and minimum is
same.
In the context of bi-variate functions, relatively maximum value of a function is that value which is
maximum in relation to the surrounding values of function for different values of variables. Similarly
relatively minimum value is that value which is minimum in relation to surrounding values of the
function for different values of variables. Technically, let z = f(x, y), it will have relatively maximum
value at (a, b) if f(a, b)  f(x, y) for all values of (x, y) close to (a, b). It will have relatively minimum at
(a, b) if f(a, b)  f(x, y) for all values of (x, y) close to (a, b).
The determination of these requires the second order partial derivatives and cross partial derivatives
of the function. Second order partial derivatives of the function is the partial derivatives of the partial
z z
derivatives of a bivariate function. Let z = f(x, y), then and are the first order partials of z with
x y
z
respect to x and y respectively. The partial derivative or with respect to x, i.e.,
x
  z  a2 x
or
x  x  x 2
z
is the second order partial of z with respect to x. Similarly partial derivative of with respect to y,
y
i.e.,
  z  a2 z
 
y  y 
or 2
y
is the second-order partial of z with respect to y.
In case z = f(x, y) is first differentiated with x keeping y as constant and resultant partial derivative is
differentiated with respect to y keeping x as constant is known as cross partial of z with respect to x and y.
  z  a2 z
Technically or is the cross partial derivative.
x  x  x y
Similarly if z is first differentiated with respect to y keeping x as constant and then the resultant
partial derivative is differentiated with respect to x keeping y as constant, also gives cross-partial
  z  a2 z
derivative of z. Technically or is the cross partial derivative. It may be noted that for
x  x  x y
a2 z
z = f(x,y),
x y
a2 z
is always equal to
x y
After obtaining all these partial derivatives of the function, the maxima and minima can be obtained
by the following steps :
z z
z = f(x, y), i.e., and
x y
equal to zero. It gives two simultaneous equations whose solution gives some values of x and y
which are also known as critical values. The function may have relatively maximum or minimum or point
of inflection or saddle point at these critical values of (x and y).
(ii) (a) Function will take relatively maximum at critical values x and y if both second order partials
are negative and their product exceeds the equare of cross derivative. Technically if
a2 z a2 z
 0 and  0 with
x 2 y 2
2
a2 z a2 z  a2 z 
.  
x2 y 2  x ay 

Is also taken as if one of the second-order partial is negative with their product exceeding the square
of cross derivatives. It implies that both the second order partials must be negative because only in that
case their product will exceed their square.
(b) Function will take relatively minimum value at (x, y) if both the second order partials are
positive and their product exceeds the square of cross partial derivatives, Technically,
a2 z a2 z
 and  0 with
x 2 y 2
2
a2 z a2 z  a2 z 
.  
x2 y 2  x y 
It is also put as one of the second-order partials in positive and product of both second-order
partials exceeds the square of cross partial derivatives. It will be possible only if both the second
order partials are positive.
(c) Function will be at the point of inflection, means neither increasing nor decreasing if both the
second order partiais are either positive or negative but their product is less than square of the
cross derivatives. Technically, if
a2 z a2 z
 0 and 0
x 2 y 2
a2 z a2 z
or  0 and  0 with
x 2 y 2
a2 z a2 z  a2 z 
.  
x 2 y 2  x y 
(d) Function will be at saddle point if second order partials take opposite signs with their product is
less than the square of cross-partial derivatives. Saddle point means the point at which function
has maximum value when viewed from one axis and mimimum when viewed from one axis and
mimimum when viewed along another axis. Technically,
a2 z a2 z
 0 and 0
x 2 y 2
a2 z a2 z
or  0 and  0 with
x 2 y 2
a2 z a2 z  a2 z 
.  
x 2 y 2  x y 
(e) No conclusion can be arrived at if
2
a2 z a2 z  a2 z 
. =  
x 2 y 2  x y 
For proof, see any text book on calculas.
Exampale. Given z = 8x – 2x2 + 12xy – 12y2 + 6y. Determine
(a) the critical values of x and y
(b) whether function is at minimum, maximum, point of inflection or saddle point, and
(c) calculate it at critical values.
Solution : Given 8x – 2x2 + 12xy – 12xy + 6y.
(a) it will have extreme value when its first order partials are equal to zero.
z
= 8–4x + 12y = 0 ...(i)
x
z
= 12x – 24y + 6 = 0 ...(ii)
y
By solving (i) and (ii) equations simultaneously for x and y, we get x = –5.5 and y = –2.5. So critical
values are x = –5.5 and y = –2.5.
(b) At these critical values, second order partials are :
a 2 z   z  
   = [8–4x + 12y] = –4
x 2
y  x  x
a 2 z   z  
   = [12x – 24y + 6] = –24
y 2
y  y  y
and cross partial will be
a 2 z   z  
   = [8 – 4x + 12y] = 12.
y 2
y  y  y
Both the second order partials are negative and their product is less than the square of cross partial,
i.e., (–4) (–24) < (12)2
It implies function z has point of inflection at the critical values.
(c) when x = –5.5 and y = –2.5
z = 5(2)2 –30(2)+4(2) (2.5) –3 (2.5)2 + 7(2.5)
= –21.25.

APPLICATION OF MAXIMA AND MINIMA OF BIVARIATE FUNCTIONS


The functions used in business and economics often appears as bivariate functions as total revenue is
a function of units of A and B commodity denoted by TR (x, y). It is also known as joint total revenue
function. Similarly joint total cost functions, TC(x, y) and joint profit fucntion  (x, y) likewise. Therefore,
maxima and minima of bi-vriate functions have important applications in economics and other business
situations. It is used for optimising the joint functions both in terms of maximisation and minimisation.
Some of these applications are as follow.
1. Determination of'firm's equilibrium (producing two products) : Monopolist and 01 producers
often produce more than one product at a time which are produced together but sold at differs prices. In
such situations, the problem is to determine the output of each product in such a way the overall profit is
maximised. It can be obtained by applying maxima and minima of bi-variate functions.'
In such situation, let P = f(x) is the demand function of .4 commodity and Q = g(y) is the demand
function of B commodity where P and Q denote their prices per unit (in Rs.) and x and y their units
respectively. TC (x, y) is the joint cost function of A and B commodities.
Then total revenue function of A commodity will be
TR (x) = x f(x)
and of B commodity
TR(y) = y g(x).
Therefore, profit function
 (x, y) = x2 + 2xy + 3y2.
 Profit function,
 (x, y) = TR(x) + TR(y) –TC (x, y).
= 36x – 3x2 + 40y – 5y2–x2–2xy–3y2
= –4x2 + 36x + 40y – 8y2 – 2xy
(i) (x, y) has extreme value when its first-order direct partials are equal to zero
  x, y  
=  4 x 2  36 x  40 y  8 y 2  2 xy 
x x 
= –8x + 36 – 2y = 0 ...(i)
  x, y  
= 4 x2  36 x  40 y  8 y 2  2 xy 
y y 
= –16y – 2x + 40 = 0 ...(ii)
Solving (i) and (ii) equations simultaneously, x = 4 units and y = 2 units.
Therefore (x, y) takes extreme value when x = 4 units and y = 2 units.
(x, y) is maximum when x = 4 y = 2 uinits and y = 2 units as second-order direct partials are
negative and their product exceeds the square of cross-partial as shown below:
 2   x, y      x, y  
   = [–8x + 36 – 2y] = –8
2 y  y 

 2   x, y      x, y  
   = [–16y + 2x + 40] = –16
y 2
y  y 

 2   x, y      x, y  
and    = [–16y – 2x + 40] = –2
y y x  y 

(ii) Putting x = 4 units and y = 2 units in corresponding


demand functions:
P = 36–3x Q = 40–5y
= 36–12 = 40–10
= Rs. 24. = Rs. 30.
(iii) Maximized profit will be :
(4, 2) = –4(4)2 – 8(2)2 – 2(4)(2) + 36(4) + 40(2)
= Rs. 112.
Note. Another form of same optimisation of bi-variate function is the profit-maximisation in case
the production function involving two inputs is given together with the cost per unit of inputs
and price per unit of output. The method is the same as in the case of equilibrium of firm
producting two products.
In this situation, the profit function can be obtained in this way. Let z =0 (x, y) is a production
function stating that z units of output can be produced from x units of input 1 and y units of input 2. the
price per unit of output is Rs. p and cost per unit of input 1 is Rs. C1 and of input 2 is Rs. C2.
Then total revenue of z units is
TR(z) = P z
and total cost of two inputs,
TC(x, y) = c1x + c2y.
This follows that profit function,
(x, y) = TR(z) – TC(x, y)
= Pz – TC(x, y)
= P.  (x, y) – TC (x, y)
= P.  (x, y) – C1x –c2y.
Its miximisation according to the method stated above gives the values of inputs x and y which
maximizes profit. The same can be used for determining production at that level by putting these values
of x and y in production function. It can also be used for finding maximised profit.
Example. A firm’s production function stating output and input relationship is given by
z = 20 – x2 + 10x – 2y2 + 5y.
If the price per unit of output is Rs. 5 with cost of input x as Rs. 2 per unit and if input y as Re. 1 per
unit, determine (i) the units of two inputs which maximizes profit, (ii) production at that level, and (iii)
maximized profit.
Solution : Given production function.
z = 20 – x2 + 10x – 2y2 + 5y.
with P = Rs. 5 per unit,
C1 = Rs. 2 per unit and
C2 = Re. 1 per unit.
 Total revenue function,
TR(z) = Pz
= 5 [20 – x2 + 10x – 2y2 + 5y]
or TR(x, y) = 100 – 5x2 + 50x – 10y2 + 25y.
and TC(x, y) = C1x + C2y
= 2x + y.
 Profit function ฀(x, y) = TR(x, y) – TC(x, y)
= 100 – 5x2 + 50x – 10y2 + 25y – 2x – y
= 100–5x2+48x–10y2+24y.
(i) it will have extreme values when its first-order direct partials is equal to zero :
  x, y  
= [100 – 5x2 + 48x – 10y2 + 24y]
x x
= –10x+48=0 ...(i)
  x, y  
= [100 – 5x2 + 48x – 10y2 + 24y]
y y
= –20y + 24y = 0 ...(ii)
Solving (i) and (ii) x = 4.8 units and y = 1.2 units. When x = 4.8 units and y = 1.2 units, both the
second order-partials are negative and their product is greater than square of cross partial as shown below
:
 2   x, y      x, y   
=   = [–10x + 48] = –10 < 0
y y  y  x

 2   x, y      x, y   
=   = [–20y + 24] = –20 < 0
y 2
x  y  y

 2   x, y      x, y   
=   = [–20y + 24] = 0
x y x  y  x
 –(10) (–20) > 0
So  x, y takes maximum value when x = 4.8 units and y = 1.2 units
(ii) Production at this level is given by putting x = 4.8 units and y = 1.2 units in production
function z.
z = 20 – x2 + 10x + 20y2 + 5y
= 20–(4.8)2 + 10(4.8) – 20(1.2)2 + 5(1.2)
= 43.76 units.
(iii) Maximised profit will be
(4.8, 1.2) = 100–5(4.8)2 + 48(4.8)–10(1.2)2 + 24(1.2)
= Rs. 229.60.
2. Equilibrium of Monopolist under Price-discrimination : Maxima and minima bi-variate
function can be applied for determining the profit-maximising output of monopolist discriminating prices
in different markets. Price-discrimination means charging different prices for the same product in
different markets. There are three different degrees of price discrimination and it is possible only under
certain conditions as resale is not possible, demand different markets [For details, see some text-book on
economies]. This results into that demand functions for the same product will differ in different markets
sltough cost function is the same.
If all the conditions of price discrimination are fulfilled, the problem is to divide the output among
different markets in such a way that the profit is maximized.
It can be obtained with the help of maxima and minima of bi-variate functions in this way. Let
P1= f(x1) and P2 = f(x2) are the two demand functions in market 1 and market 2. Here x1 and x2 denotes
the units demanded when market price is Rs. P1 and Rs. P2 Per unit in respective markets. Total cost
function is TC(x) for x units where
x = x1 + x2 or TC(x1 + x2).
Therefore total revenue function of market 1 is
TR(x1) = P1 x1
and of market 2 is
TR(x2) = P2 x2
with total cost function as TC(x1 + x2). This follows that profit function ฀(x1, x2) will be :
(x1, x2) = TR(x1) + TR(x2)–TC(x1 + x2).
If becomes a bi-variate function whose maximisation as per the procedure gives the values of x1 and
x2 which maximizes the profit. The same divides the monopolist’s output in market 1 and 2 as x1 and x2.
Example. A monoplist sells his product in two different markets at different prices P1 and P2 where
demand functions are :
market 1 x1 = 16–0.2P1
market 2 x2 = 9–0.05P2
Determine the firms profit maximizing output, prices and maximized profit if firm’s total cost
function is
TC(x) = 20 + 20x
where x = x1 + x2.
Solution : Given demand function of market 1 as
x1 = 16–0.2P1
16 x1
 P1 = 
0.2 0.2
= 80–5x1
or TR(x1) = P1 x1= 80x1 – 5x12
and TR(x1+ x2) = 20 + 20 (x1 + x2)

Therefore, profit function  (x1, x2) will be

 (x1, x2) = TR(x1) + TR(x2) – TC(x1 + x2)


= 80x1–5x12+180x2–20x22–20–20x1–20x2
For maximizing t is. take its first-order direct partials, but these equal to zero and solve for x1 and
x2 .

  x1 , y 2 
So = 80 – 10x1+20 = 0
x1
  x1 , x 2 
= 180 – 40x2 – 20 = 0
x2
Solving (i) and (ii), x1 = 10 units, x2 = 4 units. At x2 = 10 units and x2 = 4 units.

 2   x1 , x2  
= [80 – 10x1 + 20] = –10 < 0
x 2
1
x1

 2   x1 , x2  
= [160 – 40x1 ] = – 40 < 0
x 2
2
x 2

 2   x1 , x2  
and = [160 – 40x1 ] = 0
x1 x2 x1
(–10) (–40) > 0
400 > 0.
Since both the second-order partials are negative and their product exceeds the square of cross
partial, profit function, ฀(x1, x2) take maximum value when x1 = 10 units and x2 = 4 units.
Price corresponding to this are given by putting x1 = 10 units and x2 = 4 units in corresponding
demand functions.
 P1 = 80 – 5x1
= 80–50 = Rs. 30.
P2 = 180–20x2
= 180–80 = Rs.100.
Maximized Profit,
(10, 4) = TR(10) + TR(4)–TC(14)
= 300 + 400 – 300
= Rs. 400.
3. Minimization of joint cost function : Optimisation of bi-variate function can also be applied
directly for minimisation of joint cost function. Let TC(x, y) is the joint cost function of x units of A
commodity any y units of B commodity. Its minimisation involves determining the units of A and B
commodity (i.e., values of x and y) in such a way that overall cost is minimiscd. The steps involved are
same as stated earlier.
Example. The joint cost function of a firm producing two products is
TC = 6x3 – 9x – 3xy – 7y – 5y2 + 20
where x and y denotes their units. Find (i) the valuce of x and y which minimises TC(x, y) and (ii)
minimized TC(x,y).
Solution : Given
TC(x, y) = 6x2 – 9x – 3xy –7y– 5y2 + 20
(i) It will take extreme value when its first-order direct partials are equal to zero :
tc  x, y  
= [6x2 – 9x – 3xy –7y– 5y3 + 20]
x x
= 12x–9–3y = 0 ...(i)
tc  x, y  
= [6x3 – 9x – 3xy – 7y – 5y2 + 20]
y y
= –3x–7 + 10y = 0 ...(ii)

Solving (i) and (ii) simultaneously x = 1 unit, y = 1 unit, y = 1 unit. At this level, TC(x, y) is
minimum as both the second order partials are positive and their exceeds the square of cross partial as
shown below :
 2 tc  x, y  
= [12x – 9 – 3y] = 12 > 0
x 2
x
 2 tc  x, y  
= [–3x – 7 – 10y] = 10 > 0
y 2
y
 2 tc  x, y  
= [–3x – 7 + 10y] = –3
x y x
(12) (10) > (–3)2
(ii) Therefore minimised TC(x, y) will be
TC(1, 1) = 6 (1)2–3 (1) (1) – 7(1) + (1)2 + 20
= Rs. 12
Example. A closed rectangular box of 16 cubic feet volume is to be made of three kinds of
materials. The cost of the top and bottom material of Rs. 0.09 per sq. ft; cost of material for front and
back side is Rs. 0.08 per sq. and cost of materials for other two side is Rs. 0.06 per sq. ft. Find the
dimensions of box to minimize cost of material and minimised cost.
Solution : Let tength = x ft, breadth = y ft.
volume
ther height = = ft.
length.breadth
Cost of the top and bottom material is Rs. 0.09 per unit
 Total cost of to and bottom material
= (Area of front + Area of bottom) × Rate
= 2xy (0.09) = Rs. 018xy.
Cost of the front and back side material is Rs. 0.08
 Total cost of sides
= (Area of front + area of back) x Rate
 16 
=  2. .x  (0.08)
 xy 
256
= Rs.
y
Cost of the other two sides material is Rs. 0.06 per sq. ft.
 Total cost of sides
= (Area of two sides) x Rate
 16 
=  2. . y  (0.06)
 xy 
1.92
= Rs.
x
So total function of rectangular box will be :
 2.56 1.92 
TC(x, y) = Rs. 0.18 xy  
 y x 
It will have extreme value when its first-order partials are equal to zero
tc  x, y    2.56 1.92 
 = 0.18 xy  y  x 
x x  
1.92
= 0.18y – =0
x2
tc  x, y    2.56 1.92 
= 0.18 xy  y  x 2 
y y  
2.56
= 0.18x – =0
y2
Solving (i) and (ii) simultaneously
0.18x2y – 1.92 = 0 ...(iii)
0.18xy2 – 2.56 = 0 ...(iv)
0.18 x 2 y 1.92
or 2
=
0.18 xy 2.56
x 3
or =
y 4
3y
or x =
4
3y 8
Putting x = in (i) equation y = ft. (breadth), x = 2 ft. (length)
4 3
16
So height = = 3 ft.
8
.2
3
When length = 2 ft., breadth = 8/3 ft. and height = 3 ft. Total cost is minimum as the second partials
are positive and their product exceeds the square of cross partial as shown below :
 2TC  x, y    1.92  3.84
=  0.18 y  2   3  0
x 2
x  x  x
 2TC  x, y    2.56  5.12
=  0.18 x  2   3  0
y 2
y  y  y
 2TC  x, y    2.56 
=  0.18 x  2  = 0.18
x y x  y 
 3.84   5.12  2
 8   (8/ 3)2  > (0.18)
  
Therefore rectangular box has minimum cost when length = 2 ft. breadth = 8/3 ft. and height =3ft.
(ii) Minimized total cost :
 8   8  2.56 1.92 
TC  2,  = Rs. 0.18(20)    
 3   3  8/ 3 2 
= Rs. 2.88.
LESSON 5

MATRICES AND DETERMINANTS

Matrix algebra is an important tool of decision making in the problems of business and economics
under the assumptions of linearity.
Matrix is a set of elements arranged in the form of a rectangular array, such as
 a11 a 12 a 13 
A = a 21 a 22 a 23 
 a 31 a 32 a 33 
Each matrix is denoted by a single capital letter and by brackets like [ ] or ( ) as written above. Each
number of the matrix is called an element. Elements written horizontally form a row and vertically written
elements form a column. Each element has two subscripts, first one indicates its row and the second one
indicates the column of the matrix. In the above example element a11 means element of the Ist row and
first column; similarly a23 indicates the element of the second row and the third column. Rows are always
read first and columns afterwards. The number of rows and the number of columns specify the dimension
of a matrix. The above matrix is of 3 rows and 3 columns. Therefore, its dimension is 3×3 (read 3 by 3).
The dimension of a matrix is written as a subscript to the matrix, such as
 1 4 3
A =  2 1 8
0 9 5 33

p l q 
B =  
 c t r  23
 a11 a12 a13 ..........a1n 
a a 22 a 2n ..........a 2n 
C =  21
 ... ... ................ 
 
a in1 a m2 a m3 ..........a mn  m×n
Total number of elements in a matrix are equal to the product of its rows and columns. For example,
matrix A above is of 3×3 order, there, the total number of elements in the above matrix will be 9 (ie, 3×3).
Similarly total number of elements in matrix C will be mn.
The following are some important facts about matrices:
(a) If a matrix has only one row it is called a row matrix or row vector.
x = [3 2 7]
(b) If a martix has only one column it is called as a column matrix or column vector.
5 
y = 1 
3
(c) Two matrices are equal, if and only if, their corresponding elements are equal and their
dimensions are the same. For example.
 a11 a12 
A =  
 a 21 a 22  22
a11 = b11
a21 = b21

 b11 b12 
B = 
 b 21 b 22  22

A = A if
a12 = b12
a22 = b22

(d) If all the elements of a matrix are zero it is called a Null matrix (or Zero matrix). For example,
0 0 0
O =  
0 0 0 23

TYPES OF MATRICES
(1) Square Matrix
If the number of rows a matrix are equal to the number of its columns ie., if m = n, it is called a
square matrix. For example.
3 7
A =  
 4 2  22
1 3 4 
B =  5 7 6 
 4 0 2  33

Matrix A is a square matrix as there are only 2 rows and 2 columns. It is called a square matrix of
2×2. Similarly B is a square matrix of 3×3.
(2) Diagonat Marix
A squarc martix is called a diagonal matrix when all its elements except the principal diagonal
elements are zero. For example:
a11 0 0
A =  0 a 22 0 
 0 0 a 33  33
A is a diagonal matrix
(3) Identify Matrix of Unit Matrix
A square matrix whose principal diagonal are 1 and all elements above and below the principal
diagonal are zero is known as an Identity matrix. For example:
1 0 0 
I = 0 1 0 .
0 0 1  33
It is an Identity matrix of 3×3.
(4) Scalar Matrix:
If the diagonal elements of a matrix are equal it is called a Scalar matrix. For example
3 0 0
A =  0 3 0 
 0 0 3 

 x 0 0 0
0 x 0 0
X =  
0 0 x 0
 
 0 0 0 x 
Matrix A and X are Scalar matrices.

ALGEBRA OF MATRICES
(1) Addition of Matrices:
Two matrices Amxn = aij and Bmxn = bij can be added if their dimensions are same. If the addition
matrix is called matrix C, then
Cmxn = Amxn + Bmxn

Dimensions of matrix C will be similar to the dimensions of matrices A and B. Matrix C can be
obtained by adding corresponding elements of matrix A with the elements of matrix A with the elements
of matrix A with the elements of matrix B. For example :
5 4 0 
A =  7 1 2 
 1 3 6  33

1 3 2 
B =  2 4 6 
 5 1 0  33

 5  1 4  3 0  2
Then C = 7  2 1  4 2  6 
 1  5 3  1 6  0 

6 7 2
or C = 9 5 8 
6 4 6  33
 Cij = aij + bij

If the dimensions of two matrices are different, they cannot the added. For example
0 2
A =  
7 6  22
5 1 2
B =  
7 3 6  23
A2×2 and B2×3 cannot be added.
(a) Mtrix addition is commutative, i.e.,
A + B = B + A [Provided their dimensions are same]
(b) If A. B. C are three matrices of the same dimensions, then
A+(B+C) = (A+B) + C
In indicates that matrix addition is associative.
(c) If Null matrix Omxn is added with a matrix Amxn we get only matrix Amxn. For example :
0 0 
0 =  
 0 0  22
5 7 
A =  
 2 3  22
O2x2 + A2x2 = A2x2 = A2x2 + O2x2
(2) Subtraction of Matrices
Subtraction of two matrices is possible if their dimensions are same. For example :
(aij) – (bij) = (dij) where dij = aij – bij
7 0 1
A =  
 2 7 5 23
2 8 4
B =  
1 3 6  23
 D = A–B
7  2 0  8 1  4 
or D =  
 2  1 7  3 5  6
5 8 3
=  
1 4 1 23
Dimension of matrix D will be same as that of A and B.
(3) Multiplication of a matrix by a Scalar
When a real number is multiplied with all the elements of a matrix, it is called a scalar multiplication
and the number is known as a scalar (S). For example
1 3 9 
S = Scalar = 2 A =  0 7 5 
 6 2 8 
 2 1 2  3 2  9
 SA =  2  0 2  7 2  5
 2  6 2  2 2  8 
 2 6 18
or SA =  0 14 10 
12 4 16 
In general SA = (Saij)mxn
(4) Multiplication of two matrices
If the colunins of the first matrix are equal to the rows of the second matrix (known as condition of
conformity), the two matrices can be multiplied.
If Amxa = (aij) and Bmxo = (bjk) then (AB)mxo = C = (Cik)
For example :
3 2 7
A =  
 4 6 8  23
4 1 7
B =  2 2 1 
 1 3 2  33
A and B can be multiplied because the columns of matrix A are 3 and rows of matrix B are 3. The
dimensions of their product AB will be 2×3. Let the product matrix be known as matrix C. It will be
arrived at by summing the products of the elements of the ith row of matrix A and the corresponding
elements of the kth column of matrix B. For example, for product matrix C for the above two matrices A
and B the following procedure will be applied.
Each element of the Ist row of matrix A will be multiplied by each element of the Ist column of
matrix B., then its sum will give us the first element of the product matrix C. Then each element of the Ist
row of A will be multiplied by the all the elements of the second column of B. Their sum will give us the
second element of Ist row of C. Similarly each element of Ist row of A will be multiplied by all the
elements of 3rd column of matrix B. Their sum will denote the 3rd element of Ist row of C. this process
will be repeated for all the subsequent rows of A. On the basis of this procedure C can be arrived from the
mentioned matrices A and B.
 (3  4)  (3  5)  (3  1) (2  1)  (2  2)  (2  3) (7  7)  (7  1)  (7  2) 
C=  
(4  4)  (4  5)  (4  1) (6  1)  (6  2)  (6  3) (8  7)  (8  1)  (8  2) 
(3  4)  (2  5)  (7  1) (3  1)  (2  2)  (7  3) (3  7)  (2 1)  (7  2) 
(4  4)  (6  5)  (8  1) (4  1)  (6  2)  (8  3) (4  7)  (6 1)  (8  2) 
 
(12  10  7) (3  4  21) (21  2  14) 
or C =  
 (16+30+8) (4  12  24) (28+6+16) 
 29 28 37 
or C =  
54 40 50  23
Illustration 1.
5 1
A =  
3 2 
 3 1 
B =  
 4 2
find AB and BA.
Solution :
(5x  3)  (1  4) (5  1)  (1  2) 
AB =  
 (3x  3)  (2  4) (3  1)  (2x  2) 
(15  4) (5  2)   19 7 
=   =  
 (9  8) (3  4)   1 1
(3  5)  (1 3) (3x  1)  (1  2) 
BA =  
 (4  5)  (2  3) (4x  1)  (2  2) 
(15  3) (3  2)   12 5 
=   =  
 (20  6) (4  4)   14 8
Illustration 2.
 y 
 x 5y 0 2 
A =   B=  
2 x 0  x y 
 5 10 
find AB and BA.
Solution:
 x y y 
( x  0)  (5 y  5 ) (xy 2 )  (5 y  10 ) 
AB =  
 (2 x  0)  (0  x ) (2 x x y )  (0   y ) 
 5 2 10 
 5 xy xy 5 y 2 
 (0  ) (  )  xy  y 2 
 =  
5 2 10 xy
=  2 
 2 xy  
 (0  0) (  0)   0  xy 
2 
 y  y  
 (0  x)   2  2 x  (0 × 5 y)    0  
  2  
BA = 
 x   y  y    y 
  x     2x    5 y     0 
 5   10  2   10 
 2 xy  
 0  2  (0 + 0)   xy 0
   =  2 
=  2 x  xy
 x 2 xy   5 xy   xy 
     0   
 5 
 5 10   5  
Note : In Illustration 1 and 2 AB  BA.
PROPERTIES OF MATRIX MULTIPLICATION :
(a) Matrix multiplication is associative. Let Amxn, Bnxo and Coxp; then
[A B (C)]mxp = [A (BC)]mxp
(b) Matrix multiplication is distributive.
Let Amxn, Bnxp and Cnxp. Then
A (B + C) = AB + AC
AB and AC will be of the dimensions m × p.
(c) Matrix multiplication is not necessarily commutative. i.e., AB  BA
(we have proved this propetry in Illustrations 1 and 2 above).
(d) Matrix multiplication by an Identity matrix. Let Amxn and Imxn, then ;
AI = A = I A
Example :
3 1 1  1 0 0 
A =  2 7 1 I =  0 1 0 
1 5 4   0 0 1 

 (3  1)  (3  0)  (3  0) (1  0)  (1 1)  (1 0) (1 0)  (1 0)  (1 1) 



AI = (2  1)  (2  0)  (2  0) (7  0)  (7  1)  (7  0) ( 1  0)  ( 1  0)  ( 1  1) 
 (1  1)  (1 0)  (1 0) (5  0)  (5  1)  (5  0) (4  0)  (4  0)  (4  1) 

3 1 1 
=  2 7 1
1 5 4 
AI = A
Similarly IA will be equal to A.
(e) Multiplication of a matrix by itself.
If A is a square matrix by itself.
Simliarly AA2 = A.A.A = A2A [By Associative Law]

TRANSPOSE OF A MATRIX
The transpose of a matrix is obtained by interchanging its rows into columns and columns into rows.
It is denoted by A’ or At. Let A = (aij) then A’ = (aji)nxm

FOR EXAMPLE
4 2 6
 1 1 3 
A = 
1 2 0
 
 5 4 1  43
4 1 1 5
A’ =  2 1 2 4 
6 3 0 1  34

PROPERTIES OF THE TRANSPOSE OF A MATRIX :


(a) (A’)’ = A
(Transpose of a Transposed matrix is the original matrix).
(b) (AB)’ = B’A’
(The product of matrix AB when transposed is equal to the product of Transposed matrices
taken in reverse order).
(c) (A+B)’ = A’+B’
(The Transposed sum of two matrices is equal to the sum or their Transposes).
Symmetric and Skew, Symmetric Matrices.
(1) If A’ = A, it is called a symmetric matrix (provided A is a square matrix).
Example :
 4 2 1   4 2 1 
   
A =  2 3 1 A’ =  2 3 1
 1 1 3   1 1 3 
   
Since A = A’ A is a symmetric Matrix.
(2) If A’ = -A. A will be known as a skew symmetric matrix. For example
 0 4   0 4
A =   A’ =   =–A
4 0   4 0 
Determinants
A determinant of a square matrix is a unique number associated with It. To arrive at the determinant
of a matrix it is written within vertical bars. eg.
a11 a12
|A| =
a 21 a 22
|A| = a11 a22 – a12 a21
For example :
3 5
A =  
2 7
3 5
Its determinant |A| =  
2 7
|A| = (3×7) – (5×2)
= 21 – 10
= 1
Determinant of a matrix of 3×3 is arrived at in the following way :
 a11 a12 a13 
A = a 21 a 22 a 23 
 a 31 a 32 a 33 

 a11 a12 a13 


|A| = a 21 a 22 a 23 
 a 31 a 32 a 33 

a 22 a 23 a a 23 a 21 a 22
|A| = –a12 21 +a13
a 32 a 33 a 31 a 33 a 31 a 32
= 11{a22 × a33 – a23 × a32} – a12{a21×a33 – a23 × a31} + a13 {a21 × a32 – a22 × a31}
2 1 4
For examply A = 1 0 3 
 4 2 5 

0 3 1 3 1 0
|A| = 2 –1 +4
2 5 4 5 4 2
= 2 (0×5–3×2)–1 (1×5–3×4) + (1×2–0×4)
= 2(–6)–1(–7)+4(2)
= –12+7+8
= 3
(Note – if the matrix is of 1×1 its determinant will be the number itself and if the number is negative
then the positive of that number will be its determinant).

PROPETRIES OF DETERMINANTS :
(a) If the rows of a determinant are changed into columns or vice versa, the value of the determinant
remains unchanged, For example.
a11 a12 a11 a 21
=
a 21 a 22 a12 a 22
(Note–Determinant of matrix A is equal to the determinant of A’)
(b) In a determinant if two rows (or columns) are interchanged, its absolute value remains the same
but the sign changes. For example,
 a11 a12 a13  a 21 a 22 a 23 
a a 23  =  a11 a12
 a13 
 21 a 22
 a 31 a 32 a 33   a 31 a 32 a 33 
(c) The determinant of a matrix will be zero if two rows (or columns) are identical.
a11 a12
= 0
a12 a12
Similarly
3 2 7
4 2 7 = 0
6 2 7
(d) If all the elements of a row (or column) of a determinant are zero, the value of the determinant
will be zero.
0 5 2
0 1 4 = 0
0 1 3
(e) If all the elements of a row (or column) of a determinant are multiplied by a number k, than its
value is k times the original determinant, for example :
 ka11 ka12 ka13   a11 a12 a13 
a a 22 
a 23  = k a 21 a 22 a 23 
 21
 a 31 a 32 a 33   a 31 a 32 a 33 
(f) If all the elements of a row (or column) of a determinant are written as sum (or difference) of
two or more elements, then it can be written as sum (or difference) of two or more
determinants. For example :
a11  l11 a12 a a12 l a12
= 11 ± 11
a 21  l21 a 22 a 21 a 22 l21 a 22
(g) If k times of all the elements of a row (or column) are added (or subtracted) to all the
corresponding elements of another row or column, the value of the determinant remains the
same. For example :
a11 a12 a13 a11 a12 a13
a 21  ka11 a 22  ka12 a 23  ka13 = a 21 a 22 a 23
a 31 a 32 a 33 a 31 a 32 a 33
Illistration 3
1 1 1
Prove that a b c = (a–b) (b–c) (c–a)
a2 b2 c2
Solution : (C2  C2 – C1 and C3  C3 – C1)
1 1 1 1 1 0 0
a ba c = (b – a) (c – a) a 1 1
a2 b2 c2 a2 b+a c+a
= (b–a) (c–a) (c + a – b – a) = (b – a) (c – a) (c – b)
= (a–b) (b–c) (c–a)
Illustration 4:
prove that
a b c
ab bc c  1 = a3 + b3 + c3 – 3 abc
b+c c+a a+b
Solution : (R1  R1 + R3)
a+b+c a+b+c a+b+c 1 1 1
a b bc c  1 = (a + b + c) a  b b  c c 1
b+c c+a a+b b+c c+a a+b
(C2  C2 – C1 and C3  C3 – C1)
1 1 1 1 1
= (a+b+c) a  b b  c  a+b c  a  a+b
b+c c+a  b  c a+b  b  c
1 0 0
= (a+b+c) a  b 2b  c  a  2a+b+c
b+c ab a c

2a  c  a -2a  b  c
= (a+b+c)
ab a c
R2  R2 + R1
2b  c  a + a  b -2a  b  c + a  c
= (a+b+c)
ab a c
bc ba
= (a+b+c)
a b a c
= (a+b+c) {(b–c) (a–c)–(b–a) (a–b)}
= (a+b+c) (a2+b2+c2–ab–bc–ac)
= a3+b3+c3–3abc

PRODUCT OF TWO DETERMINANTS :


|A| |B| = |AB| (Matrices A and B should be square matrices and their
dimensions should be same. Dimensions of |AB| will
be equal to the dimensions of |A| or |B|.)

TRACE OF A MATRIX
Every square matrix has its trace which is equal to the sum of its diagonal elements. For example :
1 5 4 
A = 9 2 3 
6 7 1 
Trance (tr) of A = 1–2+1 = 0

SINGULAR AND NON SINGULAR MATRICES


If the determinant of a square matrix is zero it is called a singular matrix, i.e.|A| = 0 When
|A| ฀matrix A is called a non-singular matrix.

RANK A MATRIX
The rank of matrix A is equal to the highest order of non-singular matrices formed in a matrix. For
example.
1 2
A =  
2 4
 |A| = 0
The rank of matrix A is 1 beceause A2x2 is a singular matrix and the next order non-singular matrix
is 1 (which is non-zero.)
2 4 8 
A =  3 6 12  Determinant |A| = 0
 1 2 4 
Here for any 2×2 matrix the determinant is zero. But for 1×1 matrix the determinant is not Zero.
Therefore, the rank of the above matrix is 1.

MINORS AND COFACTORS OF A MATRIX.


A lower order determinant obtained after deleting the row and column is called a minor. For
example :
 a11 a12 a13 
A = a 21 a 22 a 23 
a 31 a 32 a 33 

a a 23 
for a11 the minor is  22
a 33 
for a
 a 32
a a 23 
its minor is  21
a 33 
and similarly for
 a 31
a a 22 
a13 the minor is  21
 a 31 a 32 
In a matrix total number of minors are equal to total number of elements.
When a minor is multiplied by (–a)i+j of the respective element, it is known as a cofactor.
The co-factor of a11 in the above matrix will be
a a 23 
(–1)1+1  22
 a 32 a 32 
Similarly the co-factor of element a12 will be
a 21 a 23
(–1)1+2
a 31 a 33
Again, in a matrix there will be as many co-factors as the number of elements. (To understand it
easily, when minors are alternatively multiplied by a minus sign, they form a cofactor).
ADJOINT OF A MATRIX
Transpose of a co-factor matrix is known as the adjoint matrix. For example, if the following is a co-
factor matrix of matrix A.
c11 c 21 c13
c12 c 22 c 23
c13 c32 c33
Its abjoint will be
c11 c 21 c32
Adj = c12 c 22 c32
c13 c 23 c33
Illustration : 5
Find the adjoint of the following matrix
 2 1 1 

A =  10 4 5
 5 2 2 
Solution :
 2 5 1 
Co-factor matrix of A =  0 1 1
 1 0 2 

 2 0 1 
Adj A =  5 1 0 
 1 1 2 

INVERSE OF A MATRIX
Every non-singular square matrix possesses an inverse which is denoted as A-1 (if the matrix is a).
A-1 is arrived at by the following formula.
1
A-1 = (Adj A)
|A|
(There are other methods also for ascertaining the inverse of a matrix. They will be discussed later.
The above method of finding inverse of a matrix is known as adjoint method of finding inverse.)
Note–
1. AA-1 = I = A–1 A.
2. The inverse of a matrix is unique i.e., there cannot be more than one inverse of a matrix.
3. I = I-1
Illustration 6.
Find the inverse of the following matrix
2 4
A =  
3 4
Solution :
1
We know A–1 = (Adj A)
|A|
2 4
|A| =
3 4
= 8–12
= –4
 4 3
Co-factor Matrix of A =  
 4 2 
 4 4
Adj A =  
 3 2 
1
A–1 = (Adj A)
|A|
 1 1 
1  4 4   3 1 
=  
4  3 2   
 4 2 
Illustration 7
Find the inverse of the following matrix
1 2 1
A =  2 3 1 
 5 1 1 
Solution :
1
A–1 = (Adj A)
|A|
|A| = 1(3–1)–2(2–5)–1(2–15)
= 2 + 6 + 13 = 21
 2 3 5 
Adj A =  3 6 3
 13 9 1

 2 3 5 
1 
A–1 =  3 6 3
21
 13 9 1

 2 1 5
 21 7 21 
 
1 
= 
1 2
 7 7 7
 
 13 3 1 
 21 7 21 
ELEMENTARY ROW OPERATIONS AND INVERSE OF A MATRIX
Row operations method is another technique of finding the inverse of a matrix. For finding the
inverse, following row (or column) operations may be undertaken :
(a) The rows (or columns) may be inter changed.
(b) All Elements of a row (or column) may be multiplied (or divided) by a non-zero constant and
thus arrived row (or column) may be added or subtracted from another row (or column)
(c) A row (or column) may be replaced by such row (or column) which is calculated by multiplying
the original row (or column) by a non-zero constant.
For finding the inverse, we should write the matrix and an Identity matrix in augmented form. By
any of the above operations we convert the matrix into Identity matrix and the adjacent Identity matrix
into the inverse matrix. into the inverse matrix.
For example :
4 2
If A =  
1 3
The augmented matrix will be
4 2 1 0
 
1 3 0 1
[R1  R2]
1 3 0 1
 
4 2 1 0
[R1  R2 – 4 R1]
1 3 0 0
 
0 10 1 4 

 1
R 2  R 2 + 10 
 
 0 1
1 3 1 2 
0 10 
 10 5 
[R1  R1 – 3 R2]
 3 1
1 0  
10 5
 
0 1 
1 2 
 10 5 
3 1
 10  5 
 A–1 =  
 1 2 
 10 5 
Illustration 8
Find the inverse of the following matrix by Row operation method.
1 2 7 
A = 0 1 3 
 3 2 4 
Solution :
1 2 7 1 0 0
 
0 1 3 0 1 0
 2 2 4 0 0 1 

R1  R3 – 3 R1
1 2 7 1 0 0
 
0 1 3 0 1 0
 0 4 17 3 0 1 

R1  R3 – 2 R2 and R3  R3 – 4 R2
1 0 1 1 2 0 
 
0 1 3 0 1 0
 0 0 5 3 4 1 

R3  R3 + – 5 R2 R2–3R3 and R1 – R1– R3


 
 2 6 1
1 0 0 
 5 5 5
 9 17 3
0 1 0 
 5 5 5
 3 4 1 
0 0 1 5 5 5
 
 
 2 6 1
5 5 5
 
 9 17 3
 A–1 =
5 5 5
 
 3 4 1 
 5 5 5 

SYSTEM OF LINEAR EQUATIONS


In general, a system of m linear equations in the n unknowns x1, x2,...xn is of the form
a11 x1 + a12 x2+...+ain xn = b1
a21 x1 + a22 x2+...+a2n xn = b2
. . .
. . .
. . .
. . .
am1 x1 + am2 x2+...amn xn = bm

where aij and bi are real numbers and m and n may, in general be different.
The above system of equations may be written as a single matrix equation i.e.,
AX = B
–1
and if A exists, we can write as
X = A–1 B
If A–a does not exist, the system of equations is inconsistent.
Example :
Solve the following equations using matrices.
5x1 + 2x2 = 2
4x1 – 3x2 = 10
Solution :
In the matrix form the above equations can be written as follows
AX = B
5 2  x  2
where A =   X =  1 B=  
 4 3  x2  10 
|A| = (5× – 3) – (2 × 4)
= –15 – 8
= –23
Adj
A–1 =
|A|
 3 2
Adj A =  
 4 5 
3 2
 23 
 A–1 =  23 
4 5 
 23 23 
Since A–1 exists, we can write the equations in the following form also :
X = A–1 B
 3 2 
 x1   23 23   2 
  =   
 x2   4 5  10 
 
 23 23 
6 20   26 
 23   23 
=  23   
8 50   42 
 23  
23   23 
26 42
 x1 = and x2 =
23 23
Example :
Solve the following system of linear equations by inverse method :
x + 2y + 3z = 14
3x + y + 2z = 11
2x + 3y + z = 11
Solution:
1 2 3   x 14 
 
A =  3 1 2  x =  y  B = 11
 2 3 1   z  11
|A| = 18
Adj A
A–1 =
|A|
 5 5 1 
Adj A =  1 5 7 
 7 1 5
 5 7 1 
  18 18 18 
 
5
d= 
Adj A 14 7 
A–1 =
|A|  18 18 18 
 
 7 1 5 
 18 18 18 
X = A b –1

 5 7 1 
 18 18 18 
 x  
 y  =  14 5 7 
   18 18 18 
 z   
7 1 5 
 18 18 18 
 5 7 1 
 18  14  18  11  18  11
14   
11 =  10  14  5  11  7  11
   18 18 18 
11  
 7  14  1  11  5  11
 18 18 18 
X = 1
ie. y = 2
z = 3
Linear equations can also be solved by two more method. They are :
(i) Row elimination method
(ii) Cramer’s Rule
Under Row elimination method, equation are written in the form of augmented matrix (i.e. append B
matrix to coefficient matrix A in the form A/B) and row operations, as earlier discuser in this chapter, are
applied. In the place of coefficient matrix we obtain an Identity matrix and in the place of B matrix we
obtain the values of unknown variables ie. X1, X2...Xn. This method can be easily understood by the
following example :
If x + y + z = 45
–x + 0y + z = 8
x – 2y + z = 0
In the augmented matrix form the above matrix can be written as follows :
1 1 1 45
 
 1 0 1 8
 1 2 1 0 

R2  R2 + R1 and R3  R3 – R1
1 1 1 45 
 
0 1 2 53 
 0 3 0 45 

R1  R1 + R2 and R3  R3 – 3R2
1 0 1 8 
 
0 1 2 53 
0 0 6 114 

R3  R3 + 6. R2  R2 and R1  R1 + R3
1 0 0 11
 
0 1 0 15 
0 0 14 19 

 The value of x = 11;y =15 and z = 19

SOLUTION OF LINEAR EQUATIONS BY CRAMERS’ RULE


Let a1x + b1 y + c1 z = k1
a2x + b2 y + c2 z = k2
a3x + b3 y + c3 z = k3
In the matrix form it can be written as :
AX = K
If |A|  O, then the solutions of the above equations by Cramer’s Rule can be arrived by the
following method
 k1 b1 c1   a1 k1 c1   a1 b1 k1 
k b2 c 2  a k2 c 2  a b2 k 2 
 2  2  2
k c3  a c3  a k 3 
x=  3 y=  3 and z =  3
b3 k3 b3
|A| |A| |A|
Cramer’s Rule can be easily understood by the following example
If x1 + 2x2 + x3 = 6
2x1 + 3x2 + 4x3 = 12
3x1 + x2 + 2x3 = 7
find the values of x1, x2 and x3 by Cramer’s Rule.
Solution :
1 2 1  6
A =  2 3 4  K = 12 
 3 1 2   7 

1 2 1 1 6 1 1 2 6
12 3 4 2 12 4 2 3 12
7 1 2 3 7 2 3 1 7
x = _______ y = _______ z = _______
1 2 2 1 2 1 1 2 1
2 3 4 2 3 4 2 3 4
3 1 2 3 1 2 3 1 2
11 22 11
x = ; y= ; z=
11 11 11
 X = 1, y = 2 and z = 1

LEONTIEF’S INPUT-OUTPUT ANALYSIS


Input-output analysis was first attempted by Prof. Leontief, hence the name of this technique is
Leontief’s Input-Output Analysis. In every economy the output of an industry is used as input by other
industries and also consumed by ultimate consumers (which includes government purchases, exports and
consumption by public). Thus output is said to equal to total demand. The part of output of industries
utilized as input in the economy is known as intermediate demand and the part bought by ultimate
consumers is known as final demand. Following assumptions are inherent in this analysis.
(1) Each industry produces only one type of goods. If more than one type of items are produced,
then they are considered different industries.
(2) The total output of each industry is sufficient to meet the intermediate as well as the final
demand. Shortages or surpluses are not permitted.
(3) Xij = ay xj
where xij = output of industry i that will be required by industry j
aij = (Constant) proportion of output of industry i required
by industry j for producing one unit.
xj = Total output of industry j
Input-output models are of two types : closed model and open model. Under the closed model, total
output in an economy is consumed by industries and under the open model, ultimate consumers also
consume output in addition to consumption by industries.
In an open model if there are two industries. I and Ii, output of industry I is used by industry I and II
and the remainder is used by final consumers. Similar is the case with industry II. The following table
describes the interaction of the uses of output of the two industries :
Industry I II Final Demand Total
I 16 20 4 40
II 8 40 32 80

The output of industry I is 40 units as shown in the Ist row. Out of these, 16 units are consumed by
industry I, 20 units by industry II and final consumers take 4 units. The second row can be interpreted in
the similar way. It is forecasted that the final demand will increase to 18 units and 44 units for I & II
respectively in future.
The above information can be shown in an input-output matrix. In terms of ratios, the requirement
16 8
of industry I for producing one unit will be = A and = 2 from industry I and Ii respectively,
40 40
20 40
Similarly for industry II, the ratios will be .25 and = 5. Thus the input-output matrix will be:
80 80
I II
A = I .4 .25
II .2 .5 
[Note:-Matrix A is also known as technology matrix]
We observe that columns represent the amount of material required for producing one unit of output.
I
If matrix X =   indicates the total output needed to obtain the required demand, then AX indicated the
 II 
ratios of inter-industry input. Thus,
AX + D = X
or (1–A) = D
or X = (1–A)–1 D
where (i) D is the final demand matrix
(ii) (1–A)–1 is also known as transaction matrix
1
I  1 0   .40 .25   18
 II  =  0 1    .20 .30   14
       
 .60 .25
(1–A) =   ; |1–A| = .25
 .20 .50 
Adj 1-A  .2 1 
(1–A)–1 =   
|1-A | .8 .24 
we know X = (1 – A)–1 D
I .2 1  18 
 II  = .8 .24  44
     
I  2  18  1  44   80 
 II  = .8  18  2.4  44  = 120 
     
 I = 80 and II = 120
Howkins- Simon Conditions
(i) |1–A|  0
(ii) All the diagonal elements of the transaction matrix must be positive. Sometimes, when these
conditions are not fulfilled, the solution may be inconsistent.
Illustration :
The input-output technology matrix for three industries is
 2 .3 .1
A =  0 .5 .2 
 0 0 .5 
If the final demand for three industries is
100 
D =  200 
 300 

 x1 
find the gross output X =  x2 
 x3 
Slution :
We know x = [1–A]–1 D
.8 .3 .1
1–A =  0 .5 .2 ; |1–A| = .2
 0 0 .5 

.25 .15 .11


Adj 1–A =  0 .4 .16 
 0 0 .4 
 25 15 11 
 20 20 20 
 
Adj 1-A 
= 0
16 
[1–A–1] = 2
|1-A |  20 
 
0 0 2
 
 25 15 11 
 20 20 20 
  100 
X = 0 2
16   200 
 20   
   300 
0 0 2
 
 440 
=  640 
 600 

EXERCISES
3 1 7 3
(1) If A =   , B=   , find [A + B]
2 9 6 0
Solution :
3  7 1  3 
[A+B] =  
2  6 9  0
10 4 
=  
 8 9
 5 6 1   2 3 2
(2) If A =   and B =  
4 3 4 3 4 1
5  2 6  3 1  2 
Solution : (A–B) =  
 4  3 3  4 4  1
3 9 1
=  
1 1 3 
 3 2 a b
(3) If A =   and B =   find a and b
4 1 3 5
such that AB = BA. Compute 3A+5B.
Solution :
 3 2   a b   3a+6 3b+10 
AB =    = 
 4 1   3 5   4a  3 4b  5 
 a b   3 2   3a+4b 2b+b 
BA =    = 
 3 5   4 1   9  20 6  5 
 AB = BA
 3a+6 3b+10   3a+4b 2b+b 
or   =  
 4a  3 4b  5   29 11 
3a + 6 = 3a + 4b 3b + 10 = 2a + b
b = 1.5 b–1 =5
4a + 3 = 29 4b + 5 = 11
a = 6.4 b = 1.5
 a = 6.5 b = 1.5
 3 2  9 6
3A = 3   = 
4 1 12 3 
a b  5a 5b 
5B = 5   = 
3 5  15 25 
9 6  5a 5b 
3A+5B =   + 
12 3  15 25 
 5a+9 5b+6 
=  
 27 28 
1 0 1  2 1 
(4) If A = 1 1 1 , B = 1 1 C = [1 0 2]
0 2 2   2 2 
find AB, A’B, AC, and CB
Solution :
1 0 2  2 1 
AB = 1 1 3  1 1
 
0 2 1  2 2 

 1 2  0  1  2  2 1  1  0  1  2  2 
=  1 2  1 1  3  2 1 1  1 1  3  2 
0  2  2  1  1 2 0  1  2  1  1 2 
6 5 
= 9 6 
0 4 

1 1 0  2 1 
A’B =  0 1 2  1 1
 
 2 3 1  2 2 

 1 2  1 1  0  2 1  1  1  1  0  2 

=  0  2  11  2  2 0  1  1  1  2  2 
 2  2  3  1  1  2 2  1  3  1  1  2 
3 0 
= 5 3 
5 3

1 1 0  1 
AC’ = 0 1 2  0
 
 2 3 1  2

 1 1  0  0  2  2   5 
=  1  1  1  0  3  2  =  7 
0  1  2  0  1  2   2 

1 0 2  2 1 
CB =  

 1 1
 
   2 2 
= [1×2+0×1+2×2 1×1+0×-1+2×2]
= [6 5]
(5) In a certain city there are 50 colleges and 400 schools. Each school and college has 18 peons, 5
clerks and one cashier. Each college in addition has one section officer and one librarian. The monthly
salary of each of them is as follows :
Poem–Rs 300. Clerk–Rs. 500, Cashier–Rs. 600,
Section Officer–Rs. 700 and Libration–Rs. 900.
Using matrix notation find (1) total number of posts of each kind in schools and colleges taken
together, (ii) the total monthly salary bill of all the schools and college taken together.
Soltution :
No of Colleges and schools : A = [50 400]1×2
18 5 1 1 1 
No. of employees B =  
18 5 1    25
Salary matrix S = [300 500 600 700 900]1×5
18 5 1 1 1 
Total no. of posts AB = [50 400]  
18 5 1   
= [8100 2250 450 50 50]1×5
Total monthly salary will of each college & school is
18 5 1 1 1 
BS’ =  
18 5 1    25
300 
500 
  80,100 
 600  =  
   8500 
 700 
900 
51

Total monthly bill of all colleges & schools


80,100 
A(BS’) = [50 400]  
 8500 
= [3905000]
 y  z x y
(6) show that  z  x z x  = (x + y + z) (x – z)2
 x  y y z 
Solution :
R1  R1 + R2+ R3
 2( x  y  z ) x  y  z x  y  z
 zx z x 
 
 x  y y z 

 2 1 1

= (x + y + z)  z  x z x 
 x  y y z 
C1  C1 – C2+ C3 and C2  C2+ C3
 0 0 1
= (x + y + z)  0 z  x x 
 x  z y  z z 

 0 z  x
= (x + y + z) 
x  z y  z 

= (x + y + z) (x - z)2
 1 4 2
(7) Given the matrix X =  1 2 1 
 1 3 2 

Show that XX–1 = 13


Solution :
Adj x
X–1 =
|x|

|X| = |4 – 3|–4|–2–1|+2|–3–2|
= 1+12–10 = 3
 2 1 4 2 4 2 
  
 3 2 3 2 2 1 
 1 1 1 2 1 2
Adj X =  
 1 2 1 2 1 4 
 
 1 2

1 4 1 4
 1 1 2 
 3 1 3
 1 2 0 
=  3 0 1
 5 1 2 

1 2 
3 0
3
 
 X–1 =  1 0 1
 5 1 
 2
 3 3 
1 2 
0
 1 4 2  3 3
 
X X–1 
=  1 2 1   1 0 1
 1 3 2   5 1 
 2
 3 3 
 1 5 2 1 
 1 3  4  1  2  3 1
3
 40  2
3
1  0  4  1  2  2 
 
5 2
=  1   2  1  1 1  0  2  1  1  2 
1 1
 1  2  0  1
 3 3 3 3 
 
1
 1  3  1  2  5 2 1
1  3 0  2  1  0  3  1  2  2 
 3 3 3 3 
1 0 0 
X X–1 =  0 1 0  = 13
 0 0 1 
(8) Given the following equations for two related markets A and B, find the equilibrium conditions
for each market and the price for each market :
Xa (A) = 82 – 3PA+ PB Xx (A) = –5 + 15PA
Xd (B) = 92 – 2PA–4 PB Xs (B) = –6 + 32P8
Slution :
For equilibrium conditions
Xd A = Xs(A) and Xd (B) = Xs(B)
82 – 3PA+ PB = 5+15PA 92 + 2PA– 4PB = 6 + 32PB
18PA = 87 –2PA + 36PB = 98
Matrix form
18 1  PA  87 
 2 36   P  = 98 
   B  
AP = B
P = A–1 B
 36 1 
Adj A 1 36 1   346 646 
A–1 =  =  
|A| 646  2 18   2 18 
 646 646 
 36 1   36  87 98 
  
646 87  = 6464 646 
P =  346     
 2 18  98   2  87 18  98 
 646 646   646 646 
 3230 
 646  5 
  =  
 1938   3
 646 
 PA = 5 and Pa = 3
(9) A company produces three products every day. There total production on a certain day is 45
tons. It is found that the production of third product exceeds the productions of first product by 8 tons
while the total production of first and third product is twice the production of second product. Determine
the production level of each product using Cramer’s rule.
Solution :
X + y + Z = 45
z = 8+x
X + Z = 2y
or
x + y + z = 45
–x + 0y + z = 8
x – 2x + z = 0
AX = B
|A| = 6
45 1 1
8 0 1
0 2 1 66
X = = = 11
6 6
1 45 1
1 8 1
1 0 1 90
y = = = 15
6 6
1 1 45
1 0 8
1 2 0 114
z = = = 19
6 6
x = 11; y = 15 and z = 19.
(10) Suppose the inter-industry flow of the products of two industries are given as under :

Consumption
x y Domestic demand gross output

Products x 30 40 50 120

Y 20 10 30 60

Determine the technology matrix and test Simon-Hawkins conditions for the viability of the tested.
 80 
Compute the equilibrium level of output of the products when the domestic demand vector is  
 40 
Solution
 30 40 
 60 
Technology Matrix = A = 120 
 20 10 
120 60 
1 2
 3
= 4 
4 1
 6 6 
We know Ax + D = x
(1 – A) x = D
x = (1–A)–1 D
 3/ 4 2 / 3 
(1–A) =  
 1/ 6 5 / 6 
5 2
 3
|1–A| = Adj (1–A) =  6 
1 3
 6 4 
 5 72 2 72 
Adj 1-A   3  37 
3 37 
|1–A|–1 = =  
|1  A |  1  72 3 72 

 3 37 4 37 
X = (1–A)–1 D
 60 48   60  80 48  40 
 37 37   80   37 37 
=      
 12 54   40  12  80 50  40 
 37 37   37 37 
181.62 
=  
 84.32 
Simon–Hawkins conditions :
(1) |1–A|  0
(2) Diagonal elements of (1–A) should be positive.
Both the conditions are satisfied in the above problem.
LESSON 6

LINEAR PROGRAMMING
(GRAPHIC METHOD)

Various new branches has emerged in the study of mathematics over a duration of time. They
include operationl research, mathematical statistics and computer programming. Among these.
operational research is one of the most important branches. It deals with various mathematical models
which are often applied in managerial science. Among the mathematical models, linear programming is
the most important. It inculdes various models such as simplex model, transportation model and
assignment model. Simplex model is a generalised linear programming. These models assume that there
exist linear relations amongst different quantities appering in the programme. Prof. George B. Dantiz
developed simplex to solve lincar programming problems in 1951.
The linear programming models-are applicable to problems genes ally faced by Manage-ment and
Econonmts, e.g allocating the limited resources among competitive uses iii such a way that their
utilization is optimal. The type of problems which are solved by the. linear program-ming models has
certain objective function subjecting to certain restrictions. The solution of these problems need
optimizing the objective function in terms of profit and minimising in terms of loss. Let us take an
example :
A firm produces two products-Vitamin A and B requiring different mix-up of the materials Z and Y.
Vitamin A requires 2 units of Z and 3 units of Y and Vitamin B requires 2 units of X and one unit of Y.
Vitamin A yields a profit of Rx. 2/- per unit and Vitamin B yields Rs. 1/50- per unit. If the firm bas 200
units of material X and 150 units of Y. Determine the objective function and constraints assuming firm
produces X1 units of Vitamin A and X2 units of Vitamin B. If X1 units Vitamin A and X2 units of Vitamin
B are produced, the objective function will be.
Maximize P = 2x1+1.50 x2
subject to
2x1 + 2x2 < 200 (X material)
3x1 + x2 < 1500 (Y material)
where x1 > 0, x2 > 0
Hence values of x1 and x2 are to dctetiained in such a way that P can be maximized in terms of profit
within the available materials.
Programming is a technique which is applied for finding maximum values in problems confronting
the decision-making authorities subject to certain constraints or side-conditions., which limit decisions. In
fact such problems which require determination of maxima or minima could also be solved with the help
of Calculus. But calculus can not be applied in problems having Side conditions which are not exact but
limit the requirements. Programming helps us to find out maximum or minimum values when side
conditions are inequalities and not equations. The term linear implies that the relationship existing in the
problems must be linear "while the term programming refers to determination of particular programming
which gives the line of action which would achieve the desired objective. Linear programming uses, a
mathematical model-in which a linear objective function miximised of maximised subject to certain linear
constraints.
The application of Linear Programming assumes these conditions :
(1) The Linear Programming technique can be applied to those problems which has got constraints
in their activities. These constraints are due to technological limitations or capacity and resource
limitations. They primarily due to limited resources.
(2) The Linear Programming problem is a ........ near relations. But functional relationship may not
be linear in many problem ....... can not be solved by this technique.
(3) While applying Limear Programming, .... and outputs are taken constant over a given period of
time.
(4) Activity levels are permitted to take fractional or intger values. Linear Programming consists of
three parts :
(i) Objective function : which is to maximised or minimised,
(ii) Structural constraims : which states the side conditions for different activities.
(iii) Non-negativity constraints : which states that there will no negative value of the variable
involved.
We can take few problems to illustarte the nature and structure of linear programming problems :
Example : A fertilizer company produces two types of fertilizers called grade I and grade II. Each
of these is processed through two critical chemical plant units. Plant A has a maximum of 120 hours
available in a week and plant B has a maximum of 180 hours. Manufacturing 1000 kg. of grade I fertilizer
requires 6 hourse in plant A and 4 hours in plant B and manufacturing 100 kg. of grade Ii fertilizer
required 3 hours iri plant A and 10 hours in plant B. If profit is Rs. 450/- per 1000 kg. of grade I and Rs.
550/-per 100 kg. of grade II, it is required to find out the amounts of two types of fertilizer that should be
meanufactured to maximise profit.
Solution : The problem can be solved as follows :
Denote X3 the amount in 1000 kg. of grade I fertilizer produced and x2 the amount in 1000 kg. of
grade Ii to be produced. The total profit is then
450 x1 + 550 x2
and this is to be manimised subject to plant capacity constraints. Under the production scheme,
6x1+3x2 is the number of hours to be utilized in plant A and this can not exceed 120. The constraint is
written therefore as
6x1 + 3x2 < 120
In a similar way, the plant B capacity constraint can be written as
4x1 + 10x2 < 180
We have to add two more general constraints
x1 > 0, x2 > 0
because the amount to be produced is always non-negative.
The complete mathematical formulation
Find x1, x2 ...(i)
Maximie 450x1 + 550 x2
Subject to
6x1 + 3x2< 120 (Plant A capacity constraint) ...(ii)
4x1 + 10x2 < 180 (Plant B capacity constraint) ...(iii)
x1  0
 0 
(Non-negative constraints) ...(iv)
x2
where (i) is the objective function, (ii) & (iii) are structural constraints. (iv) Non-negative constraints
Since the objective function and constraints are .... function in x1, x2 we have linear programming
problem.

GENERAL FORM OF LINEAR PROGRANMING PROBLEM


Maximize Z = c11 c1 + c12 x2 + x13 x3...............+c1n xn
Subject
a11 x1 + a22 x2+...............+...a2n xn < b1
. . . . .
. . . . .
. . . . .
. . . . .
. . . . .
where x1, x2,.........,xn > 0

It can be put as
n
Maximize Z =  Cij xj
j 1
Subject to
n
 aij < bj where i = 1, 2,......m, xj > 0
j 1
Therefore it has m inequality constraints and n variables; a, b, c are constants. The constraints in a
linear programming problem may be of less than .or equal to type; greater than or equal to type, equality
type or mix type. Generally the constraints are of < type in maximization problems and > type in
minimization problems.
The first important step in solving a linear programming problem is the determination of objective
function to be optimised together wills constraints. This is known as formulation of problem.
Example : A media specialist has to decide OB the allocation of advertising in three media vehicles.
Let xi be the number of messages carried in the ith medium, . i -1, 2, 3. The unit costs of a message in the
three media are Rs. 1000, Rs. 750 and Rs. 500. The total budget available is Rs. 25,000 for the campaign
period of a year. The first media is a monthly magazine and it is desired to advertise not more than one
insertion in one issue. Al least six messages should appear in the second media. The number of messages
in … third media should be between 4 & 8. The effective audience for unit message is given below
Vehicle .... audience
1
2
3
It is required lo find optimal allocation that would ..... total effective audience.
Solution : The linear programming formulation shall be as below :
Maximize z = 80,000 x1+ 65,000 x2+ 50,000 x3
Subject to
1,000 x1 + 750 x2+500 x3 < 25,000
x1 < 12
x2 >6
x3 < 7
x3 > 5
where x1, x2, x3 > 0
Example.: A fanner has 100 acres of farm. He can sell all the tomatoes, lettuce or radish he
produces, The price he can obtain is Rs. 1 00 per kg. for tomatoes, Rs, 0,75 a head for lettuce and Rs. 2,00
per kg. of radishes. The average .yield per acre, is 2000 kg. of tomatoes. 3000 heads of lettuce and 1000
kg. of radishes-Fertilizer is available at Rs. 0,50 per kg and the amount required per acre is 100 kg. each
for tomatoes sad lettuce and 50 kg. for reddish. Labour required for sowing, cultivating and harvesting per
acre is 5 man-hours for tomatoes and radish and 6 man-hours for lettuce. A total of 400 man days of
labour are available at Rs, 20 per man day. Formulate the problem as a Linear Programming model to
maximise the farmer's total profit.
Solution : Let x1 acres of farm is used for tomtoes cultivtion, x2 acres for acres for lattice and x3
acters for radish, then the total salve will be :

Corp Acres Yield Total yield Rate per unit Total sales
per acre
Tomato x1 2000kg. 2000 x1 Rs. 1.00 per kg Rs. 2000 x1
Lettuce x2 300 heads 3000 x2 Rs. 0.75 per head Rs. 2250 x2
Radish x3 1000 kg 1000 x3 Rs. 2 per kg Rs. 2000 x3

The cost of fertilizer will be :

Corp Fertilizer Acres Total fertilizer Rate Total cost


for each crop
Tomato 100 kg x1 100 x1 0.50 per kg. Rs. 50 x1
Lettuce 100 kg x2 100 x2 0.50 per k. Rs. 50 x2
Radish 50 kg x3 50 x3 0.50 per kg Rs. 25 x3

Cost of labour will be :


Corp Acres Man days Total man cost per day Total cost
per acre day
Tomato x1 5 5 x1 Rs. 20 Rs. 100 x1
Lettuce x 6 6 x2 Rs. 20 Rs. 120 x2
Radish x3 5 5 x3 Rs. 20 Rs. 100 x3

Therefore profit function to be maximised will be :


Profit = Total Sales–(Cost of fertilizer + cost of labour)
= 2000 x1 + 2250 x2 + 2000 x3–[(50 x1 + 50x2 + 25 x2]
+ (100 x1 + 120 x2 + 100 x3)]
= 1850 x1 + 2080 x2 + 1875 x3
Hence Maximize P = 1850 x1 + 2080 x2 + 1875 x3
Subject to
x1 + x2 + x3 < 100 (land)
5x1 + 6x2 + 5x3 < 400 (man days)
where x1, x2, x3 > 0

Example : A city hospital has the following minimal daily requirements for nurses :

Period Clock Time (24 hours day) Minimal Number of Nurses required
1 6 A.M.-10 A.M. 2
2 10 A.M.-2 P.M. 7
3 2P.M.-6P.M. 15
4 6 P.M.-10P.M. 8
5 10P.M.-2A.M. 20
6 2A.M.-6A.M. 6

Nurses report to the hospital at the beginning of each period and work for 8 consecutive hours. The
hospital wants to determine the minimal number of nurses to be employed so that there will be sufficient
number of nurses available for each period. Formulate this as a Linear Programming problem by setting
up appropriate constraints and objective function.
Solution : Let x1, x2, x3, X4, x5 and xe nurses are employed in period 1, 2, 3, 4, 5 and 6 respectively.
So objective function is to minimize the number of nurses
Mininze Z = x1 + x2 + x3 + x4 + x5 + x6
Subject to
In the first period x1 nurses are new and x6 nurses employed in 6th period are still on duty. So
x1 +.......... ..........x6 > 2
x + x2 >7
In the same way
x2 + x > 15
x2 + x4 >8
x4 + x5 > 20
x5 + x6 >6
x1, x2, x3, x4, x5, x6 >0

The Linear Programming problems can be solved by two methods :


1. Graphic method.
2. Simplex method.

GRAPHIC METHOD
The following steps are considered :
(1) Formulate the appropriate Linear Programming problem
(2) Construct graph for all the structural constraints. Convert each inequality into equality for each
equation and select two points and plot them on the graph and connect by an appropriate line.
(3) After drawing the graph for each inequality constraints with their feasibility region determine
the common region of all the constraints including non-negativity restrictions. It is known as common
feasible region or area. All the points in this area represents a solution to the problem. This area lies in the
first quadrant of the graph because of non-negativity restrictions For less than or equal to and less than
constraints the feasible area is on or below these lines and for greater than or equal to and greater than
constraints, the feasible area will be on or above these lines.
(4) The last step is tracing the point from the common feasible region which optimises the objective
function. The optimal solution lies in the corner points. Therefore optimal solution can be evaluated at
various corner points.
Example : A factory manufactures two articles A and B. To manufacture the article A, a certain
machine has to be worked for 1.5 hours, and in addition a craftsman has to work for 2 hours. To
manufacture the Article B, the machine has to be, worked for 25 hours and in addition the craftman has to
work for i.5 hours. In a week the factory can avail of 80 hours of machine time and 70 hours of craftsman
time. The profit on each article A is Rs. 5 and on each article B is Rs. 4. if ail the articles produced can
be sold away, find how many of kind should be produced to earn the maximum profit per week.
Solution:
Step I :
Formulate the Linear Programming problem. Let us take
x1 = number of units of article A
x2 = number of units of article B
Maximise P – 5 x1 + 4x2
Subject to
1.5 x1+ 2.5 x2 < 80
2 x1 + 1.5 x2 < 70
x1 , x2 >0
Step II
We construct the graph by drawing a horizontal and verticle axis which are represented by the x1
axis and x2 axis in the Cartesian plane. Since any points which satisfies the conditions X1 > 0 and x2 > 0
lies in the first quadrant only and our explanation will also be restricted to the points of the first quadrant
only.
Now the inequalities are graphed taking them as equalities e.g., the first constraint 1.5
x1 + 2.5x2 < 80 will be graphed as 1.5 x1 + 2.5 x2 = 30, and the second constraint 2x1 + 1.5x2 < 70 as
2x1 + 1.5x2 = 70 and the third constraint x1, x2 > 0, merely restricts the solution to non-negative values.
Further since the functions to be graphed are linear, we need to plot only two points for each
constraint. Thus to graph each constraint, we arbitrarily assign a value to KI and determine the
corresponding value of x2. The procedure is then repeated for another pair of values for the same
constraint. Then for the first constraint we have two such points as N (0, 32) and Q (53.3, 0) which upon
joining represents
1.5 x1 + 2.5 x2 = 80

Similarly, converting the inequality into equality 2 x1 + 1.5 x2 =70 , we can obtain point R (0, 46.7)
and S (35, 0) and draw graph for 2x1 + 1.5 x2 < 70.
Step III
The feasible region is the area of the graph which contains all pairs of values that satisfy all the
constraints. In other works, feasible region will be bounded by the two areas, and the two lines
1.5 x1 + 2.5 x2 = 80, 2 x1 + 1.5 x2 = 70, and will be the common area which falls to the left of these
constraint equations as both the constraints are of the 'less than equal to' type
Step IV
After locating the feasible area consider the corner points of this area which are O, N, T and S Let us
evaluate the objective function at these corner points.

Corner point Objective function Value


(X1 X2) P = 5x+ 4x2

O = (0, 0) 5×0+4×0 P (O) = 0


N = (0, 32) 5 × 0 + 4 × 32 P (N) = 128
T = (20, 20) 5 × 20 + 4 × 20 P (T) = 180
S = (35, 0) 5 × 35 + 4 × 0 P (S) = 175

The optimal solution is that corner point for which the objective function has the largest value. Thus
the optimal solution to the present problem occurs at the point T = (20,20) where x1 = 20 and x2 = 20 with
the objective function value of Rs. 180.
Therefore the company should manufacture 20 units of article A and 20 units of article B per week
to maximise profits.
Example : A firm makes two types of furniture :.chairs and tables. The contribution for each
product as calculated by the accounting department is Rs, 20 per chair and Rs. 30 per table. Both products
are processed on three machines M1, M2 and M3. The time required in hours by each product and total
time available! hours per week in each machine area follows
Machine Chair Table Available time
M1 3 3 36
M2 5 2 50
M3 2 6 60

How should the manufacturer schedule his production in order to maximize contribution.

(Use Graphic Method)

Solution : Assuming firm decides to make x1 chairs and x2 tables, the objective function with
constraint will be :
Maximizing P = 30 |x1+30| x2
Subject to 3 x1+3 x2| < 36
5 x1 + 2 x2 < 50
2 x1 + 6 x2 < 60
x1 + x2 < 0

1t shows that corner point of common feasible regions are A(0, 10), G(3, 9) H(26/3, 10/3) and F (10,
0), but optimal solution occurs at (3, 9) where P is maximum P = Rs. 330.
Example : A company sells two different products A and B. The company makes a profit of Rs. 40
and Rs. 30 per unit on both products. The two products are produced by a common production process
and are sold in two different markets. The production process has -a capacity of 30,000 man-hours. It
takes 3 hours to produce one unit of A and one hour to produce one unit of B The market has been
surveyed and company officials found out that the maximum units sold for product A and B are 8,000 and
12.000 units. Subject to these limitations, the products can be sold in any combinations. Formulate the
above as a linear programming problem and solve it by graphical method.
Solution : Let us assume that company sells: x1 units of A and x2 unite of B. Then objective function
will be
Maximize P = 40 x1 + 30x2
and the constraints will be limited capacity available
3x1 + x2 < 30000 (man hours)
x1 < 8000 (sales)
x2 < 12000
x1, x2 > 0
We can draw graph for the above formulated problem.
The graph shows that corner points of feasible area are B (0, 12000), E (6000, 12000), F(8000,600),
D (8000,0) and O (0,0) but the optimal solution occurs at E (6000,12,000) where the value of P will be
Rs. 6,00000. Therefore P will be maximum when the company sells 6000 units of x and 12000 units of
X2 product. So far we have considered problems for which unique optimum solution exist. But, in actual
practice linear programming may be such that a unique optimum solution does not exist. These
exceptional cases can be, where no feasible solution exist, unbound solution, multiple solution or
degenerate solution. Let us illustrate these situations in graph.
No Feasible Solution : In a linear programming problem it may happen that no solution is possible.
It arises where constraints are such that they do not form a common feasible region.
Example : Maximize P = 2x1 + x2
Subject to
x1 + x2 < 4
x2 < 5
x1 + x2 > 0
Solution : By drawing grapy, we find
That there is no feasible region and hence no solution to the problem.
Unbounded solution : Unbounded solution : A linear programming problem may have unbounded
solution which means it has no limit on the constraints. It simply means the common feasible region is not
bounded in any respect. The primary variables can take any value is the unbounded region. In such
situations, the optimum solution may exist or may not exist. It will not be existing if the value if objective
function goes on changing in the unbounded region. But the optimal solution will be existing of the value
of objective function in the unbounded feasible region is less than the value at the vertex.
Example : Maximize P = 2x1 + 3x2
Subject to
2x1 + x2 > 6
x1+x2 > 8
x1 + x2 > 0
Solution : By drawing a graph, we find out common feasible region as unbounded
Multiple solution : Another possible outcome of linear programming problem may be in the form
of multiple solution. It means that there are more than one solution which optimizes the objective
function. According to extreme point theorem if the value of objective function is same are more than one
vertex of the feasible region, all the points on the boundary of common feasible region represents the
optimum solution.
Example : Maximize P = 6x1 + 4x2
Subject to
x1 + 2x2 < 16
3x1 + 2x2 < 24
x1, x2 > 0
Solution : We can draw a graph from the problem of Linear Programming.
The optimal solution is at E (4, 6) where P = Rs. 48 but as D (8, 0) also P = Rs. 48. Hence it means
that all the points at E D boundary line, has got optimum solutions. Therefore the objective function is
parallel to second constraint.
Degeneracy : Tie linear programming problem may be degenerate type. Degeneracy happens when
the two constraint boundary, where one constraint is redundant, intersect at one axis of graph, Redundant
constraint is that which is not affecting the feasible region in any way. in this situation, there can be two
possibilities either optimal solution exist or does not exist. If If exists, the solution may be degenerate or
non-degenerate.
Example :
Maximize P = 5x1 + 4x2
Subject to
x1 + 4x2 < 28
3x1 + 2x2 < 24
4x1 + x2 < 32
x1, x2 > 0
Solution : By drawing a grapy, we find that the lines for 3x1 + 2x2< 24 land 4x1 + x2 < 32 intersect at
x-axis where 4x1+ x2 < 32 is redundant, But optimal solution is at point H (4, 6) where P = Rs. 44,
Example 1.
Consider the problem Solving the equations
3x2 + 5x2 < 15 3x1 + 5x2 = 15
5x1+2x2 < 10 5x1+2x2 = 10
x1 = x2 > 0 ----------------
Max Z = 5x1+3x2 we get
x1.....
x3 = 2.4
Draw the boundary lines 3x1 + 5x2 = 15, 5x1 + 2x2 = 10 determine the convex region of feasible
solutions.
It is clear that, all the points of the shaded region constitute the area of feasible solution. But which
point would maximise the objective functions? This we have to see. The co0ordinatesof the four vertices
O, A, B, and C are (0, 0), (2, 0), (1, 2.4), (0.3). The value of the objective function at these points are 0,
10, 12.2 and 9. Clearly the maximum value is 12.2 and it occurs at the point B i.e., (1, 2.4).
2. Maximise the function 8x + 4y + 4 given that x  0, y  0, x + y – 2  0, 6x + 7y  42, 6 – x  0
and 5 – y  0.
The region of feasible solution is the shaded area shown.
The value of the objective function at the corners (2, 0), (6, 0), (6, 1), (2, 5) and (0, 2) are
respectively 20, 52, 58, 45 and 16.
Thus the maximum value 58 occurs at (6, 1).
3. Maximise the function 9x + 4y given that x  1, y  2 and 2x + y  8, x  1 is the region to the
right side of x – 1, y  2 is the region above the line y = 2.
Clearly to the left of the line 2x + y = 8, value of 2x + y is less than 8.
Thus the triangular area shown shaded is the area of the feasible solutions.
The corners are B (1, 2), C (3, 2), A (1, 6).
The value of the objective function are, 17, 35 and 33.
Thus the maximum value occurs at C (3, 2) and it is 33.
Exercise: Solve graphically:
Min Z = 3x1 + 5x2
–3x1 + 4x2  12
2x1 – x2  –2
2x1 + 3x2  12
x1  4, x3  2;
x1 , x2  0
Clearly the shaded region is the region of feasible solutions. The optimum value will occur at a
corner. Since, we have to minimise the objective function. We have to move the line
3x1 + 5x2 = K
Parallel to itself and find out where it is minimum. K is proportional to the distance from the origin
 k 
  and so the minimum value occurs at A, A is the point (3, 2).
 34 
Hence the solution is
x1 = 3 x2 = 2
and the minimum value = 9 + 10 = 19
x – 2y + z = 0
 1 1 1  x   45
or  1 0 1  y  =  8 
     
 1 2 1  z   0 
LESSON – 7

LINEAR PROGRAMMING (SIMPLEX METHOD)


An another mathematical technique available for determining the optimal solution of allocational
problems is the simplex method. It is just a mathematical algorithm for finding the optimal solution. This
method can be applied even if the number of variables is more than two. This technique is more logical
also. It can be used for stating mathematical conditions for different peculiar conditions such as
degenerate solution, multiple solutions, unbounded solution etc.
This method can be generalised for various allocation problems appearing in the forms of allocation
of jobs, transportation models.
It can be applied in all the conditions of linear programming problems such as maximizing the
objective function with less than or equal to type constraints, minimization with greater than or equal to
type constraints, maximization with mix type of constraints, minimisation with mix type of constraints. In
general, the algorithm required in each case in same.
All these requires (i) formation of proble, involving determination of objective function subject to
constraints, (ii) obtaining the initial basic feasible solutions, (iii) testing whether basic feasible solution so
obtained is optimal or not, (iv) improving the solution if not optimal, (v) testing again and improving till
solution is not optimal. But there are some differences in each situation. So these together with the related
concepts are explained below:

MAXIMIZING THE OBJECTIVE FUNCTION WITH LESS THAN OR EQUAL TO TYPE


CONSTRAINTS
In this type of linear programming, the following are the steps—
(i) Formulating the problem: It involves the determination of objective function to be optimized
with inequality constraints.
Example. A firm produces two products A and B requiring 3 units of iron and 2 units of coal and 2
units of iron and 3 units of coal. The selling price of A is Rs. 2.50 and of B is Rs. 2. If the firm has at the
most 25 units of iron and 30 units of coal. Formulate the problem assuming firm produces x1 of A and x2
of B.
Solution. If firm produces x1 of A and x2 of B, the objective function is to maximize total sales.
Maximize S = 2.50x1 + 2x2
subject to 2x1 + 3x2  25 (iron availability)
3x1 + 2x2  30 (coal availability)
x1, x2  0.
(ii) Converting the inequality constraints in equality constrains: Since in this type of problem, the
constraints are of less than or equal to type constraints, left hand side is less than or equal to constant
value. Each of these structural constraints is converted into equality constraint by introducing a different
variable in each constraint. Such variables are known as slack variables. These represents the unutilized
capacity. So one slack variable is added in each constraint. The number of slack variables to be
introduced is equal to number of constraints in each case. Such variables assume non-negativity
restrictions. It means if there are m constraints in the problem with n variables, then total number of
variables after introducing stock variables will be (n + m) variables. Since these are just imaginary
variables only, their objective coefficients are taken as zero. In the above example, if x3 and x1 are
introduced as the slack variables in first and second constraints, the new problem with modified
objectives function will be:
Maximize S = 2.50x1 + 2x2 + Ox3 + Ox4
subject to 2x1 + 3x2 + x3 = 25
3x1 + 2x2 + x4 = 30
–x1, x2, x3, x4  0.
In this x1 and x2 are primary variables and x3 and x4 are slack variables.
The introduction of such variables form an identity matrix in the problem as in the above example it
forms a two by two identity matrix. It provides a ready basic feasible solution as explained in the next
step. The variables corresponding to this matrix are known as basic variables. So basic variables are those
whose all the entitles are zero except unit in one. All other variables are known as non-basic variables. So
in the above example x1, and x2 are non basic variables whereas x3 and x4 are basic variables.
(iii) Obtaining the basic feasible solution: After converting the inequality constraints in equality
constraints, the next step is to obtain the initial basic feasible solution.
As stated earlier, all the linear programming problems are such that number of constraints is less
than or equal to number of variables in the problem. For instance, in the previous example, there are 4
variables with two equations which is an undetermined situation with no unique solution or infinite
number of solutions. But if all the primary variables are put equal to zero, then value of remaining slack
variables can be determined as in the above example if x1 = 0, x2 = 0, then x3 = 25 and x4 = 30. Actually if
in a generalised linear programming problem having m constraints and (m + n) variables, if any of the n
variables (ignoring difference of primary and slack variables) are set equal to zero. Then it will give n + m
solution of which some are feasible, others are not feasible. In the previous example, if any of the two
variables are put equal to zero, then 4c2, i.e., 6 solutions are possible which are as follow:

Variable set equal Value of other Type of solution


to zero variable
x1 , x3 x3 = 25, x4 = 30 Feasible
x1 , x3 x2 = 25/3, x4 40/3 Feasible
x1 , x4 x2 = 15, x3 – 20 Not feasible
x2 , x3 x1 = 25/2, x4 – 15/4 Not feasible
x2 , x4 x1 = 10, x3 = 5 Feasible
x3 , x4 x1 = 8, x2 = 3 Feasible

In the above table, all the feasible solutions are known as basic feasible solutions. So any solution,
with exactly m positive variables (equal to number of constraints) and satisfying both the linear structural
constraints and non-negativity restrictions, the solution is known as basic feasible solution. It may be that
a solution contains more than m positive variables satisfying both the constraints, or less than m positive
variables satisfying both the constraints, the solution is known as feasible in the first condition and
degenerate solution in the second conditions. (For details, see the peculiar conditions at the end of this
chapter).
So in case number of variables exceeds number of equations, a solution can be obtained by setting
all the excess variables as equal to zero and solving the remaining equations. The same mathematical
technique is applied in the simple method for finding a basic feasible solution is this way: put all the non-
basic variables as equal to zero and solve for basic-variables.
The approach in the simplex method is to obtain the different basic feasible solutions being possible
and improving each solution. Fill it is not optional. It needs preparation of a tableau for each basic
feasible solution. The tableau for this may be prepared in different forms, the form used here is as follow
which gives the initial basic feasible solutions:

Tableau 1

ct xt x1 x2 x3 x4 bt bi/xi

0 x3 2 3 1 0 25 25/2

0 x4 3 2 0 1 30 30/3

ct 2.50 2 0 0 0=0

ci – z i 2.50 2 0 0

In this tableau 1 :
(a) xi and ci denote the basic variables and their objective function coefficient respectively. So in this
tableau x3 and x4 are basic variable whose objective coefficients are zero.
(b) x1, x2, x3 and x4 denote the columnar vectors each stating the coefficient of these variables in the
constraints. If a variable is not appearing in any constraint, the coefficient is implied as zero.
(c) It denotes the constraint of each constraint which in turn is also the value of basic variables
giving the initial basic feasible solution, i.e., x3 = 25, x4 = 30.
(d) ci denotes the objective co-efficient of each variable. It will be zero for all the slack variables as
stated in the modified objective function.
(c) S = 0 is the value of objective function which is given by
S 0(25) + 0(3) = 0 or c x i i

(f) ci – zi is the net evaluation row stating the loss of profit by not producing a unit of each product
as for x1 = 2.50 states that profit is reduced by Rs. 2.50, per unit ... production is not increased. In this zi =
m

c x
i 1
i i

So z1 = 0(2) + 0(3) = 0 Correspondingly c1 – z1 = 2.50 – 0 = 2.5


z2 = 0(3) + 0(2) = 0 ” c 2 – z2 = 2 – 0 = 2
z3 = 0(1) + 0(0) = 0 ” c 3 – z3 = 2 – 0 = 2
z4 = 0(0) + 0(1) = 0 ” c 4 – z4 = 2 – 0 = 2
(iv) Fasting the initial basic feasible solution : After obtaining the initial basic feasible solution
where x3 = 25, x4 = 30 and S = 0, the next step is to test whether it is optimal or not. It required the row ci
– zi are negative or zero. If any of the ci – zi is positive, the basic feasible solution is not optimal and can
be improved further.
(v) Improvement of basic feasible solution : If according to fourth step, a basic feasible solution is
not optimal, then it can be further improved. It involves the replacement of one of the existing basic
variables by one non-basic variable (as the total number of basic variables must remain same).
(a) It requires the determination of outgoing basic variable and incoming non-basic variable. The
variable which has the highest opportunity profit loss, i.e., the most positive ci – zi is the entering variable.
The column corresponding to it is known as key column or point column. It is x1 is tableau 1 as shown in
Tableau where ci – zi is most positive, i.e., 2.50. It is followed by dividing the constant of each
constraints, i.e., bi by corresponding variable coefficient of key column, i.e., xi in a separate column
named as bi/xi on the right of the tableau 1. This gives bi/xi being equal to 25/2 for first row and 30/3 for
second row.
The least positive of these denotes the outgoing basic variable representing key row or first row. It is
always taken as least positive with a view to satisfy constraints. The element where key row and key
column intersect is known as key element, i.e., 3 in the Tableau 1.
(b) Replacing the incoming variable in place of outgoing variable : It involves making the entering
variable as basic and outgoing as the non-basic. It is known as pivoting. It involves Gaussian elimination
method (As such, it has been ignored, and as simplex method is applied). It can be obtained in the way
shown in Tableau 2.
1. Divide all the elements of key row by key element including bi, i.e., 3/3, 2/3, 0/3, 1/3 and 30/3.
2. For all other elements of the xi and bi, the modified value is given by subtracting the product of
corresponding key row and key element being divided by key element from the corresponding element. In
other words:
corresponding key row × corresponding key column
New element = Old element
key element
For example in Tableau 2 :
3 2
New value of x1 = 2 – =0
3
2  30
b1 = 25 – =5
3
This transformation results in shifting of identity matrix to x1 instead of x4 resulting in x1 and x3 will
be basic variables in Tableau 2 instead of x3 and x4.

Tableau 2

ct xt x1 x2 x3 x4 bt b1/x1

0 x2 0 5/3 1 –2/3 5 3

2.50 x1 1 2/3 0 1/3 10 15


ct 2.50 2 0 0 p = 25

ct – z t 0 1/3 0 –5/6

(vi) Testing the new basic feasible solution: This tableau 2 gives an another basic feasible solution
as x1 = 10, x3 = 5 where P = 2.50 (10) + 0(5) = 25. It is followed by testing this basic feasible solution in
the same way as explained in step (iv). Apply it again as shown in Table 2, one of ci – zi, i.e., 1/3 is again
positive so this basic feasible solution is also not optimal.
(vii) Improving the new basic feasible solution: If the new basic feasible solution is also not optimal,
improve this solution by applying step (v) As shown in Table 2, x3 is the entering variable being most
positive ci – zi and x3 is the outgoing variable having least positive bi/xi, i.e., 3.

Tableau 3

ct xt x1 x2 x3 x4 bt b1/x1

2 x2 0 1 3/5 –2/5 3

2.50 x 1 0 –2/5 9/15 8

ct 2.50 2 0 0 p = 26

ct – z t 0 0 –1/5 –213/15 –

According to this table 3, the new basic feasible solution is x1 = 8, x2 = 3, x3 = 0, x4 = 0 and value of
objective function P = 8(2.50) + 3(2) + 0(0) + 0(0) = Rs. 26.
Testing this new basic feasible solution shows that all the ct – zt are negative or zero. So this solution
is optimal and optimized value of objective function is Rs. 26.0.
So some of the important features of simplex method are as follows:
1. Each tableau gives a basic feasible solution with m basic variables and n non-basic variables.
Each successive tableau involve the replacement of a non-basic variable by a basic are giving a
different basic feasible solution.
3. Each new basic feasible column often improves the value of objective function.
4. Usually optional solution is obtained after introducing all the primary variables as basic variables.

MINIMISATION WITH GREATER THAN OR EQUALITY TYPE OF INEQUALITY


CONSTRAINTS
In the linear programming problems where optimisation involves minimisation of objective function
as in case of objective function is in terms of cost, loss, input requirement etc; the inequality structural
constraints usually appears in the form of greater than or equal to type. If the linear programming is of
this type, the simplex method can be applied for obtaining optimal solution with some modifications. All
the steps with necessary modifications are as follows:
(i) Formulation of problem: It is same as in the case of maximisation. It involves the determination
of objective function with inequality constraints.
Example. Let of Mr X takes x1 pounds of food ‘A’ and x2 pounds of food ‘B’, then his objective
function with restriction will be:
Minimise C = 1.50x1 + 0.70x2
Subject to 3x1 + 5x1  180 (protein)
4x1 + 6x2  220 (carbohydrates)
x1, x2  0.
(ii) Converting the inequality constraints in equality constraints: It differs from the method in case
of maximisation. In this case, left hand side is already greater than or equal to right hand side., So a
surplus variable is introduced in each constraint for changing into equality constraints. So surplus variable
are those variables which are introduced for converting the greater than or equal to type constraints. These
are subtracted from each constraint. Since these surplus variables do not represent any real quantities,
their objective coefficients are taken as servo. In the above example if x2 and x4 are introduced as surplus
variables in first and second constraints will be:
Minimise C = 1.50x1 + 0.70x2 + 0x3 + 0x4
Subject to 3x1 + 5x2 – x3 = 180
4x1 + 6x2 – x4 = 220
x1, x2, x3, x4  0.
But these variables do not form identity matrix resulting in that it will not give a direct basic feasible
solution. In addition to this, if the initial basic feasible solution is obtained by putting x1 = 0 and x2 = 0,
then, x3 = –180 and x4 = –220 which is not possible as it violates the non-negativity restriction.
To avoid this one more variable is added in each constraint. Such variables are known as artificial
variables. Such artificial variables form identity matrix which provides a direct basic feasible solution.
The objective function coefficient of such artificial variables is taken as very large as M (plus M in case
of minimmisation and minus M in case of maximisation). It is known as big-M method or method of
penalty. So introducing x5 and x6 as artificial variables in first and second constraint, the new problem will
be:
Minimise C = 1.5x1 + 0.70x2 + 0x3 + 0x4 + Mx5 + Mx6
Subject to: 3x1 + 5x2 – x3 + x5 = 180
4x1 + 6x2 – x4 + x6 = 220
x1, x2, x3, x4, x5, x6  0.
In this x1, x2, x3 and x4 are non-basic variable whereas x5 and x6 are basic variables.
(iii) Obtaining the initial basic feasible solution: It is same in the case of maximisation, i.e., set all
the non-basic variables as equal to zero and solve for basic variables. So Tableau 4 gives the initial basic
feasible solution:

Tableau 4
ct xt x1 x2 x3 x4 x5 x6 bt b1/x1

m x5 3 5 –1 0 1 0 180 180/5

m x6 4 6 0 –1 0 1 220 220/6

ct 1.50 .70 0 0 m m c=
400m

ci – zi 1.50–7m .70–11m m m 0 0

So it shows that x3 = 180 and x6 = 210 where c = 180 (M) + 220 (M) = 400 M which is very large.
(iv) Texting the basic feasible solution: After obtaining basic feasible solution, next is testing of
optional solution whether it is optimal or not. It involves determining ci – zi for each variables in the same
way as explained in case of maximisation. In this case each ci – zi represents the opportunity cost change
of including each variable. So if all the ci – zi are positive or zero, the value of objective function can not
be reduced further, so that basic fesible is also optimal. But if any of the ci – zi is negative, then value of
objective function can be reduced further, so that basic feasible is not optimal solution. So in the Tableau
4, the basic feasible solution is not optimal as ci – zi. For x1 and x2 are negative.
(v) Improving the basic feasible solution: If the basic feasible solution is not optimal as stated in the
previous step, it can be improved by replacing a basic variable by a non-basic variables. The step
involved are as follows:
(a) Determining key column and key row: The variable which has most negative ci – zi, i.e., highest
opportunity cost reduction possibility is the key column and entering variable. The key row will
be this which has least positive ratio of bi/xi (as in the case of maximisation). This will give
outgoing variable. In the Tableau 4, x2 is the key column as (0.7, 11M) is the most negative, so it
is the entering variable x5 is the outgoing variable as bi/xi is least positive i.e., 36 in this case. So
x5 is the outgoing variable as shown in Tableau 4.
(b) Replacing this basic variable by a non-basic variable: This involves obtaining modified value
given by the same method as explained in the case of maximisation. It is given in Tableau 5.

Tableau 5

ct xt x1 x2 x3 x4 x5 x6 bt b1/x1

0.70 x2 3/5 1 –1/5 0 1/5 0 36

m x6 2/5 0 6/5 –1 –6/5 1 4


ct 1.50 0.70 0 0 m m c = 25.2
+ 4m

ci – zi 1.08–.4m 0 .14–1.2m m 11/5m–.14 0

(vi) Testing the new basic feasible solution: Applying the step (iv) again, the new basic feasible
solution, x1 = 0, x2 = 36, x3 = 0, x4 = 0, x5 = 0 and x6 = 4 where c = 25.2 + 4M is again not optimal as ci –
zi for x1 as well as x5 are negative.
(vii) Improving the new basic feasible solution: Since the basic feasible solution is not optimal,
improving it by steps (v), the new tableau 6 is as follow:

Tableau 6

ct xt x1 x2 x3 x4 x5 x6 bt b1/x1

0.70 x2 2/3 1 0 –1/6 0 1/6 200/6

0 x3 1/3 0 1 –5/6 –1 5/6 20/6

ct 1.50 .70 0 0 m m

ci – z i 1.36 0 0 .11 m m–.11

This new basic feasible solution, i.e., x1 = 0, x2 = 220/6, x3 = 20/6, x4 = 0, x5 = 0, x6 = 0 is optimal as


all ci – zi over positive or zero. So x1 = 0, x2 = 220/6 is optimal where c = Rs. 25.60.
Duality : Yet another important aspect of linear programming is the theory of duality. In fact all the
linear programming problems have two related problems. One problem is known as primal problem and
other is known as dual of the primal. Generally the original problem is known as primal and the problem
formulated from the original is known as dual.
Dual is a derived problem. The basic difference between the two is that if primal problem involves
the maximisation of objective function than its dual will minimise the objective function and vice-versa.
Both are inverse of each other. The primal problem emphasises one aspect then the dual solves another.
Let us take an example—
Example. A firm produces two product A and B requiring processing through machine 1 and 2, one
unit of product A requires 10 hours one Machine 1 and 5 hours on machine 2 in addition to one man hour.
Whereas one unit of product B requires 6 hours on machine 1 and 10 hours on machine 2 together within
2 man hours. If the firm earns a profit of Rs. 23 per unit of product A and Rs. 32 per unit of product B.
The capacity available 2500 hours on machine 1 and 2000 hours on machine 2 and 500 man hours.
Formulate the problem stating objective function with constraints.
Solution: Maximize P = 23x1 + 32x2
Subject to 10x1 + 6x2  2500 (machine 1)
5x1 + 10x2  2000 (machine 2)
x1 + 2x2  500 (men hours)
x1, x2, x3  0
This problem is known as primal problem involving maximising of objective function subject to
“less than of equal to” type inequality constraints.
If we are interested in knowing how we should price one hour on machine 1 and 2 and man hour
than it requires another formulation of problem from the given problem. The objective will be to
minimise the price keeping in view the given constraints so that the total value of resources sequired for
producing a unit of each product is greater than or equal to the price of each product. Therefore two
important considerations should be taken care of:
(i) Determining the minimum price of each resource
(ii) The sum of the value of resources required for producing a unit of each product is greater than
or equal to the price of each product. For example in the above stated problem, one unit of
product A requires 10 hours on machine 1 and 5 hours on machine 2 and one man hour, their
value taken together must be greater than or equal to Rs. 23. Similarly one unit of product B
requires 6 hours on machine 1, 10 hours on machine 2 and 2 man hours. So the total value of
these must be greater than or equal to Rs. 32.
Let us assume price of one hour on machine 1 is Rs. y1, on machine 2, Rs. y2 and of man-hour, Rs.
y3 .
The problem is stated as follows:
Minimise C = 2500y1 + 2000y2 + 500y3
Subject to 10y1 + 5y2 + y  23 (Product A)
6y1 + 10y2 + 2y3  32 (Product B)
The non-negativity conditions remain unchanged because each is representing price of resources.
y1, y2, y3  0
The optimisation of this dual will give the minimum price of each resource known as shadow price
or marginal worth., This derived problem is known as dual of the primal problem.
The basic features of dual are as follows:
(i) If the primal problem’s objective is to maximise, its dual’s objective will be to minimise and
vice-versa.
(ii) The objective coefficients of the primal appears on the right hand side of the inequality
constraints becomes the constraints of dual.
(iii) The constraints of the primal appears on the right hand side becomes the coefficients of the
objective function of dual.
(iv) The sign of equality of constraints changes from primal to dual.
 becomes 
and  becomes 
(v) The input output-coefficients of the dual are transpose of the input-output coefficients of the
primal.
The solution given by dual has a number of managerial applications. As in the above stated problem,
the management will be able to know at which price it should acquire more resources in the given price
condition of the products. Not only this it will enable the management in determining the minimum price
at which it may rent out the excess capacity. Moreover, dual often simplifies the linear programming
problem by reducing the number of constraints so that the tables required to solve the problem will be
less.
Example. The manager of a company which applies office furniture, has asked you to prepare a
profit maximising schedule for their production of desks. This company sells a basic line of 4 desks (Type
A, B, C and D) to local distributors at the price given below:
Desk Type Selling Price Production Cost
(Rs.) (Rs.)
A 32 20
B 50 30
C 56 38
D 74 34
The desks production is a two step-process, each desk is first constructed in the carpentry shop and
is next sent to the flashing shop, where it is varnished, waxed and polished. The number of man hours of
labour required in each shop is as follows:
Desk Type Selling Price Production Cost
A 4 1
B 9 1
C 7 3
D 10 40
Because of the limitations in capacity of the plant, no more than 600 man hours can be expected in
the carpentry shop and 4000 in the finishing shop in a month.
(i) Build up a line programming model to the problem.
(ii) Write down its dual. Do not solve it.
Solution. Let the company produces x1, x2, x3 and x4 desks of type A, B, C and D respectively. It is
given that profit per desk of type A, B, C and D are Rs. 12, Rs. 20, Rs. 18 and Rs. 40 as profit is selling
price minus production costs.
The company objective function will be as follow:
Maximize P = 12x1 + 20x2 + 18x3 + 40x4
Subject to 4x1 + 9x2 + 7x3 + 10x4  6000
x1 + x2 + 3x3 + 40x4  4000
x1, x2, x3, x4  0
(ii) Let the price of one hour of carpentry is Rs. y1 and of finishing is Rs. y2, then the objective
function with constraints becomes:
Minimize C = 6000y1 + 4000y
Subject to 4y1 + y2  12 (Type A)
9y2 + y2  20 (Type B)
7y1 + 3y2  18 (Type C)
10y1 + 40y2  40 (Type D)
x1 , x2  0
This is dual of the primal problem.

RELATIONSHIP BETWEEN OPTIMAL SOLUTION OF PRIMAL AND DUAL


There is a close relationship between solution of primal and dual and its dual problems as stated
earlier. The tableau which gives optimal solution to the primal problem also gives optimal solution to the
dual and vice-versa. The net evaluation row elements ci – zi of the surplus/slack variables of the primal
gives the optimal solution of the dual. If the net evaluation row elements corresponding to the variables is
negative, their negative gives the optimal solution otherwise the elements as such as the optimal solutions.
Foe instance, the final iterations primal given in tableau 1, show that ci – zi corresponding to surplus
variables x3, x4, x5 are –1, –13/5, and 0. So y1 = 1, y2 = 13/5 is the optimal solution of the dual.

Tableau 7

ct xt x1 x2 x3 x4 x5 bt

23 x1 1 0 1/7 –3/35 0 1300/7

32 x2 0 1 –1/14 1/7 0 750/7

0 x5 0 0 0 –1/5 1 100

ct 23 32 0 0 p = 7700

ci – z i 0 0 –1 –13/5 0

Similarly in net evaluation row elements corresponding to the slack/surplus variables the final
tableau of dual gives the optimal solution of the primal. As shown in tableau 2, ci – zi, corresponding to
slack variables. y4, y5 are 1300/7 and 750/3 which is the optimal solution of the primal i.e., x1 = 1300/7, x2
= 750/7. Similarly in net evaluation row element corresponding to prime variables of the primal gives the
value of slack/surplus variables of the dual. As in tableau 1, ci – zi, corresponding to primary variables x1,
x2 are zero. Therefore the stock variables of the dual y2 and y4 are also equal to zero. In the tableau 2 in ci
– zi corresponding to primary variables y1, y2 and y3 are 0, 0 and 100 respectively, hence the value of
surplus variables x3, x4 and x5 of the primal are 0, 0 and 100 respectively.
All though both these optimal solution have altogether different interpretations, but optimized value
of the objective functions of both primal and dual is equal to Rs. 7,700.
Tableau 8

ct yt y1 y2 y3 y4 y5 y6 y7 bt
2500 y1 1 0 0 –1/7 1/4 1/7 –1/14 1

2000 y2 0 1 1/5 3/35 –1/7 –3/35 1/7 13/5

ct 2500 2000 500 0 0 m m c=


Rs.7700

1300 750 1300 750


ci – z i 0 0 100 m– m–
7 7 7 7

So the important relationship between the solution of the primal and dual are as follows:
(i) The optimized value of the objective is same both for the primal and dual.
(ii) The net evaluation row elements of the primary variables of the primal is the optimized value of
the surplus/slack variables of the dual.
(iii) The net evaluation row elements of the primary variables of the dual is the optimized value of
the slack/surplus variables of the primal.
(iv) The net evaluation row elements of the slack/surplus variables of the primal gives the optimized
of the solution dual.
(v) The net evaluation row elements of the surplus/slack variables of the dual gives the optimizing
solution of the primal.
Example 1. 3x1 + 5x2  15
5x1 + 2x1  10
x1 , x2  0
Maximize Z = 5x1 + 3x2. Introduce slack and surplus variables. Then this will read
3x1 + 5x2 + x3 = 15
5x1 + 2x2 + x4 = 10
Y1, Y2....etc., will be used to denote the 1st, 2nd, 3rd....etc., columns, b denotes column on the right.

CB Basis Y1 Y2 Y3 Y4 b

0 x3 3 5 1 0 15

0 x4 5 2 0 1 10

Ci 5 3 0 0

The optimal function Z can be written as


Z = 5x + 3y + 0x3 + 0x4
The coefficients of the new variables x3 and x4 are written under the column CB.
Below this we write the coefficients of the variables in Z as a row.
Then we write another row labelled as i.
i = Ci – Zi where Zi = CB . Yi

CB Basis Y1 Y2 Y3 Y4 b

15
0 x3 3 5 1 0 15 =5
3
10
0 x4 5 2 0 1 10 =2
5
Ci 5 3 0 0

Zi 0 0 0 0
 i 5 3 0 0
 

Now in the row listed as i, 5 is the largest. Hence x1 is the entering variable. In the column x y1
occur 3 and 5 we divide the elements of b by the corresponding elements of Y1. Then we get 5 and 2
written as the last column. Among these find the minimum. It is 2. So x4 is the departing variable.
The key element is 5. We make the transformations
R2  R2/5
and then R1  R1 – 3R3
Doing this we get
CB Basis Y1 Y2 Y3 Y4 b

0 x3 0 3.8 1 –0.6 9
5 x1 1 0.4 0 0.2 2

Ci 5 3 0 0
Zi 5 2.0 0 1.0

i 0 1 0 –1

 

Here 1 is the largest so x2 is the entering variable.


Dividing the last column elements by the elements of y2, and taking maximum, we find x2 is the
departing variable.
So, we reduce the column,
 3.8  1
  to   by the row transformation, R1  R1/3.8 and then R2  R3 – 0.4 R1
 0.4  0
Doing these we get
CB Basis Y1 Y2 Y3 Y4 b

3 x2 0 1 .26 –.16 2.368


5 x1 1 0 –105 0.26 1.05

Ci 5 3 0 0
Zi 5 3 .25 .82
i 0 0 –.25 –.82

Now the last row i is –ve for all columns. Consequently we have obtained an optimal solution. The
solution is
x1 = 1.05 and x2 = 2.368
 Max Z = 5 × (1.05) + 3 × (2.368)
= 12.315
Example 3. Max Z = 3x1 + 5x2 + 4x3 such that
2x1 + 3x2  8
3x1 + 2x2 + 4x3  15
2x2 + 5x2  10
x1, x2, x3  0
Adding slack and surplus variables the problem can be expressed as
2x1 + 3x2 + 0x3 + x4 + 0x5 + 0x6 = 8
3x1 + 2x2 + 4x3 + 0x4 + 0x5 + 0x6 = 15
0x1 + 2x2 + 5x3 + 0x4 + 0x5 + x6 = 10

CB XB Y1 Y2 Y3 Y4 Y5 Y6 b
0 x4 2 3 0 1 0 0 8

0 x5 3 2 4 0 1 0 15

0 x6 0 2 5 0 0 1 10

Ci 3 5 4 0 0 0

Zi 0 0 0 0 0 0

i = 3 5 4 0 0 0
Ci – Zi  

Entering Departing
variable variable
 8 15 10  8
Min  , ,  =
3 2 2  3
[By dividing the elements of column b by the elements of column Y3]
Hence x4 is the departing variable.
Now by row, transformations reduce
3 1  Perform: R1  R1 / 3
   
 2  to  0  and then perform  R3  R2  2 R1
 2 0
     R3  R2  2 R1
We get,
CB XB Y1 Y2 Y3 Y4 Y5 Y6 b
5 x2 2/3 1 0 1/3 0 0 8/3
0 x5 5/3 0 4 –2/3 1 0 29/3
0 x6 –4/3 0 5 –2/3 0 1 14/3

Ci 3 5 4 0 0 0
Zi 10/3 5 0 5/3 0 0
i = –1/3 0 4 –5/3 0 0

Ci – Zi    

Here 4 is the largest. Hence x3 is the entering variable.


 29 14  14
Min  4, 5 
3 3  15
This occurs in the 3rd row. Hence x4 is the departing variable. We reduce
0 0
   
 4  to  0  by the row transformation.
5 1 
   
R3  R3/5
and then perform R2  R3 – 4R3
Performing these transformations we get the third simplex table.
CB XB Y1 Y2 Y3 Y4 Y5 Y6 b
5 x2 2/3 1 0 1/3 0 0 8/3
0 x5 41/15 0 0 –2/15 1 –4/5 89/15

4 x3 –4/15 0 1 –2/15 0 1/5 14/15

Ci 3 5 4 0 0 0

Zi 34/15 5 4 17/15 0 4/5

Ci – Zi 11/15 0 0 –17/15 0 –4/5

   

11
Here is the largest. Hence x1 is the entering variable
15
 8 2 89 41 14  4  
Min   ,  ,    
 3 3 15 15 15  15  
 89 7  89
4, , 
 41 2  41
[ in considering the minimum the negative element is not to be taken.]
This occurs in the 3rd now. Hence x5 is the departing variable. We reduce
 2
 3
  0
 41  to 1  by the row transformation
 15   
  0
4  
 
 
 15 
15
R2  R
41 2
2
Then R1  R1 – R2
3
4
and R2  R3 + R.
15 2
Performing these transformations we get the simplex table.

STEP IV
CB XB Y1 Y2 Y3 Y4 Y5 Y6 b
15 10 8 50
5 x2 0 1 0 –
41 41 41 41
2 15 12 89
3 x1 1 0 0 – –
41 41 41 41
6 4 5 62
4 x3 0 0 1 –
41 41 41 41
Ci 3 5 4 0 0 0
45 11 24
Zi 3 5 4
41 41 41
45 11 24
i = Ci–Zi 0 0 0 – – –
41 41 41

Since all (Ci – Zi  0) the solution is optimal.


Thus
89 50 62
x1 = , x2 = , x3 = x , 0, x5 = 0
41 41 41 4
The maximum value is
89 50 62 785
+5
3× +4 =
41 41 41 41
Example 3. Maximise = 3x1 + 2x2 subject to
x1 + x2  4
x1 – x2  4
x1 , x2  4
Converting the inequalities into equations,
x1 + x2 + x3 = 4
x1 – x2 + x4 = 2
 x1 
 
1 1 1 0   x2   4
    =  
1 1 0 1  x3  2
 
 x4 

CB XB Y1 Y2 Y3 Y4 b

0 x3 1 1 1 0 4

0 x4 1 –1 0 1 2

Ci 3 2 0 0

Zi 0 0 0 0

i = 3 2 0 0

Ci – Zi  
The respective Zi are obtained by multiplying the elements of the column CB and the columns of Y.
Among the values of Ci – Zi we find 3 is the maximum. It occurs in the first column. So Y1 is the entering
vector. We mark the column Y1 by an upward arrow. Now we have to find the outgoing vector.
4 2
Divide the elements of the column b by the element of Y1 and determine the minimum. Min  , 
1 1 
= 2. This occurs in the 2nd row. So x is the departing variable. The variable Y4 is the departing vector.
This is marked by a downward arrow.
Now we should reduce Y1 to the form of Y4, i.e.,
 1 0
  to  
 1 1 
by means of row transformation.
Perform, the transformation R1  R1 – R2.
We get

CB XB Y1 Y2 Y3 Y4 b

0 x2 0 2 1 –1 2

3 x1 1 –1 0 1 2

Zi 3 –3 0 3

Ci 3 2 0 0

j 0 5 0 –3

  

2 2 
Dividing the elements of b we get  , 
 2 1
i.e., {1, –2}. But in considering the minimum negative elements will not be taken. So 1 is to be
taken. It occurs in the 1st row. So x3 is the departing variable, i.e., Y3 is the departing vector.
 2 1 
We must therefore reduce Y2   to the form 0  by row transformations
 1  
R1
Perform R1 
2
and then perform R2  R2 + R1
Now we get

CB XB Y1 Y2 Y3 Y4 b
1 1
2 x2 0 2 – 1
2 2

1 1
3 x1 1 0 3
2 2

5 1
Zi 3 2
2 2

Ci 3 2 0 0

5 1
j 0 0 – –
2 2

Now in the last row j all the element are –ve, consequently we have obtained the optimal solution.
The solution is therefore
x1 = 3, x2 = 1, x3 = 0 and x4 = 0
Z = 3x1 + 2x2 = 3 × 3 + 2 × 1 = 11
Hence the maximum value is 11.
Example 4. Maximise Z = 3x1 + 4x2 + x3 + 7x4

subject to 8x1 + 3x2 + 4x3 + x4  7

2x1 + 6x2 + x3 + 5x4  3

x1 + 4x2 + 5x3 + 2x4  8


Introducing slack variables, x5, x6, x7 these become
3x1 + 3x2 + 4x3 + x4 + x5 + 0x6 + 0x7 = 7
2x1 + 6x2 + x3 + 5x4 + 0x5 + x6 + 0x7 = 7
x1 + 4x2 + 5x3 + 2x4 + 0x5 + 0x6 + x7 = 7
An initial basic feasible solution will be
»
XB = [x5, x6, x7] = [7, 3, 8] obtained by putting x1 = 0, x2 = 0 and x3 = 0.
The coefficients of the variables, x5, x6, x7 in the optimal function Z = 3x1 + 4x2 + x3 + 7x4 + 0x5 +
0x6 + 0x7 are [0, 0, 0].
So,

CB Basis y1 y2 y3 y4 y5 y6 y7 b

0 x5 8 3 4 1 1 0 0 7

0 x6 2 6 1 5 0 1 0 3
0 x7 1 4 5 2 0 0 1 8

Cj 3 4 1 7 0 0 0

Zj 0 0 0 0 0 0 0

j = 3 4 1 7 0 0 0
Cj – Zj  

Here in the row j, is the largest. So x4 is the entering variable. Divide the elements of col. b by
those of y4.
7 3 8  3
Min  , ,  = . This corresponds to the 2nd row.
1 5 2  5
1  0 
So x4 is the departing variable. Now we should convert y4 = 5  to the form y6 = 1  by row
 
 2  0 
operations.
R
Perform R2  2
5
then R1  R1 – R2
R3  R3 – 2R1

CB XB y1 y2 y3 y4 y5 y6 y7 b

38 9 19 1 32
0 x5 0 1 – 0
5 5 5 5 5
2 6 1 1 3
0 x4 1 0 0
5 5 5 5 5
1 8 23 2 34
0 x7 0 0 – 1
5 5 5 5 5

Cj 3 4 1 7 0 0 0
14 42 7 7
Zj 7 0 0
5 5 5 5
1 22 2 7
j – – 0 0 – 0
5 5 5 5
   

 32 38 3 2 34 1   32 3  32
Min.  , ,  = Min.  , , 34 = . So x is the departing variable. We
 5 5 5 5 5 5  38 2  48
therefore convert
38 / 5 1 
 2 / 5  into 0  by row transformations.
   
1/ 5  0 
5
R1  R1 ×
38
R2  R2 – dR1
R3  R3 – dR1

CB XB y1 y2 y3 y4 y5 y6 y7 b
9 1 5 1 16
3 x1 1 0 – 0
38 2 38 38 19
21 1 4 5
7 x4 0 0 1 – 0
19 19 19 19
59 9 1 15 126
0 x7 0 0 – – 1
38 2 38 38 19

Cj 3 4 1 7 0 0 0
321 3 1 1
Zj 3 7 0
38 2 38 38
169 1 1 1
j 0 – – 0 – – 0
38 2 38 38
Since all j = Zj – Cj are negative an optimum basis feasible solution has been attained. The solution
is
19 5
x1 = , x2 = 0, x3 = 0, x4 =
16 19
48 35 83
Max. Z = 3x1 + 4x2 + x3 + 7x4 = +0+0+ =
19 19 19
LESSON 9

COMPOUND INTEREST AND ANNUITIES


—SHYAM SUNDER
Bh a g a t S in g h Co lle g e
De lh i Un ive rs ity

Mathematics of finance deals with simple interest, compound interest, trade discount, present and
the future values of annuities, nominal and the effective rate of interest, yield to maturity etc. It is very
important as all these concepts are commonly used in the financial decision making.

SIMPLE AND COMPOUND INTEREST


Interest denotes the charge which is paid on borrowing manes and is received on lending the money.
It is based on the concept of time value of money which states that Rs. 100 of today is not equal to Rs.
100 one year after. This interest may be simple or compound.

SIMPLE INTEREST
It refers to the interest on principal only. In practice, it is applicable only when interest is paid as and
when it is due. Therefore, every year, it is calculated on principal only. For example, if interest is due
annually, it is paid annually and if it is due on six month basis, it is paid on six month basis. Simple
interest can be computed as below:
Simple Interest (S.I.) = P.r.t.
where P = principal; r = rate of interest in relation to 1, if r is in percentages it is divided by 100: t =
time period in years.
Corresponding Amount = P + S.I. = P + P.r.t. = P (1 + rt)
Note 1. Principal also denotes the present value or value in the beginning or value in the zero period
whereas Amount denotes the future value in the tth period.
Note 2. Whenever interest is paid as and when it is due, it is always assumed to be a case of simple
interest.
Example 1. A sum of Rs. 5000 is deposited by Mr. X in a finance company under quarterly interest
scheme. Find the total interest earned by him in 5 years at 12% per annum.
Solution: Interest of each quarter = Prt
12 1
= 5000 ×  = Rs. 150
100 4
Total interest earned = Rs. 150 × 5 × 4 = Rs. 3000
The main feature of the simple interest is that the total interest for the given period is always same
whether paid annually, six monthly, quarterly or monthly.
Example 2. What time will be required for a sum of money to double itself if simple rate of interest
is 10% per annum.
Solution. Let the sum of Principal = Rs. P
 Amount = P (1 + rt)
 2P = P (1 + .10t)
 .10t = 1
 t = 10 years
Example 3. If in four years Rs. 2520 amounts to Rs. 3024, in what time at the same rate will Rs.
520 amounts to Rs. 611 at simple interest.
Solution. Let the rate of interest = 100%
 Amount = P (1 + rt)
3024 = 2520 (1 + 4r)
or r = 0.05 or r = 5%
Let the time period = 1 years
 611 = 520 (1 + .05t)
 t = 3.5 years
Example 4. What is the cash payment equivalent to Rs. 297.54 due in 3 years and 6 months, simple
interest being reckoned at 4% per annum?
Solution. Cash payment = Principal
A 297.54
 Principal = 
1  rt  7
1   .04  
 2
= Rs. 261
Example 5. What sum will discharge a debt of Rs. 5300 due after one and half year at 5% per
annum simple interest today?
Solution. Sum required to discharge the debt today = Principal
A 5300
 Principal = 
1  rt  3
1   .05  
 2
= Rs. 4930.24.
Example. In how many years at 4% per annum simple interest will the interest be one quarter of the
principal.
Solution. Let the principal = Rs. P.
1
Simple Interest = P = P (.04)t
4
1 1
 t =  6 years.
16 4

EQUATING THE DIFFERENT CASH FLOW INVOLVING SIMPLE INTEREST


The equivalent value of the cash flows at different point of time involving simple interest can also
be obtained at a point of time formally known as focal date. The equivalent value denotes a lump-sum
equal to all the flows. In such cases it is preferable to take the help of time scale. All the flows may be
denoted on the time scale (exact scale is not required). For example–Four different flow– F1, F2, F3 and F4
are there at the end of each year and their equivalent value is required at 2.5th year lying in between
second and third year, then it is given by the sum total of the value of all these four flows at 2.5th year.
So the flows which are on the left hand side of the focal date their future value is required and the
flows which are on right hand side, their present value is required. (This rough time scale enables us in
deciding whether future value of present value of the particular flow is required to arrive at the equivalent
value).
Example 7. At 5% simple interest, find the value today of the following obligations: Rs. 1000 due
today, Rs. 2000 due after 6 months with interest at 6% and Rs. 4000 due in one year with interest at 8%.
Solution. Since value is required in the beginning or in zero period and the flows are at the end of 6
months and 1 year (which are on the right hand side of focal date), present value of both the flows due at
the end of 6 months and one year is required.
Amount due at the end of 6 months = P (1 + rt)
  6 
= 2000 1  .06   
  12  
= Rs. 2060
Amount due at the end of 1 year = 4000 [1 + .08(1)]
= Rs. 4320
 The value of all the flows in the beginning at 5%
2060 4320
= 1000 + 
  1   1  .05 1 
1  .05  2  
  
= Rs. 7124.04
Same can be analysed using time scale in this way

Example 8. A man owes Rs. 500 due in 3 months with interest at 5% and Rs. 1500 due in 9 months
with interest at 4%. If money is worth 6%, what single payment made at the end of 6 months will
discharge his debts?
Solution. Let the single payment at the end of six months = Rs. X
 X = future value of the amount due at the end of 3 months
+ present value of the amount due at the end of 9
months
Amount due at the end of 3 months at 5%
  3 
= 500 1  .05    
  12  
= Rs. 506.25.
  3 
 Value of Rs. 506.24 at the end of 6 month at 6% = 506.25 1  .06   
  12  
= Rs. 513.85 ... (i)
Amount due at the end of 9 months at 4%
  9 
= 1500 1  .04   
  12  
= Rs. 1545
  3 
 The value of Rs. 1545 at the end of 6 months at 6% = 1545 1  .06   
  12  
= Rs. 1522.16 ... (ii)
 Total amount due at the end of 6 months
= Rs. 513.85 + Rs. 1522.16
= Rs. 2036.01.

Example 9. X owes Rs. 500 due in 2 months, Rs. 1000 due in 5 months and Rs. 1500 due in 8
months. He wishes to discharge his obligation in two equal payments; one in six months and other due in
10 months. Find the payment of money is worth 6% simple interest and the 10th month is taken as focal
date.
Solution. Let each payment is of Rs. x. Therefore the value of both the payments at the end of 10
months should be equal to value of all the obligations at the end of 10 months.
  4 
 x 1  .06    + x
  12  
  8    5 
= 5000 1  .06    + 1000 1  .06  12  
  12     
  2 
+ 1500 1  .06   
  12  
 2.02x = 520 + 1025 + 1515
x = Rs. 1514.85
1
Example 10. X owes Y Rs. 1000 due in 6 months without interest and Rs. 2000 with interest for 1
2
year at 6% due in 9 months. Y agrees to accept 3 equal payments; first due today, second at the end of 6
months and the third at the end of 1 year. Find the equal payments using one year from today as the focal
date if money is worth 8% to Y.
Solution. Let each payment is of Rs. A
  6 
 A 1  .081 + A 1  .08    + A
  12  
  6    3    3 
= 1000 1  .08    + 2000 1  .06    1  .08  2  
  12     2    
 1.08A + 1.04A + A = 1040 + 2223.6
 3.12A = 3263.6
A = Rs. 1046.03

COMPOUND INTEREST
It carries interest on the principal and on the interest due also. It is mainly used when interest is not
paid as and when due. Therefore, in each successive periods interest will be calculated on the principal
plus the interest due. In other words, amount of each successive period becomes the principal of next
period. Thus interest of each successive period will go on increasing. For the first period, both the simple
interest and the compound interest are equal. In this case–
Amount (A) = P (1 + r)t
where A is the amount also known as future value, P is the principal or present value, r = rate of
interest per annum in relation to 1 and t = time period in years.
Compound interest (CI) = A – P = P [1 + r]t – P
= P [(1 + r)t – 1]
A
and P = = A (1 + r)–t
1 r 
t

This process is also known as discounting.


Example 11. Mr. A deposits Rs. 10000 in a finance company under commutative interest scheme.
Company offers interest at the rate of 15% per annum for 3 years. Find the amount receivable after 3
years and the interest.
Solution. Amount receivable = P (1 + r)t
= 10,000 (1 + .15)3
= 15208.75
Interest received = 15208 – 10,000
= Rs. 5208.75
Example 12. Mr. A deposits Rs. 5000 in a bank for 3 years offering interest at the rate of 9% per
annum in the first year, 10% per annum during the second year and 11% per annum in the third year. Find
his balance after 3 years.
Solution. Balance after 3 years = 5000 (1 + .09) (1 + .10) (1 + .11)
= Rs. 6654.45
Example 13. Find the sum on which the difference between the simple interest and the compound
interest for 3 years at 5% per annum is Rs. 73.20.,
Solution. Let the sum or Principal = Rs. P.
 SI = Prt
= P × .05 × 3 = .15P
CI = P [(1 + r)t – 1]
CI – SI = 73.20
 P [(1.05)3 – 1 – 1.5 = 73.20
 P = Rs. 9600.
Example 14. The compound interest on a sum of money for 3 years at 5% is Rs. 1324.5. What is the
simple interest?
Solution. Let the principal = Rs. P.
 SI = P (.05) (3)
= .15P
CI = P[1 + .05)3 – 1]
SI .15
 = = .951625693
1.05
3
CI
 SI = .951625693 × CI
 SI = .951625693 × 1324.05
= Rs. 1260.
Example 15. The difference between 2nd year and 3rd year compound interest on a certain sum at
5% compound interest is Rs. 5.25. Find the sum.
Solution. Let the principal = Rs. P.
Third year Compound Interest = CI of 3 years – CI of 2 yers
= P[(1.05)3 – 1] – P[(1.05)2 – 1]
= P[(1.05)3 – (1.05)2]
= (1.05)2d P (.05) ... (i)
Second Year Compound Interest = P[(1.05)2 – 1] – P[(1.05) – 1]
= (1.05) P (.05) ... (ii)
 (i) – (ii) = 5.25
 (1.05)2 P (.05) – (1.05) P (.05) = 5.25
 (1.05) (.05)2P = 5.25
 P = Rs. 2000.
Example 16. A sum of money put at compound interest amounts to rs. 578.40 in 2 years and Rs.
614.55 in 3 years. Find the rate of interest.
Solution. In case of compound interest, amount of 2nd year is principal of 3rd year.
 Third year Principal = Rs. 578.40
A = Rs. 614.55
t = one year
614.55 = 578.40 (1 + r)t
 r = .0625
= 6.25%
r
In case compounding in m times in a year, r is replaced by and t is replaced by mt.
m
r
 If compounding is 6 monthly i.e. two times in a year, r is replaced by and t is replaced by 2t.
2
2t
 r
Amount = P 1  
 2
r
If compounding is quarterly i.e. 4 times in a year, r is replaced by and t is replaced by 4t.
4
4t
 r
 Amount = P 1  
 4
If compounding is monthly i.e. 12 times in a year then
12 t
 r 
Amount = P 1  
 12 
mt
 r
If compounding is m times in a year then Amount = P 1  
 m
But if compounding is continuous, number of times interest compounded in a year will go to infinity
i.e. limit of m tends towards infinity.
mt
 r
 Amount = lim P 1  
m D
 m
rt
 m

 r 
= lim P 1   

r

m D  m  
 
m
 r r
= Pert where e = lim 1  
m D
 m
Example 17. Accumulate Rs. 5000 for 6 years at 10% per annum compounded quarterly.
Solution. Given P = Rs. 5000
.10
r = , t = 4 × 6 = 24
4
24
 .10 
 Amount (A) = 5000 1   ... (i)
 4 
= 5000 (1.8087)
= Rs. 9043.63
It can also be solved using log and compound interest-tables. Compound interest table gives the
compound value of Re. 1 at different rates of interest for different periods. Using log it can be written as
log A = log 5000 + 24 log 1.025
= 3.69897 + 24 × .01072
= 3.95634
 A = Antilog (3.95634)
= Rs. 9043.63
Example 18. On March, 1, 1990, X borrowed Rs. 5000 at 12% compounded quarterly. What did he
owe on 1st Sept, 1997?
Solution. Given P = Rs. 5000
.12
r = = .03
4
t = 7 years and 2 quarters
= 30 quarters
 A = 5000 (1 + .03)30
= Rs. 12136.32.
Example 19. How long will it take the sum to double itself if compound continuously at 10% per
annum?
Solution. Let Principal = Rs. P.
 Amount = P . ert
 2P = P e.10t
 2 = e.10t
Taking log
log 2 = .10t log e
log 2 .3010
 t = 
.10log e .04343
= 6.93 years [log e = log 2.7183 = .4343]
Example 20. Mr. X deposits Rs. 5000 in a bank for 3 years. Bank offers interest at the rate of 10%
compounded quarterly during first year, at 12% per annum compounded monthly during second year and
at 15% per annum compounded continuously during 3rd year. Find his balance after 3 years.
4
 .10 
Solution. Balance after 1st year = 5000 1  
 4 
= principal for second year
4 12
 .10   .12 
Balance after second year = 5000 1   1  
 4   12 
= principal for third year

e   
4 12
 .10   .12 
 Balance after 3 years = 5000 1   1  
15 1

 4   12 
= Rs. 5519.06 (1 .1268) (1.1618)
= Rs. 7225.26

EQUATING THE DIFFERENT CASH FLOWS INVOLVING COMPOUND INTEREST


The equivalent value of different cash flows involving compound interest can also be obtained in the
same way as in case of simple interest. In this case also, the flows on the left hand side are compounded
and the flows on the right hand side are discounted, their present value is required.
Example 21. In return of a promise to pay Rs. 800 at the end of 10 years, a person agrees to pay Rs.
100 now, Rs. 200 at the end of 6 years and a final payment at the end 12 years. If the rate of interest in
10% per annum, what should the final payment be?
Solution. Let the final payment = Rs. x and the focal date be 10th year.
Therefore the future value of Rs. 100 +
Future value of Rs. 200 + = Rs. 800
Present value of Rs. x =
x
 800 = 100 (1 + .10)10 + 200(1 + .10)4 +
1  .10 
2

 800 = 259.37 + 292.82 + .826x


 x = Rs. 299.85
Note. In such cases answer remains the same irrespective of the focal date.
Example 22. The sum of Rs. 2000, Rs. 3000 and Rs. 4000 are due at the end of 2, 4 and 8 years
respectively. It is proposed to replace the series of payments by a single sum of Rs. 9000 payable at the
end of n years. If the rate of interest is 10% per annum effective. Find the value of n.
Solution. One alternative is that the present value of given cash flows should be equal to the present
value of Rs. 9000 flowing at the end of nth year.
2000 3000 4000 9000
   =
1  .10  1  .10  1  .10  1  .10 
2 4 8 n
9000
or 1652.89 + 2049.04 + 1866.03 =
1  .10 
n

9000
or (1.10)n = = 1.61639
5567.95
Using log
n log 1.10 = log 1.61639
log1.61639 .2085
n = 
log1.10 .0414
n = 5.038 years

The other alternative is that the focal date may be taken at the end of nth year. So the value of given
flows at the end of nth year should be equal to Rs. 9000.
 2000(1.10)n–2 + 3000(1.10)n–2 + 4000(1.10)n–8 = Rs. 9000
Same expression is given by multiplying both sides by (1.10)n in the first alternative. But it should
be solved carefully. Each term is to be evaluated separately using log and not be taking log of both the
sides (As log properties are not applicable in case of addition and subtraction).
Example 23. If the money is worth 5% effective, what equal payments of Rs. x at the end of first
year and at the end of third year will equitably replace the obligations: Rs. 30,000 at the end of 2nd year
with interest at 6% per annum compounded semi-annually and Rs. 50,000 at the end of 5th year with
interest at 4% compounded quarterly.
Solution. If the focal date is taken as zero period, then present value of both set of floes should be
equal.
2 2 
 .06 
Amount due at the end of 2nd year = 30,000 1  
 2 

1  .03
4

Its present value at 5% effective = 30,000


1  .05
2

= Rs. 30626.09 ... (1)


4 5 
 .04 
Amount due at the end of 5th year = 50,000 1  
 4 

1  .01
20

Its present value at 5% effective = 50,000


1  .05
5

= Rs. 47802.54 ... (ii)


The total present value of both these obligations should be equal to the present value of both the
payments of Rs. x.
x x
  = 30626.09 + 47802.54
1  .05 1  .05 3
or 1.8162x = 78428.63
or x = Rs. 43182.37
Example 24. A trader buys goods of the value of Rs. 15,000. He pays Rs. 5000 down and will pay
Rs. 5000 at the end of 4 months. If the money is worth 6% compounded monthly, what kind of payment
will be necessary at the end of 6 months.
Solution. Let the final payment = Rs. x
 the present value of all the three payments including down payment = Rs. 15,000
5000 x
 15000 = 5000 + 4
 6
 .06   .06 
1   1  
 12   12 
or 15000 = 5000 + 4901.24 + .9705x
or x = Rs. 5253.64

NOMINAL AND THE EFFECTIVE RATE OF INTEREST


Nominal rate of interest is the rate of interest per annum which may be compounded yearly, six
monthly, quarterly, monthly or m times in a year or continuously. Effective rate of interest is the rate per
annum compounded only once in a year. So both these are equal if compounding is only once a year. But
if compounding is more than once in a year, effective rate of interest will increase as the frequency of
compounding increases (at the decreasing rate). Following are the relationship between nominal (r) and
effective (R) rate of interest under two different conditions
(a) If compounding is m times in a year
mt
 r
then P(1 + R)t = P 1  
 m 
m
 r
or R = 1   – 1
 m
(b) If compounding is continuous,
then P(1 + R)t = Pert
or R = et – 1
Example 25. A money lender charges interest at the rate of 5 rupees per 100 rupees payable in
advance per quarter. What effective rate of interest does he charge per annum?
Solution. Let the money borrowed = Rs. 100
 Interest deducted in advance for the quarter = Rs. 5
So net amount received by borrower = Rs. 95
5
For borrower interest rate = per quarter
95
4
 r
 effective rate per annum will be R = 1   – 1
 4
4
 5 
= 1   – 1 = 1.2277 – 1
 95 
= .2277
= 22.77%
Example 26. Mr. X deposited Rs. 10,000 in a bank for 3 years. Bank makes two offers-either 10%
compounded quarterly or 8% compounded continuously. Which offer is preferable?
Solution. Balance after 3 years under first-offer
4 3
 .10 
= 10,000 1  
 4 
= Rs. 13448.88
Balance after 3 years under second offer
= 10,000 e.08(3)
= Rs. 12712.49
Therefore first offer is acceptable.
Alternatively effective rate of interest per annum of both the proposals may be compared, Under
4
 .10 
first offer effective rate  1   – 1 = .1038 = 10.38%
 4 
Under second offer, effective rate = e.08 – 1 = .08328 = 8.328%
 First offer is acceptable.

RELATIONSHIP BETWEEN NOMINAL RATES


The same logic as explained above can also be used for establishing relationship between two
nominal rates. For example, If interest is compounded quarterly at r1 percent and the interest is
compounded half yearly at r2 percent, then relationship between the two are given below:
4t 2t
 r  r 
P 1  1  = P 1  2 
 4  2

r1  r2 
= 1    1
4  2
  r  
r1 = 4  1  2   1
  2  

Example 27. Find the nominal rate r compounded monthly equivalent to 6% compounded semi-
annually.
12 t 2t
 r   .06 
Solution. P 1   = P 1  
 12   2 
1
r
= 1.03 6  1
12
r = .05926 = 5.926%.

Annuities
An annuity denotes amount (which may be constant or varying regularly with time) flowing after regular
intervals say monthly, quarterly or yearly. It may be inflowing or out flowing.
All annuities are broadly is known classified as annuity certain and contingent annuity. Annuity
certain is that annuity where time period is known with certainty whereas contingent annuity is that whose
time period depends upon the happening of certain event. As pension by a retired employee may be
received till the person is alive. In case of annuity certain time period may be finite or infinite. If it is
infinite, annuity is known as perpetual annuity.
These are further classified as annuity due, annuity immediate and deferred annuity. Annuity due
implies that annuity starts flowing in the beginning of first period, therefore each successive flow is in the
beginning of each period, e.g., the payment of rent in advance. Annuity immediate commonly known as
ordinary annuity denotes that annuity which starts flowing from the end of first period, so each successive
flow will be at the end of each period as the payment of loan installments. Deferred annuity denotes that
annuity which is deferred for some periods before the first flow. Therefore, if the annuity is deferred for n
periods, first-flow will be available at the end of (n + 1) period. For example, installments of a bank loan
which begins from the end of 5th year implies that it has been deferred for four years.

Present value of the Annuity


Present value of an annuity denotes the equivalent lump sum value in the beginning (in zero period)
of an annuity flowing for a given time period at the given rate of interest. In other words, it is equal to the
sum total of the discounted value of future flow. It is required mainly when flows are on the right-hand
side of time scale and equivalent value is required on the left hand side of flows. For example, Mr. X is
willing to purchase a machine costing Rs. one lakh which will generate the cash inflows of Rs. 30,000 for
the next five years when the rate of interest is 10% per annum. The same can be decided by finding the
present value of the annuity of Rs. 30,000 at 10% and comparing that with the cost of the machine.
Similarly if Mr. X has taken a loan of Rs. 50,000 from a bank charging interest at the rate of 12% per
annum. The loan is repayable in 5 equal to the installments, then annual installment will again be fixed in
such a way that their present value at 12% is equal to the mitral loan of Rs. 50,000. In the first example
annuity is inflowing whereas in the second case it is out flowing. Further whenever the present value of
some lump sum is to be compared with the annuity, it will require the present value of annuity
(Comparison is possible at a future date which will require the future value of both the lump sum and the
annuity and it requires more calculations and gives the same result).
Present value of an ordinary annuity of Rs. A per year for t-years at 100r percent per annum will
be given by Total present value
A A A
=  
(1  r ) (1  r ) 2
(1  r )2

A  1 A 
1 
(1  r )t 1 
=
(1  r )  (1  r )
Inside the bracket it is finite geometrical progression of t terms with first term = 1 and the common
1
ratio = which is less than one. Sum of the n terms of a geometrical progression.
1 r
= a + ar + --- + arn–1
a(1  r n )
=
1 r
Total present value
  1 t 
1   
A   1  r  
=
1 r  1 1 
 
1  r 

A
1  1  r  
1
=
r  
This value can also be obtained using the table which gives gives the present value of the ordinary
annuity of Rs. one at 100% flowing for t years. According to this
1 1 1
 
1  r 1  r  2
1  r 
t

t
 1 
1  
1 r 
= (using g.p. formula)
r
The same is denoted by at/r
 The present value of ordinary annuity of Rs. A using table = A.at/r

Present Value of the annuity due


Present value of the annuity due of Rs. A per year for t years at 100r% per annum will be.
A A A
Total present value = A +  
1  r 1  r  2
1  r 
t 1

  1 t 
1    
= A  1 r  
 1 
 11  r 
 

= A
1  r  [1– (1 + r)–t]
r

Present value of the deferred annuity


In this case, annuity is deferred for n periods, so first flow is available at the end of n periods. It is
also reducible to ordinary annuity.
Present value of the deferred annuity
A A A
=  
(1  r )n+1 (1  r )n+2 (1  r ) t

A  1 1 
= 1       
1  r   1  r 1  r  
n+1 t 1

1  1  r  t 
A  
=
1  r 
n
r
Using table, present value of deferred annuity
= (1 + r)–n – . A. at/r
Where n denoted thye periods for which annuity is deferred t is the number of periods for which
annuity is flowing.

Present value of perpetual annuity


It denotes an annuity flowing forever i.e., the limit of t tends to infinity.
Present value of perpetual annuity
A A
=  
1  r 1  r 2

A  1 1 
=  1       
1  r  1  r 1  r 2 
A 1 A
= . 
1 r 1 1 r
1 r
A
[Note–For an infinite G.P. a + ar + ar2 + ........  = ; r < 1]
1 r
For perpetual annuity due, present value

= A
1  r 
r
In all these situations, the time period of annuity flow must be equal to interest period. For example,
if annuity is flowing annually, interest must also be compounded annually and if annuity is flowing
quarterly, interest must also be compounded on quarterly basis. Therefore, if interest is compounded m
r
times in sa year, annuity must also flow m times in a year. Here r is replaced by and t is replaced by
m
mt in all above cases. For example, if an annuity of Rs. 1000 is flowing quarterly for 5 years, rate of
interest being 8% per annum compounded quarterly then its present value
1000 
1  1  .02  
20
=
02  
 08 
as r  4 and t  4(5) 
 
= Rs. 16351.43
Using present value table
Present value = 1000a20/.02
= 1000 (16.35143)
= Rs. 16351.43
1000
Its future value = [1 + .02]20–1]
.02
= Rs. 24297.37
Using table, future value
= 1000 S20/.02 = 1000 (24.2974)
= Rs. 24297.40
Example 1 : A machine is purchased under installment system such that Rs. 20,000 is paid on
signing the contract and the balance in 4 annual installments of Rs. 5000 each at the end of each year. If
interest is charged at the rate of 10% per annum, what would be the cash price of machine?
Solution : Cash Price = Down payment + Present value of four installments
5000
= 20,000 + [1 – (1.10)]–4
.10
= 20000 + 15849.33
= Rs. 35849.33
Example 2 : A man borrowed Rs. 10,000 with an understanding that it is to be paid back in five
quarterly installments the first payment made at the end of first quarter. Find the value of each installment
if interest rate is 8% per annum compounded quarterly.
Solution : Let each installment is of Rs. A. So, the present value of 5 quarterly installments at r =
.08
= .02 should be
4
= Rs. 10.000
A
 10,00 = [1 – (1.02)–5]
02
= A.a5/.02 (using table)
10,000 = A (4.71345)
1000
 A = = Rs. 2121.58
4.71345
Example 3: On June 1, 1996, Mr. X borrows Rs. 5000 from a bank which charges interest at 8%
com pounded quarterly. He agrees to discharge his indebtness by making quarterly payments of Rs. 400
each, the first payment on Sept. 1, 1996
(a) When will he make the last payment of Rs. 400?
(b) What final payment 3 months later will be required?
Solution : Given Present value = Rs. 5,000
.08
r = = .02
4
Quarterly installment A = Rs. 400, t = ?
 500 = 400 [at/.02]
at/.02 = 12.5
Using table t ties between 14 and 15 periods, so X will pay 14 installment of Rs. 400 each and the
balance say Rs. y in the 15th installment.
 500 = 400a14.02 + y
(1  .02)15

500 = 400 (12.10624877) + y


(1  .02)15
 y = 157.50 (1.02)15 = Rs. 211.97
Example 4 : A debt of Rs. 6000 with interest at 4% compounded quarterly is to be discharged in 8
quarterly installments, the first due today. Find the quarterly installment.
Solution : It is a case of annuity due, Let the quarterly installments of Rs. A
.04
r = =.01, t = 8
4
A(1  r )
 600 = [1–(1 + r)–t]
r
A(1.01)
= [1– (1.01)–8]
.01
Using table
 6000 = A [at-2/.02 + 1]
= A [a7.01 +1]
= A (6.7281945 + 1)
6000
 A = = Rs. 776.38
7.7281945
Example 5 : A young orchard valued at Rs. 2 lakh is sold today for Rs. 50,000 down. The purchaser
agrees to pay the balance with interest at 10% compounded semi-annually by making 10 semi-annual
payments of Rs. A each, the first-due 4 years from today. Find A.
1
Solution : It is a case of deferred annuity. It has been deferred for 3 years 7 periods and the first
2
flow is available at the end of 8th periods (as payments are six monthly).
.10
t = 10, r = = .05
2
 2,00.000 = Down payment + present value of installments

A 1  1  r 1 
= 50000 +  
1  r 
n
 r 

A1  1.05 10 


= 50000 +  
1.05  .05 
7

Using table
200000 = 50000 + A (1.05)–7.a10.05

 A = A(1.01) 150000  150000(1.407)


.01 a10.05 7.7217
= Rs. 27334.01
Example 6 : On April 1, 1992. M borrowed Rs. 75,000 with interest at 4% compounded monthly.
He plans to discharge the debt by monthly payment of Rs. 1000, the first due on April 1, 1995. Find the
no. of full payments and the additional payment one month later to discharge the debt.
Solution : It is a case of deferred annuity as first payment is made 3 years later. So it has been
deferred for 35 months.
.04 .01
r =  , Present value = Rs. 75,000
12 3
monthly installment A = Rs. 1000
  .01   t 
1  1   
1000   3  
 75000 =
 .01   
35
.01
 1    
 3   3 
Using table
.35
 .01 
75000 = 1000 1 
a
 |
10 01
 3  3
35
 .01 
 = 75  1  
 3 
= 84.2625
As per table, t lies in between 99 and 100, so M will make 99 full payment and the balance say Rs. x
in 100h installment.
35
 .01  x
75000 = 1000 1 
a
  99 01 | 135
 3   .01 
1 
3

 3 
35
x  .01 
= 75000 – 1000 1 
a
135  |
99 01
 .01   3 
1 
3

 3 
= 75000 – 7446.578
x
135 = 53.422
 .01 
1  
 3 
135
 .01 
 x = 53.422 1  
 3 
= Rs. 83.72
135
 .01 
Note : 1   can be solved using log.
 3 
Example 7 : Find the present value of a perpetuity of Rs. 800 payable at the end of year if money is
worth (a) 6% effective (b) 6% compounted semi-annually (c) 6% compounded quarterly.
A 800
Solution : (a) Present value of Perpetuity =  = Rs. 13333.33
r .06
(b) In this case compounding is semi-annually where as annuity if flowing annually since the time
period of both must be same. So corresponding effective rate or annual rate is required.
m
 r
Effective rate =  1   –1
 m
2
 .06 
= 1   –1
 2 
= (1.03)2 –1 = .0609
800
Present value = = Rs. 13136.29
.06.9
4
 .06 
(c) Effective rate = 1   1
 4 
= (1.015)4 –1 = .06136
800
Present value = = Rs. 13037.06
.06136
Example 8 : An investor estimates an annual income of Rs. 5000 on an equity investment for ever.
How much should be invested if he wants an annual return of 20%.
Solution : Amount to be invested = the Present value of all the income flows at the required rate of
20%.
A 500
 Present value =  Rs. 25,000
r .20
Example 9 : How much is needed to endow a series of lectures costing Rs. 3000 at the beginning of
each year indefinitely, money worth 4% compounded quarterly.
Solution : It is a case of annuity due as payments are in the beginning of each year. Further annual
rate is required corresponding to 4% compounded quarterly as annuity if flowing annually.
 effective rate or annual rate (r1),
m
 r  .04 
r1 =  1    1   1   1
 m  4 
= .0406
A 1 r1 
Its present value =
r1
3000 1.0406 
=
.0406
= Rs. 76884.03
Example 10 : The timber used in a certain construction costs Rs. 4000 and will last 12 years. If
given a preservative treatment it will last 20 years. How much can be paid for the treatment, money worth
5% per annum?
Solution : The price which can be paid for the preservative treatment should be equal to the present
value of reduction in the annual cost for 20 years. Reduction in the annual cost is a difference between per
year cost without preservative and with preservative.
Let the annual cost without preservative is Rs A1 and with preservative Rs. A1
 400 = A1 12
a
|.05

A1 = 4000
a

4000 = Rs. 451.30
|
12 .05 8.8633

A2 = 4000
a

4000 = Rs. 320.97
|
20 .05 12.4622
So annual saving is (Rs. 451.30 – Rs. 320.97) available for 20 years
 Price for preservative = Rs. (451.30 – 320.97)
= 130.33. a
|
20 .05
= 130.22 (12.4622)
= Rs. 1624.19
Example 11 : A college estimates that its new building will require Rs. 800 for up keep at the end of
each year for next 10 years and Rs. 1500 ft the end of each year indefinitely. If the money is worth 4%,
how large an endowment is necessary for the future upkeep of the building?
Solution : Endowment fund = Present value of cash out flows.
Present value = 800  800 2      800 10
1.04 (1.04) (1.04)

+ 150011  150012      
(1.04) (1.04)
The first 10 years is a finite annuity and the remaining is a perpetual annuity
Using table
a
Present value = 800 30 | r
12
a
|
30 r = 20
12

Using Present value table, (look for 20 inside the table corresponding to 30 periods)
r 1
lies between 2 % and 3%
12 2
3
It may be taken as 2 % approx.
4
Corresponding effective rate
m
 r
= 1    1
 m
= (1 + .0275)12 –1
= .3847 = 38.47%
Example 13 : What sum must be deposit today in an account paying 4% compounded quarterly will
provide 20 quarterly withdrawals of Rs. 600 each, the first to be made 2 years from today?
Solution. It is a case of deferred annuity as first withdrawal is made at the end of 2nd year. So it has
been deferred for 7 periods. The amount to be deposited = Present value of 20 withdrawals of Rs. 600
each.
600 600 600
 Amount to be deposited =  
1  01 1  .01 1  .01
8 9 27

using table = (1.01)–7 .600.


= (1.01)–7.600. (18.0446)
= Rs. 10098.78
Example 14 : An individual borrows Rs. 1 lakh at 8% compounded annual. Equal payments are to
be made for 6 years. However at the time of 4th payment the individual decides to pay off the loan. How
much should be paid?
Solution : Let each installment be of Rs. A so the present value of payments at 8% should be equal
to Rs. 1,00,000
A A A
100000 =  
1  .08 (1  .08) 2
(1  .08)6
A
= [1–(1.08)–6]
.08
100000  0.08 
 A = = Rs. 21631.54
1  1.08 
6

Using table
A = 100000
a

100000 = Rs. 21631.64
|
6 .08 4.6224
Now amount payable at the time of 4th year to discharge the remaining loan is the total of the
present value of 4th payment, 5th payment and 6th payment.
21631.54 21631.54
So money payable at the time of 4th payment = 21631.54 + 
1.08
2
1.08
= Rs. 60206.30
Example 15 : Today X takes a loan with interest at 8% compounded quarterly which he agrees to
discharge be payments of Rs. 200 at the end of each quarter for the next 4 years, followed by payments of
Rs. 400 at the end of each quarter for the next 3 years. Find the loan.
Solution : Loan = Present value of both the annuities.
First annuity will be an ordinary ordinary annuity whereas second annuity will be deferred annuity.
.08
For first annuity r = = .02 and t = 4 (4) = 16 and for second annuity r is same and t = 4 (3) = 12, but
4
it has been deferred for 16 periods.
 loan = 200. 16
a
|.02 + (1.02)–16.400. a
|
12 .02

= 200 (13.5777) + .72856 (400) (10.5753)


= Rs. 5796.97
Example 16 : An orchard will yield the first full crop at the end of 5 years and is expected to
maintain an annual income of Rs. 10000 for 20 years in all. Find the cash price of the orchard, if money
worth 4% per annum.
Solution : Its cash price is equal to the present value of the deferred annuity. It has been deferred for
4 years as first flow is available at the end ot 5th year and is flowing for 20 years.
 Cash price = (1.04)–4 10000 a20 |.04
= .85480 (10000) (13.5903)
= Rs. 116169.88
Example 17 : A television whose cash price Rs. 20,000 is marked up 25% i.e. list price is Rs.
25,000. If it is sold on the installment basis, down payment of 20% of the list price is required and the
balance is paid in 10 monthly installments. What interest rate per month does the buyer pay? Also find the
corresponding nominal and the effective rate of interest.
Solution : List Price = Rs. 25.000
20
Down payment = x 25,000 = Rs. 5000
100
25,000  5000
Annual installment =
10
= Rs. 2000
 20,000 = 5000 + 2000 10a |r
a
|
10 r
= 7.5
1
For t = 10 at r = 5 a
|
10 .055
= 7.5376
2
and at r = 6, a
|
10 .06
= 7.3600
1
 r lies between 5 % and 6%
2
.0376
 r = 5.5 + = 5.71%
.0376  .1400
Corresponding nominal rate = 5.71 × 12 = 68.54%
12
 r 
and effective rate of interest = 1   –1
 12 
= (1 + .0571)12–1
= .9474 or 94.74%

Future Value of Annuity

Future value of a annuity (also known as amount of annuity) denotes the equirvalent lump-sum
available at a future date. It is the sum total of the future value of different value of different annuity
flows. If is required whenever the flows are on the left hand side and the equivalent value is required on
the right side. It can also obtained using formula based on geometrical progression as well as the future
value table of the annuity of Rs. 1.

Future value of ordinary annuity


This annuity starts flowing from the end of first period. So if an annuity of Rs. A is flowing at the
end of each year for t years at 100r% per annum, then its future value will be.
Future value = A (1 + r)t–1 + A (1 + r)t–2 + – + A
= A + A (1 + r)+ – + A (1 + r)t–2 + A (1 + r)t–1
[reversing the series]
= A [1 + (1 + r) + (1 + r)2 + – + (1 + r)t–1]
A
= [(1 + r)t –1]
r
Since it is a G.P. with first-term equal to 1, common ratio is (1 + r) which is greater than than 1 for t
years.
Using table, which gives the future value of ordinary annuity of Rs. 1 for years at 100r% per annum.
= 1. (1 + r)t–1 + 1. (1 + r)t–2 --- + 1

1  r  1
t

= denoted by ts |r
r
 future value of the ordinary of Rs. A = A. ts |r

Future value of Annuity due


It starts flowing in the beginning of first period. It can also be modified as ordinary, so its future
value can also be obtained using table.
Future value of annuity due = A (1 + r)t + A (1 + r)t–1 – + A (1 + r)
= A (1 + r) [1 + (1 + r) + (1 + r)2 +–+ (1 + r)t–1]
A
= (1 + r) [(1 + r)t–1]
r
Using future value table
= (1 + r) A. ts |r
Alternatively it can be modified as
Future value = [A (1 + r)t + A (1 + r)t–1 + – + A (1 + r) + A]–A
(by adding and subtracting A)
The first part is the ordinary annuity for (t + 1) periods.
 Future value = A. s
| –A
t 1 r

= A  ts1|r 1

Future value of deferred annuity


In case annuity is deferred for n periods, the first flow is available at the end of (n + 1) period. But
its future value is same as future value of ordinary annuity, so the same formula can be used for
calculating its future value. For example : If a person starts depositing Rs. 2000 at the end of each year
from 3rd year end till the 10th year end, implies that annuity has been deferred for two years. First deposit
is at the end of third year. It will be compounded for 7 years, the second deposit at the end of 4th year will
be compounded for 6 years and likewise. So its future value = Future value of ordinary annuity of 8 years.

Future value of perpetual annuity


Since annuity if flowing forever, future value will be infinite.
Note 1. Sinking fund also involves the future value of annuity. In this case, period deposits are made to
accumulate the required lump-sum. These are used for accumulating finds for the replacement
of assets and the redemption of debentures.
Note 2. In this also, the time period of the annuity and the interest period must be same. So if annuity is
r
flowing m times in a year, then r is replaced by and t is replaced by mt.
m
Example 1 : Mr. X deposits Rs. 1000 at the end of each year for 5 year at 4% per annum. Find his
balance after 5 years.
Solution : Balance after 5 years = Future value of the annuity.
This first deposit will be compounded for 4 years as deposit is at the end of first year and the last
deposit is compounded for zero period at it is deposited at the end for 5th year.
 Future value = 1000 [(1.04)4 + (1.04)3 + --- + 1]
1000
= [(1.04)5 –1]
.04
= Rs. 5416.30
Using future value table
Future value = 1000 5s |.04
= 1000 (5.4163)
= Rs. 5416.30
Example 2. X deposits Rs.200 in a saving account at 6 months intervals, which credits interest at
4% compounded semi-annually. The first deposit was made when X’s son was 6 month old and last
deposit was made when his son was 20 years old. The money remain in the account and was presented to
the son on his 25th birthday. How much did he receive?
.04
Solution : Initially it is and ordinary annuity flowing for 40 periods at= .02 rate. Afterwards it
2
become a lump-sum which remains in the account for 5 years or 10 periods. Therefore the balance at the
time of 25th birthday is :
= [200 40
s
|.02 ] [1 + .02]10
= 200 (60.4020) (1.2190)
= Rs. 14725.94
Example 3 : X deposits Rs. 500 in a savings account on June 1, 1990 which pays 4% compounded
semi-annually and continue to make similar deposits every deposits six month thereafter. After June, 1,
1996, the bank paid 6% compounded semi-annually. How much will be to his credit just after the deposit
of December 1, 1998?
Solution : It can be divided into two parts. First part can be accumulated upto June 1, 1996, i.e. for
.06
13 periods (and not 12) which remain in the account as lump-sum for another 5 periods at =.03 rate.
2
The second part is a deferred annuity which will be separately accumulated for 5 periods at.03 rate.
 Balance as on December 1, 1998
= [500 13s |10.2 ] (1 + .03)5 + 500. 5s |.03
= 500 (14.680) (1.15927) + 500 (5.3091)
= Rs. 11163.76
Example 4 : M purchased a machine paying Rs. 5000 down and promising to pay Rs. 500 every 3
months for the next 10 years. Seller charges interest at 8% compounded quarterly (a) what is the cash
price of the machine? (b) If M missed the first 10 payments, what he pays to the time of 11 payments to
bring him up-to-date? (c) After making 8 payments, M wished to discharge the entire debtness by a single
payment at the time of 9th regular payment was due, what must he pay in addition to regular payment? (d)
If M missed the first 10 payments, what must he pay when the 11th payment is due to discharge his entire
indebtness.
.08
Solution : (a) Cash Price = down payment + present value of Rs. 500 for 40 periods at =.02
4
rate.
 Cash price = 5000 + 500 a40 |.02
= 5000 + 500 (27.3555)
= Rs. 18667.75
(b) Amount payable at the time of 11th payment
= Future value of ordinary annuity of 11 periods
= 500. 11
s
|.02 = 500 (12.1687)
= Rs. 6084.35
(c) The additional payment will be equal to the present value of remaining 31 payments
 Additional payment at the time of 9th period = 500 31
a
|.02
= 500 (22.9377)
= Rs. 11468.85
(d) Amount payable at the time of 11th payment to discharge the entire indebtness will be the sum
total of future value of first 11 payments and the present value of remaining 29 payments.
 Amount payable at the time of 11th payment.
= 500 11
s
|.02 + 500 a29 |.02
= 500 (12.1687 + 21.8444) = Rs. 17006.54
Example 5 : As soon as B has saved Rs. 20,000 he intends to start a business. If he can save Rs. 600
every 3 months and invest at 4% compounded quarterly, find the number of Rs. 600 deposits, he must
make and the size of the final deposit.
Solution : The number of deposits can be obtained using future value table. Here r = .01
 20,000 = 600 ts |.01
s
|
t .01
= 33.33
For t = 28, s
|
28 .01
= 32.1291
For t = 29, s
|
29 .01
= 33.4504
Therefore he has to make 28 deposits of Rs. 600 each and the balance say Rs. x at the time of 29th
deposit. But the lump-sum of 28 deposits remains there till 29th period. So it is compounded for one
period.
 20,000 = [600 28
s
|.01 ] [1.01] + x
= 600 (32.1291) (1.01) + x
 x = 20,000 – 19470.23 = Rs. 529.77
Sinking found and Replacements of Assets
Future value of the annuity can also be used for solving fund problems. These are used for the
redemption of debentures and the replacements of assets. Sinking fund table gives the money to be
provided each year to accumulate Rs. 1 at the given rate. It is the reciprocal of future value table of the
annuity.
Example 6 : A debt of Rs. 6000 due in 5 years without interest is to be discharged by sinking fund
method. If 5 equal annual deposits, The first due at the end of first year, are made into a fund earning 4%
annum. Find the size of the deposit.
Solution : Let each deposit is of Rs. A, therefore the Future value of the deposits should be equal to
Rs. 6000.
 6000 = s
|
5 .04

 1.04 5  1
= A 
 .04 
6000
 A = = Rs. 1107.77
5.4163
Example 7 : A debt of Rs. 10,000 bearing interest at 12% compounded semi-annually is to be
discharged by sinking fund method. If 10 equal semi-annual deposits, the first due in 6 months are made
into a fund which pays 6% interest compounded semi-annually, find (a) the size of the deposit (b) the
semi-annual cost of the debt.
Solution : (a) Let each installment is of Rs. A. Therefore the present value of all the 10 installments
.06
at r = =.03 should be equal to Rs. 10,000.
2
 10,000 = A 10
a
|.03

10000  .03
or A = = Rs. 1172.30
1  1.03
10

(b) Six monthly cost of debt


= interest of 6 months + installment
= 10000 (.06) + 1172.30
= Rs. 1772.30

Replacement of Assets
In case of assets, annual cost will include repair cost (or operating cost) + interest lost by investing
the lump-sum in the asset + deprecation to be provided each year to accumulate funds required for the
replacement of the asset. This depreciation is fixed in such a way that if it is accumulated at the given
rate, it will be sufficient to replace the asset. Interest Cost is based on the required rate of return.
Example 8. A mine is expected to yield Rs. 30,000 each year for the next 20 years. If M desires an
8% return on his investment and can invest money safely at 4% per annum, what is the value of mine now
to M if after 20 years, (a) it is worthless (b), it can be sold for Rs. 5000.
Solution : Let the mine cost Rs. C. Therefore interest cost to meet the required return of 8% = .08 C
depreciation (D) to be fixed in such a way that if it is accumulated at 4% for 20 years, it accumulates Rs.
C after 20 years in the first case and Rs. C Rs. 5000 in the second case.
(a) D. 20
s
|.04 = C
D = C/ 20
s
|.04

 30,000 = .08C + s C
|
20 .04

 1   1 
30,000 = C .08  s   C .08 
 20 |.04   29.7781 
30000
 C = = Rs. 264126.99
.08  .0336
(b) 30,000 = .08C + C s 500
|
20 .04

 C = Rs. 26560.30

General Annuities
Annuities are also classified as (1) Simple annuities annuities (2) General annuities! Simple
annuities are those where the time period of he annuity is same as the interest period. For example if
annuity is flowing quarterly, interest is also compounded quarterly.
General annuities are those where annuity period is not coinciding with interest period. Therefore
either annuity is flowing more than once in an interest period or interest is compounded more than once
during an annuity period.
All there general annuities can be grouped into two types (a) More than one annuity flows are there
are there in each interest period. For example : Annuity is flowing quarterly with interest compounded
annually. Here either corresponding annual flow is obtained by combining the four flows of each interest
period or corresponding quarterly interest rate is obtained rate is obtained using the relationship between
nominal and the effective rate of interest as to quarterly interest rate is obtained using the relationship
m
 r
between nominal and the effective rate of interest as to R = 1    1 where R is the effective rate, r is
 m
the nominal rate and m denoted the interest compounded in a year. If the first alternative is used, it is
known as combining the annuity.
Example 9 : If interest is at 6% compounded semi-annually, replace the payments of Rs. 1000 at the
end of each month by a six monthly payment.
Solution : Let the corresponding 6 monthly payment = Rs. x
Therefore it should be equal to the future value of ordinary annuity of Rs. 1000 flowing for 6
12 2
 r   .06 
months with r1 rate per month given by 1  1   1  
 12   2 
r1 1
or = 1.03 6 –1
12
s
 x = 1000 6 | r1
12

1000  
6
r1 
= 1    1
r1/12  12  
= 1000 [1.03  1] = Rs. 6074.57
1
(1.03) 6  1
If an annuity of Rs. A is flowing P times in an interest period with interest rate 100r% per time
no min al A(r )
period (given by ), then the combining factor is given by 1 , but r must be the rate per
m
(1  r )  1
p

time period.
(b) Annuity flowing period is greater than the interest period i.e. interest is compounded more than
once in an annuity period. For example. Annuity is flowing annually but interest is compounded
quarterly. In this case also corresponding annual rate is obtained using the relationship between two
nominal rates and the relationship between nominal and the effective rate of annuity is split according to
interest period.
Example 10 : If interest is at 6% compounded quarterly, replace a payment of Rs. 2000 at the end of
the year by an equivalent payment at the end of each quarter.
Solution : Let each equivalent quarterly payment is Rs. x. So if the four quarterly payments ae
.06
combined together at r = =.015, their future value should be equal to Rs. 2000.
4
 200 = x 4s |.015

or x = 2000
s
= 2000 (.02444)
|
4 .015

= Rs. 488.88
Alternatively corresponding annual rate, R can be obtained by
4
 r
R = 1    1
 4
4
 .06 
= 1   1
 4 
= (1.015)4 –1
x 1s |R = 2000

x = 2000
s
1 R |

2000[1.015   1]
4

=
1.015
4

Therefore the splitting factor is given by s1 where r is the time period.


|
1 R

Example 11 : Find the amount and the present value of an annuity of Rs. 1500 per year for 6 years
with interest at 6% compounded semi-annually.
Solution : Here annuity period is twice the interest period as interest is compounded twice in each
annuity period. So one alternative is to split the annuity.
 Let each six monthly flow is of Rs. x which amounts to Rs. 1500 at the end of each year.
 x 2s |.03 = 1500

x = 1500
s
|
2 .03

Now both the annuity period and the interest period are equal.
.06
 Amount of the annuity at = .03 rate for 12 periods will be
2
Amount = x 2s |.03

= 1500
s
s
|
12 .03
2 |.03

1500[(1.03)12  1]
= .03
(1.03) 2  1
.03
= Rs. 7355.17
Alternatively interest rate may be modified according to annuity period using the relationship
between effective and nominal rate of interest. Let the annual rate is R corresponding to the nominal rate
6% compounded six monthly.
 R = (1.03)2 –1
Using this rate R
Amount = 1500 6s |R

 1  r 6  1 
= 1500  
 R 

 1.0312  1 
= 1000  
 1.03  1 
2

= Rs. 10486.72
Present value = 1500 6a |R

1  1  R 6 
= 1500  
 R 

1  1.0312 
= 1500  
 1.03  1 
2

= Rs. 7355.17
Example 12 : If the money is worth 6% compounded quarterly, replace the payments of Rs. 2000 at
the end of each year by equivalent of Rs. x (a) at the end of each monthly (b) at the beginning of each
month.
Solution : If is a case of combining the annuity as annuity period is less than interest period.
Annuity is flowing 3 times in each interest period.
One alternative is to ascertain the monthly rate corresponding to quarterly rate using the relationship
between two nominal rates. Let the monthly rate is r/12 corresponding rate 6% compounded quarterly.
12 4
 r   .06 
 1   = 1  
 12   4 
r 1
or = 1.015 3  1
12
r
Let the monthly payments is of Rs. x at rate for 12 periods.
12
(a) It is a case of ordinary annuity. Therefore future value of annuity of Rs. x should be equal to Rs.
2000.
s
 2000 = x 12 | r
12

2000
or x = s
12 | r
12

= 2000
12
 r 
1    1
 12 
r
12
 1

1.015   1
3

= 2000  
1.015  1
4

= Rs. 162.15
(b) It is a case of annuity due
 
 2000 = x  13s | r 1
 12 
2000
or x = s
13 | r
12

2000
x =
 r 
13

 1   –1 
  12  – 1
 r 
 
 12 
2000
=
 13
 
  (1.015) 3
 
 1  – 1
  
 (1.015) – 1  
3

= Rs. 161.35

PRESENT AND FUTURE VALUES OF ANNUITIES USING DEFINITE INTEGRATION


Whenever an annuity is compounded continuously, its present value and the future value is obtained
using definite integration. In such cases rate of interest is also known as rate of force. For present value,
the discounting factor is e–rt and for future value, compounding factor is ert.
Present value of finite annuity–If an annuity of Rs. A is flowing every year for for t-years rate of
interest being 100r% compounded continuously, then its present value is given by
t

 Ae
–rt
Present value = dt
0
t
e – rt
= A
–r

A
t
1 – e– n 
0

Present value of perpetual annuity


It is given by evaluating the limit of finite annuity when t tends towards infinity.
Present value of perpetual annuity
a A
= lim (1–e–rt) =
t  r r
(as– = 0)

Future value of finite annuity


Future value of an annuity of Rs. A flowing for t years at 100r% is given by Future value
t

 Ae
–rt
= dt
0

5000
= (–1 + e5)
.10
= Rs. 32436.06
But in case it is continuously decreasing at the rate of 10% per annum, then cummulative sales of
first 5 years is given by
5

 5000
–10t
= dt
0

5000
= (–1  e5 )
.10
= Rs. 19673.46
Example 14 : A bank pays interest at the rate of 6% per annum compounded continuously. If person
deposits Rs. 2000 in the bank each year, how much will be his balance after 5 years?
Solution : Balance after 5 years
= Future value of annuity of Rs. 2000 of 5 years at 6% compounded continuously.
5

  2000e
.06t
Balance = dt
0

2000 3
= (e – 1) = Rs. 11661.96
.06
Example 15 : A bank pays interest at the rate of 10% p.a. compounded continuously. How much
should be deposited each year is order to accumulate Rs. 15000 in 10 years.
Solution : Let he deposits Rs. A each year
then its future value = Rs. 15000
10

  Ae
.10t
15000 = dt
0

A
= (e – 1)
.10
15000(.10)
 A = = Rs. 872.95
e –1
Example 16 : How much should invested every month in an account paying 12% p.a. compounded
continuously so as to accumulate Rs. 10,000 in 5 years.
.12
Solution : Let the monthly deposit be Rs. A. then its future value at = .01 rate for 60 months
12
should be equal to Rs. 10000.
60

  Ae
.01t
10000 = dt
0

A
= (e6 – 1)
.10
15000 (.10)
 A = = Rs. 121.64
e –1
Example 17 : Find the present value of annuity of Rs. 100 per annum assumed to be payable
continuously for 10 years at the rate of 4% p.a. compound continuously.
t

 Ae
–rt
Solution : Present value = dt
0

10

= 100  e
–.04t
dt
0

100
= (1 – e–4) = Rs. 824.20
.04
Example 18 : An income stream decreases continuously over time for m years, the income rate at t
years firm now being ae–bt per year. What is its present value if interest is reckoned at 100r%
compounded continuously. Show this equals the present value of a uniform income stream of Rs. a per
for m years if the rate of interest is raised to [100 (r + b)]% per year.
Solution : In the first situation,
A = ae–bt t = m years, r = 100r%
t

e
–.rt
 Present value = dt
0

m m

 ae e dt =  ae –(b+r)t dt
–bt – rt
=
0 0
a
= (1 – e–m(b+r))
.b  e
In the second case A = Rs. a, t = m years and r = 100 (b + r)%
m

 ae
–(b+r)t
 Present value = dt
0

a
= (1 – e–m(b+r))
br
So both the present values are equal.
Example 19 : A company’s dividend on enquiry shares is according to Dt = D0egt where D0 is
constant g is the growth rate growing continuously. Determine the total present value of the dividend of
first t-years if the rate of interest is 100r% compounded continuously. Also determine the total present
value if t (assuming r > g)
t

De
–rt
Solution : Total present value of t years dividend = 1 dt
0

D e
gt – rt
= 0 e dt
0

= D0  e
–t(r–g)
dt (as r > g)
0

= D0 (1 – e–t(r–g))
r–g
In case t 
Total Present value = lim D0 (1–e–t(r–g))
t  r–g

= D0
r–g
Example 20 : Assume that is 1996, the annual world use of natural gas was 580 trillion cubic ft. The
annual consumption of the gas is increasing at the rate of 3% compounded continuously. How long will it
take to use all available gas, if it is known that in 1996, there were 2200 trillion cubic ft of the proven
reserves? Assume that no new discoveries are made.
Solution : Let the reserves are lasting for t years. So the cumulative consumption of t years should
be equal to the reserves.
t

 2200 =  50.e
.03t
dt 
.03
 e – 1
50 .03t
0

2200(.03)
 e03t = +1
50
= 2.33
Taking log on both sides
.03t log e = log 2.32
 t = log 2.32  log 2.32
.03log e .03log 2.7183

= .3655 = 28.05 years.


(.03)(.4343)
B.Com (Hons) IInd Year
Paper IX—BUSINESS MATHEMATICS
Assignment–I (1994)
Answer all questions. Use of simple calculator is allowed.
1. (a) Find the elasticities of demand and supply at equilibrium price for demand function p =
100 – x 2 and supply function x = 2p – 10, where p is price and x is quantity.
Or
Suppose the demand and total functions of a monopolist are p = 20 – 4x and TC = 4x + 2
respectively, where p is price and x is quantity. If the government imposes tax @ 20% of sales,
determine the total tax revenue that the government will be able to collect.
(b) Show that the production function
x = f (l, k) = 2 lk
(where x, l and k are the units of output, labour and capital respectively) gives constant return to
scale and diminishing return to inputs.
Or
Find the first and second order total differentials of the function
z = f (x,y) = 7y log (1+x).
(c) Find the consumer surplus and producer surplus under pure competition for demand function p
8 1
= –2 and supply function where p = (x+3) where p is price and x is quantity.
x 1 2
Or
A bank pays interest at the rate of 6% per annum compounded continuously. Find how much
should be deposited in the bank each year in order to accumulate Rs. 6,000 in 3 years.
2. (a) The following matrix gives the number of units of three products (P, Q and R) that can be
processed per hour on three machines (A, B and C) :
A B C 
P
 10 12 15 
Q
13 11 20 
R 
16 18 14 
Determine, by using matrix algebra, how many units of each product can be produced, if the
hours available on machines A, B and C are 54, 46 and 48 respectively.
(b) Vitamins A, B and C are found in foods F1 and F2. One unit of F1 contains 1mg of A, 100mg of
B and 10mg of C. One unit of F2 contains 1mg of A, 10mg of B and 100mg of C. The minimum
daily requirement of A, B and C are 1mg, 50mg and 10mg respectively. The costs per unit of F1
are F2 are Rs. 1 and Rs. 1.5 respectively. You are required to (i) formulate the above as a linear
programming problem minimising the cost per day, (ii) Write the dual of the problem and (iii)
solve the dual by using simplex method and read therefrom the answer to the primal.
Or
(a) Solve the following liner programming problem graphically:
Maximise z = 4x + 6y
Subject to constraints x + y = 5
x  2
y  4
x, y  0
(b) A firm has three service departments, S1, S2 and S3, and two production departments, P1 and P2.
The direct cost of department and the percentage to total cost of each service department
allocated to various department are given below:
Direct cost Percentage allocation of
Dept. (Rs.) total cost of department
S1 S2 S3
S1 14,000 0 30 20
S2 12,000 10 0 10
S3 45,000 10 10 0
P1 50,000 50 30 40
P2 60,000 30 30 30
Total 100 100 100
3. (a) State the additive and multiplicative laws of probability.
Or
A question paper contains 6 questions of equal value divided into two sections of three question
each. If each question poses the same amount of difficulty to Mr. X, an examinee, and he has
only a 50% chance of solving it correctly, find the answer to any one of the following:
(i) If Mr. X is required to answer only three questions from any one of the sections, find the
probability that he will solve all three questions correctly.
(ii) If Mr. X is given the option to answer the three questions by selecting one question out of
the two standing at serial number one in the two sections, one question out of the two standing
at serial number two in the two sections, and one question out of the two standing at serial
number three in the two sections, find the probability that he will solve all three questions
correctly.
(b) There are four different choices available to a customer who wants to buy a transistor set. The
first type costs Rs. 800, the second type Rs. 680, the third type Rs. 880 and the fourth type Rs.
760. The probabilities that the customer will buy these types are 1/3, 1/6, 1/4 and 1/4
respectively. The retailer of these sets gets a commission @ 20%, 12%, 25% and 15% for these
sets respectively. What is the expected commission of the retailer?
Or
In an automobile factory, certain parts are to be fixed to the chassis in a section before it moves
into another section. On a given day, one of the three person A, B and C carries out this task. A
has 45% B has 35% and C has 20% chance of doing it. The probabilities that A, B or C will take
more than the allotted time are 1/16, 1/10 and 1/20 respectively, If is it found that one of them
has taken more time, what is the probability that A has taken more time?
(c) What do you mean by the continuity of a function at a point? What are the discontinuities, if
any, of the function
x2 – 7
f(x) =
x3 – x 2  x – 1
Assume that the marginal cost in lakhs of rupees in given by
3 –x
MC = 4 + 5x2 + e ,
2
where x is the quantity produced. Find the total cost of production when x = 2, if fixed cost is
Rs. 6 lakhs.
B.Com (Hons) IInd Year
Paper IX—BUSINESS MATHEMATICS
Assignment–II (1995)
Answer all questions. Use of simple calculator is allowed.
1. (a) The proportion of people responding to an advertisement of new product in a certain marketing
area is given by
(1 – 2–0.3t)
Where t is the number of days advertisement runs. The marketing area contains 4,00,000
potential customers and each response to the advertisement results in an average profit of Rs. 2.
This profit is exclusive of advertising cost. The fixed cost of producing the advertising is Rs.
30,000 and the variable cost is Rs. 4,000 for each day the advertisement runs. How many days
should the advertisement run to maximise total net profit?
(b) Determine consumer surplus and producer surplus under pure competition for the demand
function
p = 36 – x2
and supply function
x2
p = 6+
4
where p is price and x is quantity.
(c) Find the elasticity of substitution for the production function
Q = f (L, K) = AkaL1–a
Or
(i) A firm’s annual sales are s units of product which the firm purchases from a supplier. The
ordering cost of the firm is Rs. r per order. The cost of storing one unit for a year is Rs. h. Find
the economic order quantity for the firm by using calculus.
(ii) If interest is reckoned continuously at 4% per annum. Find the capital value of a uniform
income stream of Rs. 100 per year for 10 years.
(iii) The following are the demand function of two commodities X1 and X2 :
x1 = p1–17 p20.6, x2 = p10.7 p2–0.2,
where x1 and x2 are the quantities of X1 and X2 respectively and p1 and p2 are their respective
prices.
Find the four partial elasticities of demand and determine whether the commodities are
competitive or complementary.
2. (a) A firm has three S1, S2 and S3 and two production department P1 and P2. The direct cost of each
department and the percentage of the total cost of each department allocated to various
departments are given below:
Direct cost Percentage allocation of
Dept. (Rs.) total cost of departmen
S1 S2 S3
S1 25,000 0 30 10
S2 1,24,000 20 0 10
S3 77,000 10 10 0
P1 1,00,000 40 30 30
P2 2,00,000 30 30 50
Total 100 100 100
Find the total cost (direct plus allocated) of each department by using matrix algebra. Verify that
the sum of the service costs allocated to the production departments equals the sum of the direct
costs of the service departments.
(b) Explain how you will identify the cases of redundant constraint, no solution, multiple solution
and unbounded solution from the graph of a linear programming problem involving two
variables. Give a rough sketch of each.
Or
(i) Define and distinguish between matrix and determinant. Write the important application of
matrix algebra to business situations.
(ii) The meat department of a supermarket deals in three types of meat, say, A, B and C. The
manager of the department finds that he has 240 pounds of A, 900 pounds of B and 450 pounds
of C on Saturday morning. From past experience, he knows that he can sell two thirds of these
quantities as straight cuts. The remaining meat will have to be ground into hamburger patties
and picnic patains 20% A and 60% B. Each pound of picnic patties contains 50% B and 30% C.
The remainder of each product consists of an inexpensive non-meat filler which is available in
unlimited quantities. Find graphically how many pounds of each product should be made so that
the maximum amount of meat is used.
3. (a) Write the various approaches to probability.
(b) A bag contains 4 white and 6 black balls. Another bag contains 5 white and 8 black balls. Tow
balls are transferred at random from the first bag to the second bag and then a ball is drawn from
the second bag. If the ball drawn from the second bag happens to be black, find the probability
that one white ball and one black ball were transferred.
(c) Find the demand function whose price elasticity or demand is unit throughout.
Or
(i) A woman has 10 has 10 keys of which only one fits her door. If she tries them successively,
find the probability that a key fits in at the most 4 tries.
(ii) A box contains five tickets, two of which carry a prize of Rs. 6 each and the other three Rs. 1.50
each. If two tickets are drawn at random, find the expected value of the prize.
(iii) Examine whether the following function has minimum or maximum value:
U = f (x,y,z) = x2 –3xy + 3y2 + 4yz + 6z2.
B.Com (Hons) IInd Year
Paper IX—BUSINESS MATHEMATICS
Assignment–III (1995)
Answer all questions. Use of simple calculator is allowed.
1. (a) The average cost per repair of a colour TV set is found to be Rs. 180 and the number of repairs
is given by t4/3, where is measured in months. If the replacement cost of the set is estimated to be
Rs. 12,500, find the optimal replacement period.
Or
Discuss the continuity of the following function on interval (–3, 3):
9x
f(x) = for x < 1,
x2
= 3 for x = 1,
x3 x3
= for x > 1. ,
x x
(b) Find the degree of homogeneity and elasticity of substitution for the production function
Q = f, (l, k)= Akal1–a.
Or
A monopolist has the possibility of discriminating between the domestic and foreign markets
where his demand functions are x1 = 21 – 0.1 p1 and x2 = 50–0.4 p2 respectively (p1 and p2 are
the prices when x1 and x2 unit are demanded in the respective markets). His total cost function is
TC = 10 (x1 + x2) + 2000. What prices should the monopolist charge to maximise his profit?
Verify that he will charge higher price in the market that has the lower price elasticity of
demand.
(c) If a sum of Rs. 500 is deposited each year in a saving account paying interest @ 5.5% per
annum compounded continuously, how much is in the account after 4 years?
Or
If a marginal revenue function is MR = 8 –3 (where x is quantity), find the total and
( x  4) 2
average revenue functions.
2. (a) Solve graphically the following linear programming problem:
Maximise z = 3x + 2y
Subject to the constraints
–2x + y  1
x  2
x+y  3
x, y  0.
(b) A firm has divisions and each division produces a single product. Each division uses a part of
the products of the other divisions as input in its own production process.
The input-output matrix (in lakhs of rupees) is as under:

Inputs Outputs
Product Product Product Sales and Total
A B C Inventory output
Product A 0 15 20 65 400
Product B 30 0 20 100 150
Product C 20 30 0 150 200
Raw material 10 30 40
Labour 20 20 40
Overhead 10 30 50
Profit 10 25 30
Total inputs 100 150 200
Find by using matrix algebra. (i) the gross outputs of the three products to meet the projected
demands (sales and inventory) of Rs. 130 lakhs of product A, Rs. 200 lakhs of product B and
Rs. 300 lakhs of product C and (ii) the input requirements other than those of the final products
to meet the projected demands.
Or
(a) Three products (I, II and III) are produced by using four inputs (P, Q, R and S). The following
table gives the input requirements per unit of output:
Input Input requirement per unit of output
1 II III
P 2 4 3
Q 2 1 4
R 4 8 5
S 3 2 6
Determine, by using matrix algebra, the total requirement of each input, if 400 units of each
product are produced.
(b) A firm makes three types of leather belts (A, B and C) which are processed on three machines
(M1, M2 and M3). One unit of A requires 2 hours on M1 and 3 hours on M3.One unit of B
requires 3 hours on M2, 2 hours on M2 and 2 hours on M3. One unit of C requires 5 hours on M2
and 4 hours on M3. The maximum hours available per day on the three machines are 8, 10 and
15 respectively. The profit gained from A is Rs. 2 per unit, from B Rs. 5 per unit and from C Rs.
4 per unit. Find the optimal optimal daily production of each type of belt. State whether the
hours available on the different machines can be curtailed without affecting the optimal output.
3. (a) “The serious limitation to the classical approach to probability that it is applicable only to the
problems where the cases are equally likely led to the development of the statistical approach.
But the statistical approach was also found to be insufficient to cover those problems which do
not lend themselves to experimentation; hence, personality approach was developed.” Explain
and exemplify.
Or
In an office, there officers each of whom requires the services of one of three typists in a typing
pool. If each officer selects a typist at random, find the probability that (i) the same typist is
asked for, and (ii) all typists are asked for.
(b) Assume that the middle and upper class of consumers comprise 35% of the population and each
consumer of this group has the probability 0.8 of purchasing a particular brand of a product.
Each consumer of the rest of the population has the probability 0.3 of purchasing the brand. If a
consumer chosen at random is a buyer of the brand, find the probability that he belongs to the
middle and upper class of consumers.
Or
1 1 1
A problem is given to three students, A, B and C, whose changes of solving it are , and
3 4 5
respectively. Find the probability that the problem will be solved by at least two students.
(c) The demand and marginal cost functions of a monopolist are p = (6 – x)2 and MC = 14 + x
respectively, where p is price and x is quantity. Find the consumer surplus.
Or
What do you mean by total differential? Use the concept of total differential to find dx of the
dy
function:
2x2 – 3xy3 – y2 = 4x3y + 8
B.Com (Hons) IInd Year
Paper IX—BUSINESS MATHEMATICS
Assignment–IV (1997)
Answer all questions. Use of simple calculator is allowed.
1. (a) A company makes a profit of Rs. 5 on each unit of its product it sells. If it spends an amount of
Rs. A per week on advertising, then the number of items per week it sell is given by :
x = 200 (1 – e–ka), where k = 0.001
Find the value of A that will maximise the net profit.
Or
Using calculus, show that:
 1 
MR = AR  1 – 
 Ep 

a– p
and verify for the demand function x = b 0.
b
(b) If the supply function x = f (p1, p2,...Pm) is homogenous of degree n, show that the sum of the
partial price elasticities of supply equals n. (x denote the quantity supplied of a particular
commodity and p1, p2,......Pm are the price of different commodities.)
Or
A consumer has a utility function U = xy where x and y are the quantities that the consumes of
two goods available to him where U is and index of utility and 0 < x,  < 1 (i) show that there is
diminishing marginal utility to the increased consumption of either commodity when the
consumption of other is held constant. (ii) show that indifference curves have negative slope.
(iii) What happens to the marginal utility of x as y is increased? (iv) Do you need any further
calculation to discover what happens to the marginal utility of y as x increased?
(c) The price elasticity of demand of a commodity Ep= 3P . Find the corresponding
( p – 1)( p  2)
demand function if quantity demanded is 8 units when the price is Rs. 2.
Or
If t = 0, the annual world use of natural gas was 50 trillion cubic feet. the annual contain of gas
is increasing at the rate 10% continuously. Assuming the rate remains the same in future–
(i) How many cubic feet will be consumed in the next 10 years from t = 0?
(ii) How long will it take to use all the available gas, if it is known that world reserves are of 2200
trillion cubic feet in the beginning (assume that no new discoveries are made)?
2. (a) Given the Simplex tableau for a maximisation problem of linear programming:
CJ Product Mix x1 x2 S1 S2 Quantity
4 x2 1 1 1 0 6
0 S2 1 0 –1 1 2
Cj 3 4 0 0
(i) Test whether the above table gives optimum solution.
If not, improve it and find the optimum solution.
(ii) If S1, and S2 are slack variables at machine I and machine II, what is the maximum price you
will like to pay for one hour of each machine?
Or
A firm produces two products P1, and P2, passing through two machines M1, and M2 before
completion. M1 can produce either 10 units of P1 or 15 units of P2 per hour. M2 can produce 15
units of either product per hour. Find daily production of P1 and P2 if time available is 12 hours
of machine M1 and 10 hours of M2 per day using matrix inversion.
(b) A firm has two machines M1 and M2 costing Rs. 45,000 and Rs. 30,000. Each has 5 years life
with scrap value nil. Find depreciation of each machine for each year using matrix notations if
(i) both are depreciated by sum of the year's digit method, (ii) first is depreciated by sum of the
year's digit method and second by straight line method.
Or
The standard weight of a special purpose brick is 5 kg and it contains two basic ingredients B1
and B2. B1 costs Rs. 5 per kg and B2 costs Rs. 8 per kg. Strength considerations direct that the
brick contains not more than 4 kg of B1 and minimum of 2 kg of B2. Since the demand for the
product is likely to be related to the price of the brick, find the minimum cost of brick satisfying
the above conditions.
3. (a) Evaluate
4x  x
lim
x 0 3x – 2 x
Or
For the demand function x = a – b (where P is a price, x is quantity demanded and a, b are
p
positive constants). Show, by using the concept of limit, that the demand increases to infinitely
large amount as the price falls. Also show the total revenue reaches a limiting value as the
quantity demanded increases.
(b) Machine A costs Rs. 10,000 and has useful life of 8 years, Machine B costs Rs. 8,000 and has
useful life of 6 years. Suppose machine A generates an annual savings of Rs. 2,000 while
machine B generates an annual savings of Rs. 1,800. Assuming the time value of money is 10%
per annum which machine is preferable?
Or
A company may obtain a machine either by leasing it for 5 years (useful life) at annual rent of
Rs. 2,000 or by purchasing the machine for Rs. 8,100. If the company can borrow money at 10%
per annum, which alternative is preferable?
(c) Distinguish between the nominal and effective rate of interest. Also establish the relationship
between nominal and effective rate of interest when compounded n times in a year and when
compounded continuously.
Or
A company's dividend on the equity share is according to D1 = D2. egt where D0 is constant, g is
the growth rate continuously. Determine the total present value of dividends of first t years if the
rate of discount is 100 r percent compounded continuously. Also determine total present value if
t  (assume r > g).
B.Com (Hons) IInd Year
Paper IX—BUSINESS MATHEMATICS
Assignment–V (1998)
Answer all questions. Use of simple calculator is allowed.
1. (a) A firm requires 10,000 units of a material per annum. The cost of purchasing is Re. 1 per unit,
the cost of replenishment of stock of material is Rs. 25 and the cost of storing material is 12.5
per cent per annum of the average rupee inventory. Find the optimum order size and the
corresponding total cost using calculus.
(b) A firm's production function is:
Q = f(L, K) = ALK.
Under what conditions will it give increasing returns to scale and diminishing returns to inputs,
if possible.
The demand equation of a product is P–10e–x = 0. Find the consumer surplus when the market
price p = 1.
Or
(a) A shoe manufacturer produces both the high grade and the low grade shoes in the units of X
hundreds of pairs and Y hundreds of pairs respectively. The plant is capable of producing the
42 – 5X
shoes according to the relationship Y = X < 10. are sold at a price twice the low grade
10 – X
shoes, determine the number of pairs of both which should be produced to maximise total
revenue.
(b) Find the marginal rate of technical substitution and the elasticity of substitution for the
production function.
LK
Q = f (L, K) =
LK

(c) A firm's marginal revenue function is MR (X) = 20e–x/10 1 – X  . Find the corresponding
 10 
demand function.
2. (a) A firm has two grades of cashew nuts : Grade I – 750 kg and Grade II – 1200 kg. These are to
be mixed in two types of packets of 1 kg each - economy and special. The economy pack
consists of grade I and grade II in the proportion of 1:3, while the special pack combines the two
in equal proportion. The contributions of the economy and the special packs are Rs. 5 and Rs. 8
per pack respectively.
(i) Formulate this as a linear programming problem to maximize contribution and solve it by
simplex method.
(ii) Write the dual of the above problem, read the optimum solution of the dual and give its
economic interpretation.
(b) A firm produces three products P1, P2 and P3 requiring the mix up of three materials M1, M2 and
M3. The per unit requirement of each product for each material (in units) is as follows:
M1 M 2 M 3
P1  2 3 1
A =  
P2  4 2 5
P2  4 4 2 

Using matrix notations, find–


(i) the total requirement of each material if the firm produces 100 units of each product,
(ii) the per unit cost of production of each product if the per unit cost of materials M 1, M2 and M3
are Rs. 5, Rs. 10 and Rs. 5 respectively, and
(iii) the total cost of production if the firm produces 200 units of each product.
Or
(a) Given below is a table obtained after few iterations using simplex method, to solve a linear
programming problem to maximise total contribution from product A and product B.
Basic
Variables X1 X2 S1 S2 S3 Quantity
X2 0 1 –3/5 -2/5 0 300
X1 1 0 –2/5 3/5 0 300
SB 0 0 –1/5 –1/5 1 400
Ci 8.5 10.5 0 0 0
Give short answers to the following giving reasons :
(i) Is the above solution optimal ?
(ii) Is the above solution feasible ?
(iii) Does the problem have alternative solution ? If so, find the other solution.
(iv) What are the shadow price of the resources ?
(b) (i) Two industries input-output relationships are given below in A with final demand vector B
(in units):
I II
I  75 
A = I  50 75  B =  
 
II 100 20 
II  50 

I  400 
If the gross output increases to 600  , determine the final demand which can be
II  
satisfied. Also test the Hawkins Simon conditions.
 5/ 7 1/ 7 
(ii) Given: A–1 =  
 3/ 7 2 / 7 
evaluate A2 + 2A.
3. Answer any three of the following :
(a) A purchased a television paying Rs. 5000 down and promising to pay Rs. 200 every three
months for next 4 years. The seller charges interest at 8% per annum compounded quarterly.
(i) What is the cash price of television?
(ii) If A missed the first three payments, what must he pay at the time the fourth is due to bring him
upto date?
(b) At six-month intervals, A deposited Rs. 100 in a saving account which credit interest at 10% per
annum compounded semiannually. The first deposit was made when A's son was six month old
and the last deposit was made when his son was 8 years old. The money remained in the account
and was presented to the son on his 10th birthday. How much did he receive?
(c) Evaluate:
– mt
 r
lim p 1  
m  m
(d) A firm's present annual sales are Rs. 10,000. The sales are continuously increasing at the rate of
20% per annum. Find the cumulative sales of first five years using calculus.
(e) The sums of Rs, 2,000, Rs. 3,000 and Rs. 4,000 are due at the end of 2, 4 and 8 years
respectively. It is proposed to replace this series of payments by a single sum of Rs. 9,000
payable at the end of n years. If the rate of interest is 10% per annum effective, find the value of
n.
B.Com (Hons) IInd Year
Paper IX—BUSINESS MATHEMATICS
Assignment–VI (1999)
Answer all questions. Use of simple calculator is allowed.
1. (a) A chemical company acquires its requirement in 20 orders of the size 250 drums each. Each
drum cost Rs. 80. The replenishment cost is Rs. 40 irrespective of the order size, and the annual
carrying cost is 2% of the average rupee inventory. Find how much can company save if it buys
according to economic order quantity, derived using calculus.
(b) For the production function:
Q = f (L, K) = AkLea(K/L)
Where A, ,  and a are positive constants, find the marginal products of labour and capital.
Also verify Euler's theorem.
(c) The marginal cost function of the firm is MC(x) = (Inx)2. Find the total cost of 100 units if the
cost of producing one unit is Rs. 22.
Or
(a) A firm's demand function is:
 10 
x = 4000 In  
 p
Find the price and quantity where total revenue is maximum. Also, find price elasticity of
demand at that price.
(b) Two commodities joint demand functions are :
– a11 a12 p 2a1
x1 = p1 e
And
– a 22 a21 p1 a2
x2 = p1 e
Where x1 and x2 are the units demanded when prices are p1 and p2 per unit respectively. Show
that direct price elasticities of demand are independent of prices whereas cross price elasticities
of demand depend on the price of other commodity. Also find the condition when both the
commodities are
(i) Competitive: (ii) Complementary.
(c) A monopolist's demand function is x = 210 - 3p where x is the quantity demanded when 10
10
price is Rs p per unit. With the average cost function AC (x) = x + 6 + , find the consumer
x
surplus at the price which monopolist will like to fix.
2. (a) A company produces 2 types of leather belts–Type A and B. Contribution per belt is Rs. 4 for
type A and Rs. 3 for type B. The time requirements of one belt of type A and type B are in the
ratio of 2 : 3. Time available is sufficient to produce 500 belts of type A. The leather is
sufficient for only 400 belts. Belt A requires a fancy buckle and only 200 fancy buckles are
available,
(i) Formulate above as a linear programming problem.
(ii) Solve it by simplex method and comment on this optimum solution.
(iii) Write its dual and read the optimum solution of the dual.
(b) Mr. X has invested a part of his investment in 10% bond A and a part in 15% bond B. His
interest income during first year is Rs. 4,000. If he invests 20% more in 10% bond A and 10%
more in 15% bond B, his income during second year increases by Rs. 500. Find his initial
investment and the new investment in bonds A and B, using matrix method.
Or
(a) A retired person wants to invest upto an amount of Rs, 30,000 in the fixed income securities.
His broker recommends investing in two bonds-bond A yielding 7% per annum and bond B
yielding 10% per annum. After some consideration he decides to invest at the most Rs. 12,000
in bond B and at least Rs. 6,000 in bond A. He also wants that the amount invested in bond A
must be at least equal to the amount invested in bond B. What should the broker recommend if
the investor wants to maximise his return on investment ? Solve graphically.
(b) A transport company uses 3 types of trucks T1, T2 and T3, to transport 3 types of vehicles V1, V2
and V3. The capacity of each truck in terms of 3 types of vehicles is given below:
V1 V2 V3
T1 1 3 2
T2 2 2 3
T3 3 2 2
Using matrix method find:
(i) The number of trucks of each type required to transport 85,105 and 110 vehicles of V1, V2 and
V3 types respectively.
(ii) Find the number of vehicles of each type which can be transported if company ius 10, 20 and 30
trucks of each type respectively.
3 Attempt any three parts :
(a) M purchased a television paying Rs. 5,000 down and promising to pay Rs. 200 every quarter for
next 10 years. The seller charges interest at the rate of 12% per annum compounded quarterly. If
M missed the first 10 payments, what must he pay when the 11th payment is due, to discharge
his entire loan ?
(b) An orchard will yield its first full crop at the end of 5 years and is expected to maintain an
annual income of Rs. 5,000 for 20 years in all. Find the cash price of the orchard if money is
worth 3% per annum.
(c) An income stream decreases continuously over time for years, at the rate being ae -bt per year at t
years form now. Find its capital value at 100r percent compounded continuously. Show that this
equals the capital value of a uniform income stream of Rs. a per year for t years if rate of
interest is raised to 100 ( r + b) per year compound continuously.
(d) Mr. X deposited Rs. 10,000 in a bank for 3 years offering interest at the rate of 6% compounded
half yearly during first year, at the rate of 12% compounded quarterly during second year and at
10% compounded continuously during 3rd year. Find his balance after 3 years.
(e) How much money is needed to endure a series of lectures costing Rs. 2,500 at the beginning of
each year indefinitely, if money is worth:
(i) 3% compounded annually;
(ii) 12% compounded quarterly.
B.Com (Hons) IInd Year
Paper IX—BUSINESS MATHEMATICS
Assignment–VII (2000)
Answer all questions. Use of simple calculator is allowed.
1. (a) Apiece of machinery costs Rs. 12.000. The cost of operation from time of purchase up to a time
t is given by the function 20t2 + 15t. If the machine is sold as scrap after t years, the resale value
is given by the function 6880-60t2. Find the optimal time when the machine should be replaced.
Or
A monopolist's total cost is TC = ax2 + bx + c, and the demand function is x = –1(–p). where
x and p are the units of output and price respectively, and a, b,  and  are positive constants. If
the government imposes a tax at the rate of t per unit of output, what tax rate would lead to the
total tax to be the maximum ?
(b) For the production function given below, show that the marginal products of the factors depend
only on the ratio of factors.
2hlk – al 2 –  2
x = f(l, k) =
el  dk
Also, determine the degree of homogeneity and verify Eulcr's theorem for the function.
Or
A firm sells two products. The annual total revenue R behaves as a function of the number of
units sold. Specifically,
R = 400x – 4x2 + 1960y – 8y2
Where x and y equal, respectively, the number of unit sold of each product. The cost of
producing two products is–
C = 100 + 2x2 + 4y2 + 2xy
(i) Determine the number of units which should be produced and sold in order to maximise annual
profit.
(ii) What is total revenue equal to at the output level determined in (i) ?
(iii) What is the maximum profit ?
(c) The demand and supply functions under pure competition are P = 1600 – x2 and P = 2x2 + 400
respectively. Find the consumers' and producers' surplus.
Or
The cost of a truck engine overhaul is Rs. A, and the operating costs are at the rate of Rs. a + bx
per km, where the engine has run x kms since the last overhaul. If the engine is overhauled at M
kms, find the total cost function, C.
2. (a) (i) A 24 hour supermarket has the following minimal requirements for cashiers :
Period: 1 2 3 4 5 6
Time of day. 3-7 7-13 11-15 15-19 19-23\ 23-03
(24 hour clock)
Minimum No.
Required 7 20 14 20 10 5
Period 1 follows immediately after period 6. A cashier works eight consecutive hours: starting at
the beginning of one of the six periods. To determine a daily employee work sheet which
satisfies the requirements with the least number of personnel, formulate the problem as a linear
programming problem.
(ii) Write the dual to the following LPP :
Max. z = 20x1 + 15x2 + 18x3 + 10x4
Subject to 4x1 – 3x2 + 10x3 + 4x4  60
x1 + x2 + x3 = 27
–x2 + 4x3 + 7x4  35
x1, x2, x3  x4 : unrestricted in sing.
Or
(a) A manufacturer makes three types of decorative tensor lamps; model 1200, model 1201, and
model 1202. The requirement of raw materials for all lamps is the same, while the production
time for each differs. Each model 1200 lamp requires 0.1 hr of assembly time, 0.2 hr of wiring
time, and 0.1 hr of packaging time. The model 1201 requires 0.2 hr of assembly, 0.3 hr of wiring
time, and 0.1 hr of packaging time. The model 1202 requires 0.3 hr of assembly, 0.4 hr of wiring
time, and 0.1 hr of packaging time. The manufacturer makes a profit of Rs. 120 on each model
1200 lamp, Rs. 190 on each model 1201 lamp, and Rs. 210 on each model 1202 lamp. The
manufacturer can schedule up to 80 hrs of assembly labour, 120 his of wiring labour, and 100
hrs of packaging labour.
Assuming that all lamps can be sold, determine the optimal quantises of each model and the
marginal values of each of the resources.
Write the dual of the given problem and obtain optimal values of the\dual variables.
(b) The manager of a car rental company is planning its maintenance programme for the next year
and wants to determine the needs for certain repair parts of the expected costs. The company
rents out three types of cars : large, medium and small. The matrix, N gives the number of each
size of cars available for renting in the four regions.
Re gions
I II III IV
160 150 100 120  Laree
N   400 300 100 400  Medium
500 200 150 300  Small
Because of cost and frequency of replacement, the manager is concentrating on four repair parts
in particular : fan belt spark plugs, batteries and tyres. On the basis of past records the average
number of repair parts needed per car during a year are given in matrix R. Further, the matrix C
gives the cost per unit for the four parts.
L arg e Medium Small
 1.7 1.6 1.5 Fan bells
12.0 8.0 5.0 Spark Plugs
R = 
 0.9 0.75 0.5 Batteries
 
 4.0 6.5 6.0  Tyres
C = (125 80 300 650)
Through matrix operations, compute :
(i) the total demand for each type of car,
(ii) total demand for each repair part needed for the fleet, and
(iii) total combined cost for all repair parts.
Or
(c) For a three-sector economy, the input-output coefficients, aij's are as given below : a11 = 0.5, a21
= 0.2, a31 = 0.1, a31 = 0.1, a12 = 0.6, a22 = 0.2, a32 = 0.1, a23 = 0.2 and a33 = 0.6
(i) Using Simon-Hawkins conditions, test whether the system is viable.
(ii) Determine the gross output for the three sectors when the find demand values for them are
21,000,42,000 and 63,000 units respectively,
(iii) Determine the new level of output if the final demand increases by 1500, 3000 and 4500 units.
3. Answer any two parts from the following:
(a) Calculate the present value of an annuity of Rs. 30,000 per annum, assumed to be payable
continuously for 10 years at the rate of interest of 8 per cent per annum compounded
continuously.
(b) A machine costing Rs. 80,000 would reduce to Rs. 20,000 in 8 years. Find the rate of yearly
depreciation, given that depreciation is calculated using diminishing balances method.
(c) Write expression to find the required value in each of the following cases :
(i) To find the amount, A, of the principal, P, invested for n1, years at a rate of interest of r1 per
annum compounded quarterly, and next n2 years at the rate r2 per annum compounded annually.
(ii) To obtain the time it will take to triple an amount, P, invested at a rate of r per year compounded
quarterly.
(iii) To calculate the effective rate r per annum when the normal rate is i and the compounding is
done m times a year.
B.Com (Hons) IInd Year
Paper IX—BUSINESS MATHEMATICS
Assignment–VIII (2001)
Answer all questions. Use of simple calculator is allowed.
1. (a) If demand equation for a certain commodity is q = 500 – 2p (for 0  p  250),
(i) Then determine where the demand is elastic, inelastic and of unit elasticity with respect to price.
(ii) Use the result of part (i) to determine the intervals of increase and decrease of the revenue
function and the price at which revenue is maximized.
Or
A bicycle manufacturer buys 6000 tyres in a year from a distributor. The ordering cost is Rs. 20
per order, the storage cost is 96 paisa per tyre per year. Each type costs Rs. 25. Suppose that the
tyres are used at a constant rate throughout the year. Using calculus, find the Economic Order
Quantity (EOQ). If the tyre requirement increases fourfold and other conditions remain the
same, what will be new EOQ?
(b) Show that the production function given below has constant elasticity of substitution :
x = f (l, k) = {k– + (1 – )l–}–1/
Where x is the total output obtained by using l and k units of labour and capital respectively and
a and Q are constants.
Or
Given the demand function of two commodities as
Q1 = 2000 + 400 50p2
p1  3

And Q2 = 2000 – 100p2 + 500


p1  4
(i) Find the nature of commodities.
(ii) Calculate the four partial elasticities of demand at p1 = 5 and p2 = 1.
(c) Suppose that when it is x years old, an industrial machine generates revenue at the rate or R' (x)
= 6025 – 8x2 rupees per year and results in costs that accumulate at the rate of C' (x) = 4681 +
13x2 rupees per year:
(i) For how many years is the use of the machine profitable ?
(ii) What are the net earnings generated by the machine during its period of profitability ?
Or
In the year 2000, the annual world use of coal was 4.2 billion tons. The annual rate of depletion
of coal is 10 percent. Assuming this rate stays the same in the future :
(i) How many tons will the world use from 2000-20 10?
(ii) How long will it take to use all the available coal, if it is known that in 2000 there were 660
billion tons of proven reserves?
2. (a) (i) Shyam an Agriculturist, has a farm with 125 acres. He produces radish, peas and potato.
Whatever he raises is fully sold in the market? He gets Rs. 5 for radish per kg., Rs. 4 for peas
per kg and Rs. 5 for potato per kg. The average yield is 1500 kg of radish per acre, 1800 kg of
peas per acre and 1200 kg of potato per acre. To produce each 100 kg of radish and peas and to
produce each 80 kg of potato a sum of Rs. 12.50 has to be used for manure. Labour required for
each acre to raise the crop is 6 man days for radish and Potato each and 5 days for peas. A total
of 500 man days of labour at a rate of Rs. 40 per man day are available. Formulate this as a
linear programming model to maximize the Agriculturist's total profit.
(ii) Solve the following L.P.P. by graphic method :
Maximize Z = 2.75x1, + 4.15x2
Subject to:
2x1 + 2.5x2 100 4x1 + 8x2  160
7.5x2 + 5x2 150 x1, x2 0.
Or
'X', A boat manufacturing company, makes three different kinds of boats. All can be made
profitably in this company, but the company's production is constrained by the limited amount
of labour, wood and screws available each month. The director will choose the combination of
boats that maximizes his revenue in view of the information given in the following table :
Input Row Boat Canoe Kayak Monthly
Availability
Labour Wood 12 7 9 1260 hours
Board (feel) 22 18 1619008
Screws (kg) 2 4 3 396kg
Selling price Rs. 4,000 2,000 5,000
(i) Formulae the above as a linear programming problem,
(ii) Solve it by Simplex method.
(iii) Which, if any of the resources are not fully utilized ? It so, how much is the spare capacity left ?
(iv) What are the shadow prices ?
(v) State the dual of the formulated problem
(b) A baker makes bread, sweet patties and biscuits. He requires flour, egg, sugar, milk and yeast
for his preparations. The requirement of these basic items for making the bread, sweet patty and
biscuit is as follows :
Bread S\veet Patty Biscuit
Flour 250g 50 g 8g
Egg 0 1/2 1/4
Sugar 125g l00g 25g
Milk 0.05 litre 0.02 litre 0.01 litre
Yeast ¼ cake 1/8 cake 0
The baker buys the flour for Rs. 2.20 per kg and sugar for Rs. 2.40 per kg at controlled prices.
An egg costs him Rs. 0.25 a litre of milk Rs. 1.80 and a cake of yeast Re. 0.80. What is the cost
of making a bread, a sweet party and a biscuit ? Use matrix system.
Or
Consider an economy consisting of three sectors; agriculture, manufactures and services. The
hypothetical flow of goods and services in physical units is summarized in the following table:

Agriculture Manufactures Services Final


Demand
Agriculture 60 180 0 120
Manufactures 30 60 30 180
Services 0 40 10 100
If the demand charges to [150, 160, 180], what will be the new output? (Calculate upto two
decimals).
Q.3. Answer any two parts from the following:
(a) A company 'X' issued a convertible bond on 1-1-1996, each bond valued at Rs. 1,000. The rate
of interest is 10% per annum payable half yearly till 31 -12-2000. In addition, at the end of 5th
year, the bondholder is entitled to 3 equity shares at a price of Rs. 200 per share and a refund of
Rs. 400. It is expected that the market price per share at the end of 5 the year would be Rs. 300.
Calculate the yield, to maturity of the convertible bond.
(b) Mr. Ram, a businessman, plans to shift his business from Delhi to Faridabad because of closure
of certain type of industries in Delhi. He has to make a choice between two machines, both of
which are designed to improve operations by saving on labour costs. Machine A costs Rs. 9,750
and will generate an annual labour savings of Rs. 2,400. Machine B costs Rs. 7,750 and will
save Rs. 2,300 annually. Machine A has a useful life of 8 years while Machine B has a useful
life of 6 years only. If time value of money is 10% per annum, which machine is preferable?
(Assume annual compounding and that the saving is realized at the end of the year).
(c) Mohan has first purchased a house for Rs. 7,00,000 and has made a down payment of Rs.
1,50,000. He proposes to repay the balance in 25 years by monthly installments at 9%.
(i) What are the monthly payments ?
(ii) What is total interest payment ?
B.Com (Hons) IInd Year
Paper IX—BUSINESS MATHEMATICS
Assignment–IX (2002)
Answer all questions. Use of simple calculator is allowed.
1. (a) Show using calculus that profit of a monopolist is maximum when :
(i) MR = MC;and
(ii) Slope of MR < Slope of MC.
(b) The elasticity of demand of a commodity is given by
5p
 p  2 p – 3
Where p is price. Find the demand function, if quantity demanded is 5 at p = 3.
(c) Show that the following production function has the constant elasticity of substitution:
x = f (l, K) = ak (1 – a ) l–1/
where x is total output obtained by using l and k units of labour and capital respectively and 
and  are constants.)
Or
(a) The cost of fuel in running a vehicle is proportional to the square of the speed and is Rs. 45 per
hour for a speed of 15 km per hour. Other expenses amount to Rs. 500 per hour. Find the most
economical speed for the vehicle and also the minimum cost for a distance of 100 km.
(b) Under pure competition for a commodity, the demand and supply laws are
8 1
p = –2 and p = (x + 3)
x 1 2
respectively (where p is price and x quantity). Find the consumer surplus and producer surplus.
(c) Define the degree of homogeneity of a function and state Euler's theorem. Show that the
production function
q = f (l, k) = lk
(where q is total output obtained by using l and k units of labour and capital respectively) gives
constant returns to scale and diminishing returns to inputs.
2. (a) In the context of linear programming, what do you mean by no optimal solution, multiple
optimum solution and unbounded optimum solution ? How would you recognise these in the
optimum simplex tableau?
(b) A hypothetical economy produces only two committees, X and Y. The two commodities serve
as intermediate inputs in each other's production. To produce a unit of X. 0.6 unit of X and 0.1
unit of Y are needed. Similarly, to produce a unit of Y, 0.7 unit of X and 0.4 unit of Y are
needed. Two and five labour-days are required to produce a unit of X and a unit of Y
respectively.
The wage rate is Rs. 70 per man-day, The economy needs 100 units of X and 50 units of Y for
final consumption,
You are required to :
(i) examine the viability of input-output system using Hawkins-Simon conditions;
(ii) Calculate the gross output of each of the two commodities in tons;
(iii) Determine the equilibrium prices; and
(iv) Find the total value added.
Or,
(a) A firm produces chairs, tables and cupboards, each requiring three types of raw material-timber,
nails and varnish. You are given below, the units of different raw materials required for
producing one unit of each product:

Product Timber Nails Varnish


(c.ft.) (dozen) (litres)
Chair 0.7 2 1
Table 1 4 1.5
Cupboard 3,2 6 2

If the firm produces 300 units of each product, find the quantity of each raw material using
matrix algebra.
(b) Alpha Heavy Engineering company produces earthmovers and harvesters. Each product passes
through two assembly departments A and B, which, respectively, have 300 hours and 320 hours
of available time for the next month's production. Each earthmover requires 20 hours in
department A and 40 in department B and each harvester requires 30 hours in department A and
20 in department B. The two products are tested in a third department. Each earthmover is given
60 hours of testing and each harvester 20, and as per the agreement with the labour union, the
total labour hours devoted to testing cannot fall below 270. The management has the operating
policy of manufacturing at least one harvester for every two earthmovers produced. A major
customer has placed an order for a minimum of 5 earthmovers and harvesters (in any
combination, whatever) for next month, and so., at least that many must be produced. Each
earthmover gives a profit of Rs. 10,000 and each harvester Rs, 8,000.
Formulate the "hove as a linear programming problem and solve the same graphically.
3. Answer any three of the following:
(a) "Interest and discount are the two methods of looking at the same problem" Comment. Denoting
the effective rate of discount, nominal rate of discount and force of discount by d, d(m) and '
respectively, write the relationship among them. Show that the force of discount (') is equal to
the force of interest (). (The m in d(m) shows the number of times the rate is convertible per
annum, not the exponent.)
(b) Find, for each of the following, the amount to which Rs. 100 will accumulate:
(i) at the rate of interest 12% per annum compounded quarterly for 10 years,
(ii) at the force of interest 3% per annum for 3.5 years.
(iii) at the effective rates of interest 3% per annum for 10 years, 4% per annum for 4 years and 5%
per annum for 2 years.
(iv) at the rate of interest corresponding to 3% per annum effective rate of discount for 8 years.
What constant force of interest would produce the same amount after 16 years as the rates in
(iii) above ?
(c) The debenture holders of a company receive interest at 10% per annum payable half-yearly on
30th June and 31st December and an additional interest at 2% per annum payable annually on
31st December each year. If an investor purchases one debenture of Rs. 100 for Rs. 90 on 1st
January, find his yield expressed as an effective rate of interest per half-year. What would be the
yield per annum ?
(d) If the present value and amount of an ordinary annuity of Re. 1 per annum for n years are Rs.
8.1109 and Rs. 12.0061 respectively, find the rate of interest and the value of n without
consulting the compound interest table.
(e) A loan of Rs. 10,000 is to be repaid by equal annual installments of principal and interest over a
period of 20 years. The rate of interest is 3% per annum effective. Find:
(i) The annual installment;
(ii) The capital contained in 8th installment; and
(iii) The principal repaid after 12 installments have been paid.
B.Com (Hons) IInd Year
Paper IX—BUSINESS MATHEMATICS
Assignment–X (2003)

Answer all questions. Use of simple calculator is allowed.


1. (a) The number of minutes it takes a worker to pack a product is given by:
50 – 2t
M (t) =
7t – 1
Where t is the experience of the worker measured in weeks.
(i)Is this function continuous at t = 10 ?
(ii)Is this function continuous at t = 1/7 ?
(iii) Is this function continuous at t = 25 ?
Interprete the results.
Or
Using calculus, show that the profit of a monopolist is maximum when MR equals MC and the
slope of MR is less than the slope of MC.
(b) After producing. 35 units of a product, the production manager determine that the production
facility is following the learning curve of the form f (x) = 300 – 190 e–2x, where f (x) is the rate
of labour hours required to produce xth unit. How many labour hours would be required to
produce additional 25 units?
Or
60 – 10 p 20
The demand function for a monopolist is x = and his average cost function is
3 x
+ 1 + 0.2x,where p and x refer to the price and quantity of the commodity respectively.
Determine consumer surplus.
(c) A production function is given by
Q = AL1/3 K1/3
Where L is labour and K is capital.
(i) Find the behaviour of marginal product of each factor.
(ii) What is the nature of returns to scale ?
(iii) Show that the total product is not exhausted if each factor is paid a price equal to its
marginal product.
Or
Find the marginal rate of technical substitution and the elasticity of substitution for the
following production function:
1
x = f (l,k) = k  (1 – )l  
– –

(where x is the total output obtained by using l and k units of labour and capital respectively.)
2. (a) An economy consists of two sectors-manufacturing and agriculture. To produce one unit of
manufacturing output, 0.3 unit of manufacturing goods and 0.01 unit of agriculture goods are
required as inputs. One unit of agriculture output requires 40 units of manufacturing goods and
nil unit of agricultural goods as inputs. A unit of manufacturing goods requires 4 man-hours of
labour and a unit of agricultural output requires 100 man-hours of labour. Calculate the total
labour requirement if 50 million units of manufacturing goods and 2 million units of agricultural
goods need to be produced for final consumption.
Supposing the wage rate as Rs. 10 per man-hour, calculate the equilibrium prices of
manufacturing goods and agricultural output.
Or
ABC Co. has two service departments S1 and S2 and four production departments P1, P2, P3 and
P4. Overhead is allocated to production department for inclusion in the stock valuation. The
analysis of benefits received by each department during the last quarter and the direct overhead
expenses incurred by each department were:
Percentage to be allocated to Deptts.
Service Deptt. S1 S2 P1 P2 P3 P4

S1 0 20 30 25 15 10
S2 30 0 10 35 20 5
Direct Overhead Exp. 20 40 25 30 20 10
(in thousand of Rs.)

(i) Express the total overhead of the service department in the form of simultaneous equations.
(ii) Express these equations in a matrix form and solve for total overhead of service
departments using matrix inverse method.
(iii) Determine total overhead to allocated from each S, and S, to the production departments.
(b) In case of Linear Programming Problems, what is a feasible solution and an infeasible solution?
Describe each giving an imaginary graph.
Solve the following linear programming problem by using simplex method :
Maximise z = 8x + I2y
Subject to
x+y = 5
A firm assembles and sells two different types of outboard motors A and B using four resources.
The production process can be described as follows :
Resources Capacity Per Month
I Motor Unit Shop of the two 400 type A or 250 type B units or any linear
combination
II Type A gear and drive shop resource 175 type A units
III Type B gear And drive shop Resource 225 type B units
IV Final Assembly Resource 200 type A units or 350 type B units or any
linear combination of the two
Type A units bring in a profit of Rs. 90 each and Type B units Rs. 60 each.
Formulate the above as a linear programming problem to Maximise profit and solve the same by
graphic method.
Q. 3. Answer any three of the following
(a) (i) A certain sum of money is invested at 4% compounded annually. The interest for second
year is Rs. 25. Find the interest for the third year.
(ii) A sum of money is put at compound interest for two years at 20% per annum. It would fetch
Rs. 482 more, if the interest were payable half yearly than if it were payable yearly. Find the
sum.
(b) What should be the monthly sales volume of a company if it desires to earn a 12% annual return
convertible monthly on its investment of Rs. 2.00.000? Monthly costs are Rs. 3,000. The
investment will have eight year life with no scrap value.
(c) A machine depreciates at the rate of 10% per annum for the first two years and then 7% per
annum for the next 3 years, depreciation being calculated on the diminishing value. If the value
of the machine be Rs. 10,000 initially, find the average rate of depreciation and the depreciated
value of the machine at the end of the fifth year.
(d) Explain the concepts of effective rate of interest, nominal rate of interest and force of interest.
Bring out the relationship among them.
(e) Derive, by using calculus, the present value of an annuity of Re. 1 per annum assumed to be
payable continuously for n years at the rate of interest  per rupee per annum convertible
continuously.

You might also like